Leadership 3

¡Supera tus tareas y exámenes ahora con Quizwiz!

If untreated, squamous cell carcinoma of the external ear can spread through the temporal bone causing a) diplopia. b) motor impairment. c) facial nerve paralysis. d) nystagmus.

facial nerve paralysis. Correct Explanation: If untreated, squamous cell carcinomas of the ear can spread through the temporal bone, causing facial nerve paralysis and hearing loss.

The nurse should explain that the most common cause for the unhappiness some children experience when first entering school is due to which factor? a) poor language development b) emotional maladjustment c) social isolation d) feelings of insecurity

feelings of insecurity Explanation: The child entering school is moving into a new environment after having experienced security at home. Unhappiness with resulting feelings of insecurity is a normal response to the lost sense of security. Social isolation, emotional maladjustment, and poor language development all suggest a psychosocial disturbance and should not play a role in a normal child's unhappiness about entering school.

The presence of crystals in synovial fluid obtained from an arthrocentesis is indicative of a) degeneration. b) infection. c) gout. d) inflammation.

gout. Correct Explanation: The presence of crystals is indicative of gout, and the presence of bacteria is indicative of infective arthritis.

When planning the care for a client who is being abused, which measure is most important to include? a) helping the client develop a safety plan b) teaching the client about abuse and the cycle of violence c) being compassionate and empathetic d) explaining to the client the client's personal and legal rights

helping the client develop a safety plan Explanation: The client's safety, including the need to stay alive, is crucial. Therefore, helping the client develop a safety plan is most important to include in the plan of care. Being empathetic, teaching about abuse, and explaining the person's rights are also important after safety is ensured

A client diagnosed with tuberculosis is taking the prescribed chemotherapy of isoniazid, rifampin, and pyrazinamide. Although side effects are rare, the nurse should assess the client for which side effect of this drug combination? a) ototoxicity b) optic neuritis c) hepatotoxicity d) nephrotoxicity

hepatotoxicity Correct Explanation: The major side effect of these three drugs is liver toxicity. While the client is taking these drugs, the nurse should carefully monitor the client's liver function tests.

A newly admitted pt with major depressive disorder isolates self in room and stares out the window. Which intervention would be most appropriate to implement when first establishing a nurse-client relationship? 1 sit with the pt and offer self frequently 2 notify the client of group therapy schedule 3 introduce the client to others on the unit 4 help the client identify stressors of life the precipitate a crisis

1

A suicidal jewish client is admitted to an in patient unit 2 days after the death of a parent. Which intervention should the nurse include 1 allow the pt time to mourn the loss during this time of shiva 2 distract the client from the loss and encourage participation in group 3 teach the pt alternative coping skills to deal with grief 4 discuss positive aspects the client has in his life to build on strengths

1

Which client is at highest risk for the diagnosis of major depressive disorder? 1 24 yo married women 2 64 yo single woman 3 30 yo single man 4 70 yo married man

1

Which intervention takes priority when working with a newly admitted client experiencing suicide ideations? 1 monitor the client at close, but irregular intervals 2 encourage the client to participate in group therapy 3 enlist friends and family to assist the client in remaining safe after discharge 4 remind the client that it takes 6-8 weeks for antidepressants to be fully effective

1

A newly admitted client with major depressive disorder has a history of 2 suicide attempts by hanging. Which dx takes priority 1 risk for violence directed at others 2 social isolation 3 risk for suicide 4 hopelessness

3

a client is exhibiting behavioral symptoms of depression. Which charting entry would appropriately document these symptoms? 1 rates mood as 4/10 2 expresses thoughts of poor self esteem during group 3 became irritable and agitated on waking 4 rates anxiety as 2/10 after receiving lorazapam

3

a client with major depressive disorder is being considered for ECT. Which client teaching should the nurse prioritize? 1 empathize with the client about fears regarding ECT 2 monitor for any cardiac alterations to prevent possible negative outcomes 3 discuss with the client and family expected short term memory loss 4 inform the client that injury related to induced seizure commonly occurs

3

The client diagnosed with major depression who attempted suicide is being discharged from the psychiatric facility after a 2-week stay. Which discharge inter- vention is most important for the nurse to implement? 1. Provide the family with the phone number to call if the client needs assistance. 2. Encourage the client to keep all follow-up appointments with the psychiatric clinic. 3. Ensure the client has no more than a 7-day supply of antidepressants. 4. Instruct the client not to take any over-the-counter medications without consult- ing with the HCP.

3 Providing phone numbers for the client and family is an intervention that the nurse could implement, but it is not prior- ity over the psychological and physical safety of the client. Follow-up appointments are important for the client after being discharged from a psychiatric facility, but it is not priority over the psychological and physical safety of the client. Ensuring the psychological and physi- cal safety of the client is priority. As antidepressant medications become more effective, the client is at a higher risk for suicide. Therefore, the nurse should ensure that the client cannot take an overdose of medication. This is an appropriate intervention, but it is not priority over the psychological and physical safety of the client.

which statement describes a major difference between a client diagnosed with major depressive disorder and a client diagnosed with dysthymic disorder? 1 a client diagnosed with dysthymic disorder is at a higher risk for suicide 2 a client diagnosed with dysthymic disorder may experience psychotic features 3 a client diagnosed with dysthymic disorder experiences excessive guilt 4 a client diagnosed with dysthymic disorder has symptoms for at least 2 years

4

At what age is an individual most at risk for acquiring acute lymphocytic leukemia (ALL)? a) 20 to 30 years b) 60 to 70 years c) 40 to 50 years d) 4 to 12 years

4 to 12 years Correct Explanation: The peak incidence of ALL is at 4 years of age. ALL is uncommon after 15 years of age. The median age at incidence of CML is 40 to 50 years. The peak incidence of AML occurs at 60 years of age. Two-thirds of cases of chronic lymphocytic leukemia occur in clients older than 60 years of age

A client with bipolar disorder begins taking lithium carbonate (lithium), 300 mg four times a day. After 3 days of therapy, the client says, "My hands are shaking." The best response by the nurse is which of the following? A "Fine motor tremors are an early effect of lithium therapy that usually subsides in a few weeks." B "It is nothing to worry about unless it continues for the next month." C "Tremors can be an early sign of toxicity, but we'll keep monitoring your lithium level to make sure you're okay." D "You can expect tremors with lithium. You seem very concerned about such a small tremor."

A

A 90-year-old woman has a staphylococcus infection in her decubitus ulcer. Staphylococcus is the: a) Environment b) Agent c) Disease d) Host

Agent Correct Explanation: The agent is any factor that leads to illness.

The American Nurses Association's Code of Ethics for Nurses a) Delineates nurses' conduct and responsibilities b) Plays an important role in legal proceedings c) Serves to establish personal ethics for nurses d) Serves as a guideline for all healthcare

Delineates nurses' conduct and responsibilities Explanation: The ANA recently revised the Code of Ethics for Nurses that delineates the conduct and responsibilities expected of all nurses in their nursing practices.

The nurse notices that a client diagnosed with major depression and social phobia must get up and move to another area when someone sits next to her. Which action by the nurse is appropriate? a) Question the client about her avoidance of others. b) Convey awareness of the client's anxiety about being around others. c) Have nursing staff follow the client as moves away. d) Ignore the client's behavior.

Convey awareness of the client's anxiety about being around others. Correct Explanation: The nurse conveys empathy and awareness of the client's need to reduce anxiety by showing acceptance and understanding to the client, thereby promoting trust. Ignoring the behavior, questioning the client about her avoidance of others, or telling other clients to follow her when she moves are not therapeutic or appropriate.

Which of the following is a formal systematic study of moral beliefs? a) Morality b) Ethics c) Veracity d) Fidelity

Ethics Correct Explanation: Ethics is the formal, systematic study of moral beliefs. Veracity is the obligation to tell the truth and not to lie or deceive others. Fidelity is promise keeping. Morality is the adherence to informal personal values

A dying patient tells the nurse that he doesn't want to see his family because he doesn't want to cause them more sadness. Which action by the nurse is most appropriate? a) Allow the patient time for quiet reflection b) Help the patient clarify his values c) Arrange a meeting between the family and the patient d) Educate the patient on death and dying concepts

Help the patient clarify his values Correct Explanation: Values clarification is a method of self-discovery by which people identify their personal values and value rankings. The patient's value of family may be obscured because of his overwhelming need to protect his family

When planning nursing interventions, the nurse must review the etiology of the problem statement. The etiology: a) Identifies factors causing undesirable response and preventing desired change b) Identifies the unhealthy response preventing desired change c) Suggests patient goals to promote desired change d) Identifies patient strengths

Identifies factors causing undesirable response and preventing desired change Correct Explanation: The cause of the patient health problem is referred to as the etiology. The problem statement of the nursing diagnosis suggests the patient goals, and the cause of the problem (etiology) suggests the nursing interventions. Identifying the unhealthy response preventing desired change would occur during the evaluation phase of the nursing process. Patient strengths are identified during the nursing diagnosis phase.

After educating a class about depression, the instructor determines that additional class time is needed for teaching when the class identifies which of the following as an indicator of depression? a) Feelings of worthlessness b) Sleep disturbances c) Increased thoughts about death d) Increased concentration

Increased concentration Correct Explanation: With depression, the patient often has difficulty concentrating. Feelings of worthlessness, thinking about death or suicide, and sleeping difficulties are associated with depression

A hospital owned by a Catholic order of nuns will not allow tubal ligations to be performed. This is considered to be a) Personal morality b) Institutional policy c) Personal values d) Legal obligation

Institutional policy Correct Explanation: Institutional policies are guidelines developed by healthcare institutions to direct professional practice.

A patient recently diagnosed with pancreatic cancer asks the nurse not to share the diagnosis with her family members. After visiting the patient, the patient's daughter approaches the nurse and states, "Mom just did not seem herself today. Are biopsy reports back and do they confirm pancreatic cancer?" What is the best response from the nurse to patient's daughter? a) It is unethical of me to discuss biopsy results with anyone but the patient involved. b) It is unethical and illegal for me to give you the biopsy results; please ask your mother. c) It is unethical and illegal for me to discuss your mother's medical information with you. d) It is illegal for me to discuss biopsy results with anyone but the patient involved.

It is unethical and illegal for me to discuss your mother's medical information with you. Correct Explanation: Providing a firm response in explaining the need to protect patient information is one strategy to aid the nurse in ethical decision-making. The U.S. Department of Health and Human Services (DHHS) provides for patient confidentiality. Violations of a patient's confidentiality could result in criminal or civil litigation. While it is unethical/illegal to discuss biopsy results with the daughter, statements by the nurse indicating biopsy results are back but cannot be shared indirectly provide the daughter with confidential information. Encouraging the daughter to ask her mother about the biopsy results indirectly provides the daughter with information that the mother knows the biopsy results

Which of the following are byproducts of fat breakdown, which accumulate in the blood and urine? a) Creatinine b) Hemoglobin c) Ketones d) Cholesterol

Ketones Correct Explanation: Ketones are byproducts of fat breakdown, and they accumulate in the blood and urine. Creatinine, hemoglobin, and cholesterol are not byproducts of fat breakdown.

Erythropoietin growth factor increases production of which of the following? a) White blood cells b) Plasma c) Platelets d) Red blood cells

Red blood cells Correct Explanation: Erythropoietin growth factor increases the production of red blood cells, thus decreasing the symptoms of anemia.

Which statement made by an adolescent who has just begun taking an antidepressant would indicate the need for further teaching? a) "After a week of taking my antidepressant, I can sleep a little better—6 hours or so each night." b) "A week ago when I started my antidepressant, I did not care about eating. Now I want to eat a bit more." c) "Now that I have had a week of my antidepressant, it is a little easier to get up in the morning." d) "Now that I have been taking my antidepressant for 1 week, I am going to feel better about myself."

"Now that I have been taking my antidepressant for 1 week, I am going to feel better about myself." Correct Explanation: In the first week or so of taking an antidepressant, the vegetative symptoms of depression (poor sleep, appetite, and energy level) improve. However, it takes 3 to 4 weeks for improvement in self-concept/self-esteem to take place

Which symptom would the nurse expect to assess in a client experiencing serotonin syndrome? 1 confusion, restlessness, tachycardia, labile blood pressure, and diaphoresis 2 hypomania, akathisia, cardiac arrhythmias, and panic attacks 3 dizziness, lethargy, headache, nausea 4 orthostatic hypotension, urinary retention, constipation, and blurred vision

1

A health care provider (HCP) prescribes a lengthy x-ray examination for a client with osteoarthritis. Which action by the nurse would demonstrate client advocacy? a) Contact the HCP to determine if an alternative examination could be scheduled. b) Contact the X-ray department, and ask the technician if the lengthy session can be divided into shorter sessions. c) Request padding for the hard x-ray table. d) Request a prescription for acetaminophen prior to the examination.

Contact the X-ray department, and ask the technician if the lengthy session can be divided into shorter sessions. Explanation: Shorter sessions will allow the client to rest between the sessions. Changing the HCP's prescription to a different examination will not provide the information needed for this client's treatment. Acetaminophen is a nonopioid analgesic and an antipyretic, not an anti-inflammatory agent; thus it would not help this client avoid the adverse effects of a lengthy x-ray examination. Although the x-ray table is hard, it is not possible to provide padding and obtain the needed diagnostic x-rays

A client says to the nurse, "You are the best nurse I've ever met. I want you to remember me." What is an appropriate response by the nurse? A "Thank you. I think you are special too." B "I suspect you want something from me. What is it?" C "You probably say that to all your nurses." D "Are you thinking of suicide?"

D

The nurse observes that a client with bipolar disorder is pacing in the hall, talking loudly and rapidly, and using elaborate hand gestures. The nurse concludes that the client is demonstrating which of the following? A Aggression B Anger C Anxiety D Psychomotor agitation

D

A patient who is obese is exploring bariatric surgery options and presented to a bariatric clinic for preliminary investigation. The nurse interviews the patient, analyzing and documenting the data. Which of the following nursing diagnoses may be a contraindication for bariatric surgery? a) Deficient Knowledge Related to Risks and Expectations of Surgery b) Disturbed Body Image Related to Obesity c) Anxiety Related to Surgery d) Chronic Low Self-Esteem Related to Obesity

Deficient Knowledge Related to Risks and Expectations of Surgery Correct Explanation: It is expected that patients seeking bariatric surgery may have challenges with body image and self-esteem related to their obesity. Anxiety is also expected when facing surgery. However, if the patient's knowledge remains deficient regarding the risks and realistic expectations for surgery, this may show that the patient is not an appropriate surgical candidate

A client is diagnosed with posttraumatic stress disorder (PTSD). Which finding would the nurse most likely assess? a) Bradycardia b) Diaphoresis c) Muscle flaccidity d) Constricted pupils

Diaphoresis Correct Explanation: With posttraumatic stress disorder (PTSD), the client may exhibit diaphoresis with cold, clammy skin, dilated pupils, tachycardia or palpitations, and muscle tension. These physiologic findings are related to increased sympathetic nervous system activity, increased plasma catecholamine levels, and increased urinary epinephrine and norepinephrine levels.

A nurse is caring for a hospitalized patient who states: "I feel so sick all the time, my aura must be disturbed by all of these bad force fields." What is an appropriate NANDA diagnosis for this patient? a) Disturbed energy field b) Impaired coping c) Hopelessness d) Social isolation

Disturbed energy field Correct Explanation: Defining characteristics of the disturbed energy field relate to movement, sounds, temperature change, visual changes, and disruption. The aura the patient is referring to in this scenario is a vague, luminous glow surrounding something that may be an information containing electromagnetic field, which can be likened to the data contained within a computer. Social isolation would refer to a patient who is feeling isolated from others. Impaired coping refers to a patient who is having difficulty with dealing with a particular situation. Hopelessness refers to a patient who feels there is no hope for improvement from an illness or in life in general

Which of the following statements best conveys the concept of ethical agency? a) Ethical practice requires a skill set that must be conscientiously learned and nurtured. b) Ethical practice is best learned and fostered by surrounding oneself with people who exhibit ethical character. c) Individuals who enter the nursing profession often innately possess ethical characteristics. d) A nurse's understanding and execution of ethical practice is primarily a result of increased years of experience.

Ethical practice requires a skill set that must be conscientiously learned and nurtured. Explanation: Ethical agency must be cultivated in the same way that nurses cultivate the ability to do the scientifically right thing in response to a physiologic alteration. It is inaccurate to assume that it will passively develop from the presence of other ethical practitioners or from years of experience and it is not an innate characteristic of personality.

An elderly client takes 40 mg of Lasix twice a day. Which electrolyte imbalance is the most serious adverse effect of diuretic use? a) Hypernatremia b) Hypokalemia c) Hyperkalemia d) Hypophosphatemia

Hypokalemia Correct Explanation: Hypokalemia (potassium level below 3.5 mEq/L) usually indicates a defict in total potassium stores. Potassium-losing diuretics, such as loop diuretics, can induce hypokalemia.

A nurse is assessing a patient with posttraumatic stress disorder (PTSD) who is exhibiting physiologic manifestations. The nurse interprets these manifestations as being the result of which of the following? a) Decreased plasma catecholamine levels b) Increased sympathetic activity c) Increased parasympathetic activity d) Decreased urinary epinephrine levels

Increased sympathetic activity Correct Explanation: The physiologic responses associated with PTSD result from increased activity of the sympathetic nervous system, increased plasma catecholamine levels, and increased urinary epinephrine and norepinephrine levels.

Ayurvedic medicine has its roots in which of the following areas? a) China b) India c) Japan d) Southeast Asia

India Correct Explanation: Ayurvedic medicine has its roots in India.

A nurse is caring for a 15-year-old male recently diagnosed with osteosarcoma of the right femur with lung metastasis. He and his visiting parent are discussing their uncertainty about the need for surgery or other therapies. How can the nurse best demonstrate the advocacy role? a) Provide additional educational resources and materials. b) Notify the physician to meet with the family. c) Ask the client and his parent what they are unsure about. d) Refer the family to ancillary support services.

Notify the physician to meet with the family. Explanation: Discussing uncertainty about the need for treatment and surgery involves the issue of informed consent. The physician or provider who will perform the procedure must secure this consent. Asking what the family is unsure about, providing additional educational resources and materials, or referring the family to ancillary support services does not eliminate the need for the physician to speak with the family to obtain informed consent

What is true of nursing responsibilities with regard to a physician-initiated intervention (physician's order)? a) Nurses are not legally responsible for these interventions. b) Nurses are responsible for reminding physicians to implement orders. c) Nurses do carry out interventions in response to a physician's order. d) Nurses do not carry out physician-initiated interventions.

Nurses do carry out interventions in response to a physician's order. Correct Explanation: A physician-initiated intervention is initiated in response to a medical diagnosis but carried out by a nurse in response to a doctor's order. Both the physician and the nurse are legally responsible for these interventions

A nurse reporting for the scheduled shift finds an assignment that includes the nurse's aunt, who was admitted during the night with a fractured hip. What should the nurse do in response to the client assignment? a) Notify the supervisor that this is a relative but the relationship will not be a conflict. b) Notify the supervisor and provide care until another nurse can be assigned to the client. c) Ask the aunt if she would like the nurse to take care of her while in the hospital. d) Accept the assignment and not disclose the relationship with the client.

Notify the supervisor and provide care until another nurse can be assigned to the client. Correct Explanation: The nurse should notify the supervisor of the relationship with the client and ask to be reassigned. If no other nurse is immediately available, the nurse should provide the necessary care until another nurse can assume responsibility for the aunt's care. The other answers are incorrect because the nurse may not be able to ensure that the therapeutic nurse-client relationship can be maintained when caring for a family member.

During adolescence, values are primarily formed from a) Schools b) Work c) Parents d) Peers

Peers Explanation: In adolescence, peer groups are the primary source of value formation. The attitudes, beliefs, and behaviors that grow out of peer group relationships are powerful.

A client recovering from gastric bypass surgery accidentally removes the nasogastric tube. It is best for the nurse to a) Place the nasogastric tube to the level of the esophagus. b) Reinsert the nasogastric tube to the stomach. c) Document the discontinuation of the nasogastric tube. d) Notify the surgeon about the tube's removal.

Notify the surgeon about the tube's removal. Explanation: If the nasogastric tube is removed accidently in a client who has undergone esophageal or gastric surgery, it is usually replaced by the physician. Care is taken to avoid trauma to the suture line. The nurse will not insert the tube to the esophagus or to the stomach in this situation. The nurse needs to do more than just document its removal. The nurse needs to notify the physician who will make a determination of leaving out or inserting a new nasogastric tube.

A nurse is preparing to administer saquinavir, which is prescribed for a client who is HIV positive. The nurse integrates knowledge of this drug, identifying it as which of the following? a) Non-nucleoside reverse transcriptase inhibitor (NNRTI) b) Protease inhibitor (PI) c) Nucleoside reverse transcriptase inhibitor (NRTI) d) Fusion inhibitor

Protease inhibitor (PI) Explanation: Saquinavir is a PI. Enfuvirtide and maraviroc are examples of fusion inhibitors. NNRTIs include agents such as delavirdine and nevirapine. NRTIs include abacavir, didanosine, lamivudine, and zalcitabine

A client with end-stage respiratory failure is admitted to hospice for nursing care. What nursing goals will the nurse use to direct care? Select all that apply. a) Reduce anxiety level to mild or none. b) Increase tidal volume to 600 ml. c) Maintain oxygen saturation at 85%. d) Support end-of-life discussions. e) Control pain level at or below 3 of 10.

Reduce anxiety level to mild or none. • Control pain level at or below 3 of 10. • Support end-of-life discussions. Explanation: The focus of planning for clients at end of life moves from promotion and restoration of health to supportive care. This includes emotional and spiritual support, pain control, and comfort measures such as ice chips, positioning, and skin care. A client may not be restored to physical health, but may experience emotional and spiritual health

The nurse is caring for a patient who is Arabic. The nurse remembers learning that three elements are frequently used to identify diversity. Choose the three from the following list. a) Geographic area b) Age c) Skin color d) Religion e) Income

Religion • Skin color • Geographic area Correct Explanation: Cultural diversity has been identified in a number of ways including differences in skin color, religion, and geographic area.

A nurse finds that a fire has broken out in a client's room at the health care facility. Which of the following interventions is of the highest priority? a) Extinguish the fire. b) Rescue the client. c) Raise an alarm. d) Confine the fire.

Rescue the client. Correct Explanation: The first priority in case of fire is to rescue the client. As per the RACE principle of fire management, the rescue of the client is the first step, followed by raising an alarm, confining the fire, and finally, extinguishing the fire

Which of the following disorder is characterized by a butterfly-shaped rash across the bridge of the nose and cheeks? a) Rheumatoid arthritis b) SLE c) Polymyositis d) Scleroderma

SLE Correct Explanation: The most familiar manifestation of SLE is an acute cutaneous lesion consisting of a butterfly-shaped rash across the bridge of the nose and the cheeks. This type of rash does not characterize RA, scleroderma, and polymyositis

Which of the following surgical procedures involves taking a piece of silicone plastic or sponge and sewing it onto the sclera at the site of a retinal tear? a) Pneumatic retinopexy b) Pars plana vitrectomy c) Scleral buckle d) Phacoemulsification

Scleral buckle Correct Explanation: The scleral buckle is a procedure in which a piece of silicone plastic or sponge is sewn onto the sclera at the site of the retinal tear. The buckle holds the retina against the sclera until scarring seals the tear. The other surgeries do not use this type of procedure

Ethics is best defined as a) Dealing with conflict b) Basis for moral reasoning c) Decision making d) Standards of conduct

Standards of conduct Explanation: Ethics is the branch of philosophy dealing with standards of conduct and moral judgment.

Which of the following is the most common type of prostate surgery? a) Suprapubic prostatectomy b) TURP c) Retropubic prostatectomy d) Perineal prostatectomy

TURP Correct Explanation: TURP is the most common procedure used and can be carried out through endoscopy. Suprapubic, perineal, and retropubic prostatectomies are surgical procedures for the prostate, but are not the most common

When a nurse is planning discharge teaching for a patient admitted with a sickle cell crisis, which of the following should the nurse include in the teaching? a) Teach patients to receive pneumococcal and annual influenza vaccinations b) Teach patients to take a daily multivitamin with iron c) Teach patients to limit fluids to 2 quarts a day d) Teach patients to drink only one caffeinated beverage daily

Teach patients to receive pneumococcal and annual influenza vaccinations Correct Explanation: Patients with sickle cell anemia must treat infections promptly with appropriate antibiotics; infections, particularly pneumococcal infections, can be serious. These patients should receive pneumococcal and annual influenza vaccinations.

A nurse on the crisis team in the emergency department is caring for a client who is angry and is experiencing delusional episodes. The client says to the nurse, "I'm going to kill my wife and chop her up to get rid of her." What is the nurse's priority action in this situation? a) Ignore the remarks as delusional symptoms. b) Note it on the mental status form only. c) Notify the wife that she may be in danger. d) Include "risk for injury" on the care plan.

Notify the wife that she may be in danger. Correct Explanation: The client is making statements that may be acted on. The nurse is obliged to notify the wife that she might be in danger. If the nurse believes the statements reflect a new symptom, such as delusions, the attending psychiatrist should be contacted for further direction. The other options are incorrect because they do not protect the client's wife, whom he has clearly indicated he has a plan to harm.

If the dosage is inappropriate for a client, who is responsible? a) Medical technician b) Nurse c) Physician d) Pharmacist

Nurse Correct Explanation: Whereas physicians and other healthcare providers prescribe and pharmacists dispense therapeutic agents, it is the nurse's legal domain to administer medications in a safe and timely manner.

A client has a newly placed tracheostomy tube. The nurse should assess the client for which possible complication? a) damage to the laryngeal nerve b) acute respiratory distress syndrome c) pneumothorax d) decreased cardiac output

damage to the laryngeal nerve Explanation: Tracheostomy tubes are associated with several potential complications, including laryngeal nerve damage, bleeding, and infection. Tracheostomy tubes do not cause decreased cardiac output, pneumothorax, or acute respiratory distress syndrome

The nurse is participating in a discussion about controlled substances. Which statement, made by the nurse, indicates the nurse is aware of laws governing the distribution of controlled substances? a) "When a nurse abuses controlled substances in the workplace and gets help, she will not be charged with a criminal act." b) ?An impaired nurse is promptly punished by being terminated and having his or her license suspended.? c) ?The nurse is only at risk if she diverts medication from the client; using personal drugs decrease your risk.? d) ?Nurses are responsible for adhering to specific documentation about controlled substances.?

Nurses are responsible for adhering to specific documentation about controlled substances.? Correct Explanation: Nurses have specific responsibilities regarding controlled substances, including specific documentation. Violation of controlled substances at the workplace is serious and is considered a criminal act. Substance abuse is treatable and the objective is to detect and treat the problem early. It does not matter where the nurse obtains the drugs; she is still liable for her actions.

What is the best explanation for the way evidence based practice (EBP) has changed the way nursing care is delivered? a) Nurses now have to take part in research. b) Nursing care now incorporates research studies into patient care. c) Nursing care now uses EBP as a means of ensuring quality care. d) Nurses now spend time looking up the best way to give nursing care.

Nursing care now uses EBP as a means of ensuring quality care. Correct Explanation: The facilitation of EBP involves identifying and evaluating current literature and research, as well as incorporating the findings into patient care as a means of ensuring quality care. The other answers are incorrect because they are things that nursing has been doing even before EBP became a major force in the delivery of nursing care.

An example of a curative surgical procedure is a) a biopsy. b) the placement of gastrostomy tube. c) a face-lift. d) the excision of a tumor.

the excision of a tumor. Correct Explanation: An example of a curative surgical procedure is the excision of a tumor. A biopsy, face-lift, and the placement of a gastrostomy tube are not examples of curative surgical procedures.

Which is an initial sign of Parkinson's disease? a) tremor b) akinesia c) bradykinesia d) rigidity

tremor Correct Explanation: The first sign of Parkinson's disease is usually tremors. The client commonly is the first to notice this sign because the tremors may be minimal at first. Rigidity is the second sign, and bradykinesia is the third sign. Akinesia is a later stage of bradykinesia

A client about to undergo gastric bypass surgery calls the nurse into the room. The client whispers to the nurse concern that friends will learn about her upcoming surgery. She pleads with the nurse to keep her surgery a secret. Which response by the nurse is best? a) "I am not at liberty to discuss your case with anyone except those directly involved in your care unless you authorize me to do so." b) "I will avoid any questions." c) "I can't lie to them if they ask me." d) "I promise I won't tell anyone."

"I am not at liberty to discuss your case with anyone except those directly involved in your care unless you authorize me to do so." Correct Explanation: Federal legislation has been developed to protect the right of confidentiality. According to the Health Insurance Portability and Accountability Act (HIPAA) (U.S. Department of Health and Human Services [USDHHS], 2003), efforts must be made to protect each client's private information, whether it is transmitted by verbal, written, or electronic means of communication

Which statement by a client taking valproic acid for bipolar disorder indicates that further teaching about this medication is necessary? a) "I need to take the pills at the same time each day." b) "I can take the pills with food." c) "I can chew the pills if necessary." d) "I need to call my health care provider if I start bruising easily."

"I can chew the pills if necessary." Correct Explanation: Chewing the pill or capsule form of valproic acid can cause mouth and throat irritation and is contraindicated. Taking the pills at the same time each day is important to maintain therapeutic effectiveness of the drug. Taking the pills with food is appropriate if the client is experiencing gastrointestinal upset. Valproic acid may cause clotting problems; therefore, bruising should be reported.

Which client statement indicates the need for additional teaching about benzodiazepines? a) "I can stop taking the drug anytime I want." b) "Diazepam can make me drowsy, so I should not drive for a while." c) "I cannot drink alcohol while taking diazepam." d) "Diazepam will help my tight muscles feel better."

"I can stop taking the drug anytime I want." Correct Explanation: Diazepam, like any benzodiazepine, cannot be stopped abruptly. The client must be slowly tapered off of the medication to decrease withdrawal symptoms, which would be similar to withdrawal from alcohol. Alcohol in combination with a benzodiazepine produces an increased central nervous system depressant effect and therefore should be avoided. Diazepam can cause drowsiness, and the client should be warned about driving until tolerance develops. Diazepam has muscle relaxant properties and will help tight, tense muscles feel better

A nurse is caring for an older adult client with advanced Parkinson's disease. Which client statement about advance directives indicates a need for further instruction? a) "I don't really need to sign anything. I'm depending on my physician to tell my family what to do if something bad happens." b) "My family will take care of me. I've given my daughter durable power of attorney for health care." c) "I've signed the advance directive papers and will fight to maintain the highest quality of life until my time comes." d) "I know that I'll eventually be unable to make decisions. Signing an advance directive now will save my family grief."

"I don't really need to sign anything. I'm depending on my physician to tell my family what to do if something bad happens." Correct Explanation: The client requires additional teaching if the client states that he/she will depend on the physician to tell the family what to do in regards to his/her health. The client should not rely on the physician to tell the family what to do. The best way for the client to convey his/her health care wishes is to put them in writing in an advance directive. The client stating that he/she has designated his/her daughter to make health care decisions when the client cannot, that the client has signed an advance directive, or that the client knows an advance directive will help when he/she is unable to make decisions indicate that the client has made decisions about his/her end-of-life care

The nurse is attending a conference on evidence-based practice. Which statement by the nurse indicates further teaching is needed? a) "I can learn about evidence-based practice by reading professional nursing journals." b) "I must conduct research to validate the usefulness of my nursing interventions." c) "The Agency for Healthcare Research and Quality is a resource for evidence-based practice." d) "Nursing interventions should be supported by a sound scientific rationale."

"I must conduct research to validate the usefulness of my nursing interventions." Correct Explanation: Nursing interventions should be supported by a sound scientific rationale; however, nurses do not need to personally conduct research to establish the rationale for nursing interventions. Nurses can learn about evidence-based practice by reading professional nursing journals, attending nursing workshops, and consulting evidence-based practice resources, such as the Agency for Healthcare Research and Quality.

The client is receiving 6 mg of selegiline transdermal system every 24 hours for major depression. The nurse should judge teaching about selegiline to be effective when the client makes which statement? a) "I need to avoid using the sauna at the gym." b) "I can cut the patch and use a smaller piece." c) "I might gain at least 10 lb (4.5 kg) from the medication." d) "I need to wait until the next day to put on a new patch if it falls off."

"I need to avoid using the sauna at the gym." Correct Explanation: Selegiline transdermal system is the first transdermal monoamine oxidase inhibitor. The client needs to avoid exposing the application site to external sources of direct heat, such as saunas, heating lamps, electric blankets, heating pads, heated water beds, and prolonged direct sunlight because heat increases the amount of selegiline that is absorbed, resulting in elevated serum levels of selegiline. Cutting the patch and using a smaller piece will result in a decreased amount of medication absorption, most likely leading to a worsening of the symptoms of depression. The client should apply a new patch as soon as possible if one falls off to ensure an adequate amount of medication absorption. Selegiline is not associated with significant weight gain, although a weight gain of 1 to 2 lb (2.2 to 4.4 kg) is possible.

A client recently diagnosed with hyperparathyroidism demands to see what the physician has written about him in the chart. What is the nurse's best response? a) "Your chart is hospital property. I'm not allowed to let you see it." b) "Physicians use medical language. It might be difficult for you to understand what he has written." c) "I'll get the chart and set up a time for you to review it with your physician." d) "The chart might be difficult to understand. I suggest you review it with your physician."

"I'll get the chart and set up a time for you to review it with your physician." Correct Explanation: Every client has a right to access information the hospital has collected about him. However, it's in the client's best interests to have a knowledgeable professional present to explain complicated information and unfamiliar terminology the chart might include

A female client is admitted to a mental health unit with a diagnosis of depression and is participating in group sessions. She asks a male nurse if he is married or has a girlfriend. What is the best response by the nurse to maintain a therapeutic relationship? a) "Tell me how you knew that I was not married or had a girlfriend." b) "I'm curious about your question, but I want to know how you are feeling today." c) "It sounds as though you are interested in developing a relationship with me." d) "Group therapy is not the appropriate time to discuss my relationships."

"I'm curious about your question, but I want to know how you are feeling today." Correct Explanation: Nurses must practice in a manner that is consistent with providing safe, competent, and ethical care. If the nurse shared personal information with the client, the nurse would have crossed the boundary of a therapeutic relationship and changed the focus of the discussion from a client focus to a social focus. It is very important in all areas of care, but especially in the mental health setting, that the relationship between the nurse and the client has very clear boundaries and is client focused. The other options are incorrect because they do not follow the principles of a therapeutic nurse-client relationship.

After teaching nursing students about substance abuse and its effects on individuals and families, the instructor determines that additional teaching is necessary when the students state which of the following? a) "Substance abuse involves use of alcohol and illegal, prescribed, or over-the-counter drugs." b) "Individuals frequently engage in substance use and abuse to enhance their decision-making ability." c) "People experiencing substance abuse problems often have difficulty using adaptive behaviors." d) "Substance abuse is widespread, occurring in all types of settings."

"Individuals frequently engage in substance use and abuse to enhance their decision-making ability." Correct Explanation: Substance abuse refers to the use of alcohol and illegally obtained, prescribed, or over-the-counter drugs alone or combined in ineffective attempts to cope with the pressures, strains, and burdens of life. Thus, individuals with substance abuse often have difficulty identifying and implementing adaptive behaviors. Substance abuse occurs in all settings. Individuals who abuse substances are unable to make healthy decisions and to solve problems effectively

A patient seeking care because of recurrent heartburn and regurgitation is subsequently diagnosed with a hiatal hernia. Which of the following should the nurse include in health education? a) "Drinking beverages after your meal, rather than with your meal, may bring some relief." b) "Many patients obtain relief by taking over-the-counter antacids 30 minutes before eating." c) "Instead of eating three meals a day, try eating smaller amounts more often." d) "It's best to avoid dry foods, such as rice and chicken, because they're harder to swallow."

"Instead of eating three meals a day, try eating smaller amounts more often." Explanation: Management for a hiatal hernia includes frequent, small feedings that can pass easily through the esophagus. Avoiding beverages and particular foods or taking OTC antacids are not noted to be beneficial

The nurse is teaching a patient with acute lymphocytic leukemia (ALL) about therapy. What statements should be included in the plan of care? a) "The goal of therapy is palliation." b) "Side effects are rare with therapy." c) "Intrathecal chemotherapy is used primarily as preventive therapy." d) "Treatment is simple and consists of single drug therapy."

"Intrathecal chemotherapy is used primarily as preventive therapy." Correct Explanation: Intrathecal chemotherapy is a key part of the treatment plan to prevent invasion of the central nervous system. The therapy uses multiple drugs, with many side effects. The goal of therapy is remission

A nurse is caring for a 14-year-old adolescent who states, "No one understands me." Which of the following statements by the nurse best demonstrates empathy? a) "I'm sure your parents understand you." b) "Don't be so negative; things will get better." c) "Explain why you think no one understands you." d) "It is difficult being a teenager. Tell me more."

"It is difficult being a teenager. Tell me more." Explanation: Empathy is the ability to put oneself in another's place and experience a feeling as that person is experiencing it. The correct answer acknowledges the child's feelings and conveys an understanding without intimidating the child.

A group of nursing students are reviewing current nursing Codes of Ethics. Such a code is important in the nursing profession because: a) Nurses are responsible for carrying out actions that have been ordered by other individuals. b) Nurses are highly vulnerable to criminal and civil prosecution in the course of their work. c) Nurses interact with clients and families from diverse cultural and religious backgrounds. d) Nursing practice involves numerous interactions between laws and individual values.

Nursing practice involves numerous interactions between laws and individual values. Correct Explanation: A code of ethics is necessary to guide nurses' conduct, especially with regard to the interaction between laws and individual values. Diversity and legal liability do not provide the main justification for a code of ethics, though each is often a relevant consideration. The fact that nurses often carry out the orders of others is not the justification for a code of ethics

Completing your preoperative assessment, you mentally rehearse your client's needs to determine if there is increased risk for complications intra operatively or postoperatively. Which of the following are general risk factors? Select all that apply. a) Ethnicity b) Age c) Nutritional status d) Health status e) Gender f) Physical condition

Nutritional status • Age • Physical condition • Health status Explanation: General surgical risk factors are related to age; nutritional status; use of alcohol, tobacco, and other substances; and physical condition.

A patient with a history of alcoholism and scheduled for an urgent surgery asks the nurse, "Why is everyone so concerned about how much I drink?" What is the best response by the nurse? a) "It is important for us to know how much and how often you drink to help prevent surgical complications." b) "It is a required screening question for all patients having surgery." c) "We can have counselors available after surgery; if it is determined you need help for your drinking." d) "The amount of alcohol you drink will determine the amount of pain medication you will need postoperatively."

"It is important for us to know how much and how often you drink to help prevent surgical complications." Correct Explanation: Alcohol use and alcoholism can contribute to serious postoperative complications. If the medical and nursing staff is aware of the use or abuse, measures can be implemented proactively to prevent complications. Although alcohol may interfere with a medication's effectiveness, it does not determine the amount of pain medications that are prescribed following surgery. Even though this is a required screening question and counselors can be made available for those who want help, those are not the best responses to answer the patient's question.

The registered nurse is teaching a community health class about illness prevention. Which of the following statements reflects understanding of this concept? a) "My family will call hospice for an evaluation." b) "If I am injured, I will call an ambulance." c) "It is important to enroll in a smoking cessation class." d) "I will take my medication for hypertension every day."

"It is important to enroll in a smoking cessation class." Correct Explanation: Enrolling in a smoking cessation class is an example of illness prevention. It will prevent conditions such as asthma and COPD. A hospice evaluation is for someone who is terminally ill, hypertension is already a disease entity, and an ambulance for injury does not denote illness prevention.

A nurse is counseling a client about risk factors for hypertension. While reviewing the client's history, which information is consistent with the diagnosis of primary hypertension? Select all that apply. a) Hormonal contraceptives b) Obesity c) Stress d) High intake of sodium or saturated fat e) Diabetes mellitus f) Head injury

Obesity • Stress • High intake of sodium or saturated fat Explanation: Primary or essential hypertension has no identifiable cause and tends to develop gradually over years. Obesity, stress, high intake of sodium or saturated fat, and family history are all risk factors for primary hypertension. Diabetes mellitus, head injury, and hormonal contraceptive use are risk factors for secondary hypertension

A nurse is preparing a delusional client for a computed tomography scan of the brain to rule out an organic etiology. As the nurse accompanies the client to the radiology department, he looks around anxiously and states, "The Interpol is coming to kill me." What is the nurse's best response? a) "The Interpol isn't here." b) "No one can hurt you here." c) "It sounds like you're frightened." d) "Your illness is causing you to hear voices."

"It sounds like you're frightened." Explanation: Even though the client's thinking processes are distorted and irrational, his feelings are very real. The nurse should intervene by empathizing with his emotions. Assuring the client that the Interpol isn't present, telling the client that his illness is causing him to hear voices, and telling him that no one can hurt him appeal to the logical reasoning his illness has impaired. These responses may increase his anxiety by denying the reality of his current emotional experience.

A nurse is conducting an assessment of a client. Which client statement would indicate to the nurse that the client has a nonmodifiable risk factor for mental health problems? a) "My father was diagnosed with depression in his 40s." b) "My job has been getting more and more stressful." c) "I drink about a case or two of beer each week." d) "I'm probably going to get evicted from my home."

"My father was diagnosed with depression in his 40s." Correct Explanation: Nonmodifiable risk factors for mental health problems include age, gender, genetic background, and family history. The statement about the client's father being diagnosed with depression is an example. Problems with housing, economic difficulties, or alcohol abuse or use would be risk factors that could be modified.

The nurse needs to perform an admission assessment on a patient that does not speak the same language as the nurse. The patient's wife is fluent in both the language of the nurse and the patient. When completing the physical assessment is critical in planning patient care, how should the nurse proceed? a) Ask the patient's wife to assist with interpretation during the admission assessment b) Complete the admission assessment, provide patient privacy, and document the language barrier c) Obtain a translator to assist with interpretation during admission assessment d) Plan nursing care on the objective physical findings from the admission assessment

Obtain a translator to assist with interpretation during admission assessment Correct Explanation: Translation services should be provided for non-English-speaking patients. Asking the patient's wife violates the patient's confidentiality. Physical findings alone are not sufficient; the nurse must understand the patient's interpretation of the physical findings to provide culturally competent nursing care. Completion of the admission assessment in privacy and documenting the language barrier does not address the need for interpretation of the patient's history, perception, and description of assessment findings.

The nurse is working in a community mental health clinic. A client who is diagnosed with schizophrenia is taking clozapine and reports of a sore throat. What is the most appropriate action for the nurse to take? a) Encourage the use of saline mouth rinses until the sore throat is gone. b) Have the client decrease the daily amount of clozapine by half. c) Obtain an order for the client to have a white blood cell count drawn. d) Suggest that the client drink warm beverages and rest.

Obtain an order for the client to have a white blood cell count drawn. Explanation: The report of a sore throat may indicate an infection caused by agranulocytosis, a depletion in white blood cells. The way to determine this is by obtaining a white blood cell count. The other options do not get to the cause of the client's concern.

In assessing an adolescent client at an outpatient clinic, the nurse is able to recognize that depression in adolescents is often which of the following? a) Similar in symptomology to that of adult clients b) Situational and not as serious as that of adult clients c) Often masked by aggressive behaviors d) A sign that the teenager needs to be admitted to the hospital

Often masked by aggressive behaviors Correct Explanation: Depression in adolescents is often masked by anger or aggressive behaviors. Symptoms are usually different from adults in that adolescent exhibit intense mood swings.

A nurse is teaching family members about the brain's connection to behaviors commonly seen in mental illnesses during a family support network program. Which of the following statements indicates understanding of the concept of neurotransmitters if made by the family members? Select all that apply. a) "Neurotransmitters are hormones manufactured in glands such as the pituitary." b) "Neurotransmitters are chemicals stored in a neuron." c) "Neurotransmitters are a network of nerves that transmits impulses in the brain." d) "Amounts of neurotransmitters have an effect on human emotions and behavior." e) "Neurotransmitters are in the brain and peripheral nervous system."

"Neurotransmitters are chemicals stored in a neuron." • "Amounts of neurotransmitters have an effect on human emotions and behavior." • "Neurotransmitters are in the brain and peripheral nervous system." Explanation: Neurotransmitters are chemicals that are stored in the neuron and are released as neural messengers when stimulated by an electrical impulse. Neurotransmitters are involved with functions that affect human emotions and behavior. Neurotransmitters are the target for drugs used to treat mental illnesses. The neurotransmitters are stored in the axon terminal of the presynaptic neuron. A neurotransmitter is not a hormone stored in a gland or a location in the brain, nor is a neurotransmitter a nerve. Rather, it is a chemical that carries messages found within the brain and the peripheral nervous

A nurse should intervene when a depressed client makes which statement? a) "Television does not interest me anymore." b) "I have trouble falling asleep." c) "I have gained a little weight." d) "Nobody cares about me."

"Nobody cares about me." Correct Explanation: The client's statement "Nobody cares about me" may indicate the presence of suicidal ideation. Major themes associated with suicide are loneliness, abandonment, psychic pain, loss, helplessness, and hopelessness. The nurse should ask the client directly about suicidal thoughts and plans. Sleep disturbances are a sign that the depression still exists, but they do not indicate suicidal ideation. Thus, the statement regarding difficulty falling asleep does not require immediate intervention by the nurse. Change in appetite and decreased interest in usual activities are common symptoms of depressed mood.

A nurse observes a male client who is hyperactive and intrusive sitting very close to a female client with his arm around her shoulders. The nurse hears the male client tell a sexually explicit joke. The nurse approaches the client and asks him to walk down the hallway. Which statement by the nurse should benefit the client? a) "You need to be careful about what you say to other people." b) "She will not want to be around you with that kind of talk." c) "Telling sexual jokes and touching others is not permitted here." d) "I think a time-out in your room would be appropriate now."

"Telling sexual jokes and touching others is not permitted here." Correct Explanation: The nurse clearly informs the client about behavior that is unacceptable on the unit, such as voicing jokes with sexual content and touching others. Setting limits on behavior provides safety and security to the client and conveys to the client that he is worthy of help. Saying, "She will not want to be around you with that kind of talk" and "You need to be careful about what you say to others" does not clearly inform the client about behaviors that are unacceptable and implies that the client can control behaviors if he chooses. A time-out in the client's room does not inform the client about the inappropriateness of his behaviors and could be interpreted by the client as punitive as well as diminishing his self-esteem

A 70-year-old client asks the nurse if she needs to have a mammogram. Which is the nurse's best response? a) "Having a mammogram when you are older is less painful." b) "The incidence of breast cancer increases with age." c) "It will be sufficient if you perform breast examinations monthly." d) "We need to consider your family history of breast cancer first."

"The incidence of breast cancer increases with age." Correct Explanation: The nurse should explain that the incidence of breast cancer increases with age and current guidelines recommend women have a mammogram every 2 years until age 74. While mammograms are less painful as breast tissue becomes softer, the nurse should advise the woman to have the mammogram. Family history is important, but only about 5% of breast cancers are genetic. Several breast cancer screening guidelines recommend against breast self-examinations for women

The client with heart failure asks the nurse about the reason for taking enalapril maleate. The nurse should tell the client: a) "This drug will slow your heart rate down." b) "This drug will dilate your blood vessels and lower your blood pressure." c) "This drug will constrict your blood vessels and keep your blood pressure from getting too low." d) "This drug helps your heart beat more forcefully."

"This drug will dilate your blood vessels and lower your blood pressure." Correct Explanation: Enalapril maleate is an angiotensin-converting enzyme inhibitor that prevents conversion of angiotensin I to angiotensin II. Angiotensin II is a potent vasoconstrictor and also contributes to aldosterone secretion. Thus, enalapril decreases blood pressure through systemic vasodilation. Enalapril does not cause increased vasoconstriction, which would raise blood pressure. The medication has no effect on myocardial contractility or the heart's conduction system.

Which of the following statements by the nurse is an example of deception? a) "I will administer your pain medication as soon as it is due." b) "I am going to teach you how to give yourself insulin." c) "It is important to get out of bed to prevent pneumonia." d) "This injection of Novocain will feel like a little pinch."

"This injection of Novocain will feel like a little pinch." Explanation: Telling a client that Novocain will feel like a little pinch, is not being truthful, thus a deception. The other choices are true statements and interventions, thus not deception.

A nursing student is studying depression in the elderly adult. Faculty members knows the student has mastered the information when she states which of the following? a) "Depression is usually not accompanied by changes in behavior." b) "Sadness is most often associated with suicidal intent." c) "Depression can resolve without treatment." d) "Treatment of depression includes counseling."

"Treatment of depression includes counseling." Explanation: Treatment of depression usually involves psychotherapy or counseling along with antidepressant medication. In an older adult, hopelessness rather than sadness is more often associated with suicidal intent. Depression usually does not resolve without treatment and is frequently underdiagnosed. There is usually a distinct change of behavior accompanied by other specific signs and symptoms of depression

A group has asked the nurse to discuss how lifestyle factors affect heart health. Which of the following statements by members of the group would indicate that the teaching was effective? Select all that apply. a) "Chewing tobacco rather than smoking it lessens the negative effect on the heart." b) "If I change my diet and lessen my intake of saturated fats and trans fatty acids, this may decrease my cholesterol levels." c) "Walking is excellent exercise to strengthen my heart." d) "Gradually increasing my exercise levels will help enhance circulation through the heart." e) "As a borderline diabetic, if I lose weight and lessen my intake of simple carbohydrates, this should benefit my heart."

"Walking is excellent exercise to strengthen my heart." • "Gradually increasing my exercise levels will help enhance circulation through the heart." • "If I change my diet and lessen my intake of saturated fats and trans fatty acids, this may decrease my cholesterol levels." • "As a borderline diabetic, if I lose weight and lessen my intake of simple carbohydrates, this should benefit my heart." Correct Explanation: Increasing exercise levels, diet changes, losing weight, and walking are all important elements of heart health. Chewing tobacco is still harmful to the body.

Two nursing students are role-playing a patient assessment situation. One of the students is acting as the nurse, and the other student is acting as the patient. The task is to focus on assessing the patient's lifestyle. Which question would be most appropriate for the student acting as the nurse to ask? a) "Where do you currently live?" b) "Where do your parents come from?" c) "Can you tell me about your childhood?" d) "What do you usually do for fun?"

"What do you usually do for fun?" Explanation: Assessing a patient's lifestyle involves questions related to behaviors such as sleep patterns, exercise, nutrition, and recreation, as well as personal habits such as smoking and the use of ilicit drugs, alcohol, and caffeine. The question about what the patient does for fun reflects activities. The question about where the patient lives provides information about the physical environment; the question about where the patient's parents came from provides information about culture, which is also part of the environment. The question about the patient's childhood provides information about the patient's past life events related to health.

A client with Alzheimer's disease says, "I'm so afraid. Where am I? Where is my family?" How should the nurse respond? a) "You are in the hospital and you're safe here. Your family will return at 10 o'clock, which is 1 hour from now." b) "The name of the hospital is on the sign over the door. Let's go read it again." c) "You know where you are. You were admitted here 2 weeks ago. Don't worry, your family will be back soon." d) "I just told you that you're in the hospital and your family will be here soon."

"You are in the hospital and you're safe here. Your family will return at 10 o'clock, which is 1 hour from now." Correct Explanation: Providing the specific information requested comforts and reassures the client, who's lost and confused, and promotes orientation. The nurse shouldn't assume that a client with Alzheimer's disease will remember being admitted to the hospital and should supply specific information about when the family will visit. The nurse shouldn't scold or infantilize the client or assume that the client will remember the name of the hospital after seeing the sign.

A client states the following to the nurse: "I am a failure, and I wish I had died." Which of the following statements by the nurse demonstrates a therapeutic response? a) "You are depressed right now, so feeling like a failure is a normal manifestation." b) "I think you have had many successes in your life and you should focus on them." c) "I am glad to hear you speak about your feelings and I am glad you did not die." d) "You feel like a failure; would you like to talk more about the way you feel?"

"You feel like a failure; would you like to talk more about the way you feel?" Correct Explanation: Acknowledging the client's feelings by repeating what the client states is therapeutic. It is also therapeutic for the nurse to offer to discuss the client's feelings further. The other options are incorrect because they dismiss the client's feelings.

A client who is obese and the nurse have established a goal for the client to achieve a weight loss of 1 pound each week. One month later, the nurse evaluates that the client has lost 2 pounds. The nurse first states a) "You are not achieving satisfactory weight loss." b) "You need to work harder to achieve your goal." c) "You have succeeded in making positive progress." d) "We will have to re-evaluate your goal."

"You have succeeded in making positive progress." Correct Explanation: In the evaluation stage of the nursing process, the nurse validates even small increments toward goal achievement, as reflected in statement b. This is important for enhancement of client self-esteem and reinforcing client behavior. Change is a slow process, and success may be defined as making some progress. The nurse and client will then need to re-evaluate the goal, as in statement d, and either continue with the current goal, change the goal, or discontinue the goal. Statements a and c are negative criticisms and would diminish client self-esteem

A client admits to having thoughts of suicide. He is lethargic, withdrawn, and irritable. In conversations with the nurse, he stresses his faults. When he starts to point out the things he cannot do, which response by the nurse is best? a) "Try to think more positively about yourself." b) "You were able to write a letter to your friend today." c) "You can do anything you put your mind to." d) "Let us talk about your plans for the weekend."

"You were able to write a letter to your friend today." Correct Explanation: Pointing out the client's progress by describing what he can now do is therapeutic. Telling the client that he can do anything he puts his mind to is not therapeutic because it g

A client has a dx of risk for suicide r/t a past suicide attempt. Which outcome would the nurse prioritize? 1 the client will remain free from injury throughout hospitalization 2 the client will set one realistic goal related to relationships by day 3 3 the client will verbalize one positive attribute about self by day 4 4 the client will be easily redirected when discussion about suicide occurs by day 5

1

Which nursing dx takes priority for a client immediately after ECT? 1 risk for injury 2 impaired social interaction 3 activity intolerance 4 chronic confusion

1

Which pt statement is evidence of the etiology of major depressive disorder from a genetic perspective 1 my maternal grandmother was diagnosed with bipolar affective disorder 2 my mood is a 7 out of 10, and i won't harm myself or others 3 im so angry that my father left when i was 6 4 i just can't do anything right now, I'm worthless

1

Which symptom is an example of physiological alterations exhibited by clients diagnosed with moderate depression? 1 decreased libido 2 difficulty concentrating 3 slumped posture 4 helplessness

1

a client admitted after experiencing suicde ideations, is prescribed citalopram (Celexa). Four days later, the client has pressured speech and is noted wearing heavy makeup. What may be the potential reason for the clients behavior? 1 the pt is in a manic episode cause by the celexa 2 the client is showing improvement and is close to discharge 3 the client is masking depression in an attempt to get out of the hospital 4 the client has cheeked medications and taken them all in an attempt to OD

1

The client with major depressive disorder is suicidal. The client was prescribed the tricyclic antidepressant imipramine (Tofranil) 3 weeks ago. Which priority interven- tion should the nurse implement? 1. Determine if the client has a plan to commit suicide. 2. Assess if the client is sleeping better at night. 3. Ask the family if the client still wants to kill himself or herself. 4. Observe the client for signs of wanting to commit suicide.

1 The nurse should ask if the client has a plan to commit suicide. As the client begins to recover from both psychologi- cal and physical depression, the client's energy level increases, making the client more prone to commit suicide during this time. It takes 2-6 weeks for thera- peutic effects of tricyclic antidepressants to be effective. As the depression gets better, the client will start sleeping better, which indicates the medication is effective, but this is not a priority intervention because the client is suicidal. The family is an excellent resource to determine how the client is tolerating the medication and if it is effective, but the nurse should ask the client directly, not the family members, if he or she has thoughts of suicide. If the client seriously wants to commit suicide, usually the client will not show objective signs of wanting to kill themselves. The nurse must directly ask the client if he or she has a plan to commit suicide.

A pt is admitted to an in patient unit with a diagnosis of major depressive disorder. which of the following data would the nurse expect to assess. SATA 1 lost of interest in almost all activities and anhedonia 2 a change of more that 5% of body weight in 1 month 3 fluctuation between increased energy and loss of energy 4 psychomotor retardation or agitation 5 insomnia or hypersomnia`

1, 2, 4, 5

Which of the following medications may be administered before electroconvulsant therapy? SATA 1 Glycopyrrolate (robinul) 2 thiopental sodium (pentothal) 3 succinylcholine chloride (anectine) 4 lorazapam (ativan) 5 divalproex sodium (depakote)

1,2,3

which of the following are examples of anticholinergic side effects from tricylic antidepressants SATA 1 urinary hesitancy 2 constipation 3 blurred vision 4 sedation 5 weight gain

1,2,3

The client admitted to the psychiatric unit for major depressive disorder with an attempted suicide is prescribed an antidepressant medication. Which interventions should the psychiatric nurse implement? Select all that apply. 1. Assess the client's apical pulse and blood pressure. 2. Check the client's serum antidepressant level. 3. Monitor the client's liver function status. 4. Provide for and ensure the client's safety. 5. Evaluate the effectiveness of the medication.

1,3,4 Antidepressant medications may cause orthostatic hypotension, and the nurse should question administering the medication if the blood pressure is less than 90/60. Antidepressant medications do not have a therapeutic blood level; the effectiveness and side effects of the medication are deter- mined by the client's behavior. Many antidepressants may cause hepato- toxicity; therefore, the nurse should monitor the client's liver function tests. The nurse should ensure the client's safety. Many antidepressants may cause orthostatic hypotension and increase the risk for dizziness, falls, and injuries. Antidepressant medications take at least 3 weeks to become effective; therefore, when the client is first admitted to the psychiatric department and prescribed an antidepressant, evaluating for the effec- tiveness of the medication is not an appro- priate intervention.

A group of students are reviewing information about genomics and how things are changing toward a genomic era of personalized medicine. The students demonstrate understanding of this information when they identify which of the following as a characteristic? a) Treatment of the symptoms of the presenting disease b) Evaluation of a single gene as responsible for a disease c) Optimization of risk reduction related to genetic predisposition d) Strategy of waiting for disease symptoms to appear

Optimization of risk reduction related to genetic predisposition Explanation: The genomic era of personalized medicine focuses on the interaction of genes with one another and the environment, identifying genetic predisposition and optimizing risk reduction to prevent disease. The medical era focuses on single genes, waiting for disease symptoms to appear and treating the symptoms of the presenting disease

The nurse is planning a presentation about ovarian cancer to a group of women. Which topic should receive priority attention in the lesson plan? a) Ovarian cancer should be considered in any woman older than 30 years of age. b) A rigid board-like abdomen is the most common sign. c) Methods for early detection have made a dramatic reduction in the mortality rate due to ovarian cancer. d) Ovarian cancer signs and symptoms are often vague until late in development.

Ovarian cancer signs and symptoms are often vague until late in development. Correct Explanation: Ovarian cancer is rarely diagnosed early. Methods for mass screening and early detection have not been successful. Signs and symptoms are often vague until late in development. Ovarian cancer should be considered in any woman older than 40 years of age who has vague abdominal and/or pelvic discomfort or enlargement, a sense of bloating, or flatulence. Enlargement of the abdomen due to the accumulation of fluid is the most common sign

A nurse is caring for a client who has a body mass index (BMI) of 26.5. Which of the following categories should the nurse understand this client would be placed in? a) Healthy weight b) Overweight c) Underweight d) Obese

Overweight Explanation: A client with a BMI below 18.5 should be considered underweight. A client with a BMI of 18.5-24.9 is considered to be at a healthy weight. A client with a BMI of 25 to 29.9 is considered overweight; a client with a BMI of 30 or greater indicates obesity. A BMI greater than 40 is considered extreme obesity.

A nursing faculty is discussing laissez-faire values with students. Which of the following is an example of those values? a) Teaching children right from wrong b) Parents allowing a child to decide not to have an intravenous line inserted c) Telling a child an injection will feel like a pinch d) Modeling healthy behaviors for teenagers

Parents allowing a child to decide not to have an intravenous line inserted Correct Explanation: A laissez-faire environment is one that allows others, especially children, to make decisions without guidance, resulting in a decision that may not be a sound one. The other choices are not reflective of the definition of a laissez-faire environment.

Which of the following is a disorder due to a lesion in the basal ganglia? a) Guillain-Barré b) Parkinson's disease c) Multiple sclerosis d) Myasthenia gravis

Parkinson's disease Explanation: Disorders due to lesions of the basal ganglia include Parkinson's disease, Huntington's disease, and spasmodic torticollis.

The nurse is caring for a client with acute respiratory distress syndrome. What portion of arterial blood gas results does the nurse find most concerning, requiring intervention? a) Partial pressure of arterial carbon dioxide (PaCO2) of 51 mm Hg b) pH of 7.29 c) Bicarbonate (HCO3-)of 28 mEq/L d) Partial pressure of arterial oxygen (PaO2) of 69 mm Hg

Partial pressure of arterial oxygen (PaO2) of 69 mm Hg Explanation: In acute respiratory failure, administering supplemental oxygen elevates the PaO2. In acute respiratory distress syndrome, elevation of the PaO2 requires positive end-expiratory pressure. In both situations, the PaCO2 is elevated and the pH and HCO3- are depressed.

An obese adolescent tells the nurse that he would like to lose weight and asks the nurse's opinion on how to accomplish his goal. Which suggestion would be most appropriate? a) Cut down on sweets and other snacks. b) Participate in an adolescent weight-reduction program. c) Exercise more often. d) Strictly limit calorie intake.

Participate in an adolescent weight-reduction program. Explanation: Weight loss treatment modalities that include peer involvement have been proven to be the most successful approach with obese adolescents. This is because peer support is critical to adolescents, especially with an all-encompassing problem such as obesity. Increasing the amount of exercise is helpful, but this is just one aspect of a weight-reduction program. Strict calorie restriction is not recommended because it can result in use of muscle protein as well as fat for energy. Although decreased ingestion of nonnutritive snacks is helpful in dietary control, weight loss needs to be about long term behavior changes that also include physical activity.

A client is prescribed venlafaxine (Effexor) 37.5 mg bid, complains of dry mouth, orthostatic hypotension, and blurred vision. Which nursing intervention is appropriate? 1 hold the next dose, and document symptoms immediately 2 reassure the client that side effects are transient, and teach ways to deal with them 3 call the physician to receive an order for benztropine (Cogentin) 4 notify the dietary department about restrictions related to MAOIs

2

A clients outcome states " the client will make a plan to take control of one life situation by discharge" Which nursing dx documents the clients problem that this outcome addresses? 1 impaired social interaction 2 powerlessness 3 knowledge deficit 4 dysfunctional grieving

2

A instuctor is presenting statistics regarding suicide. Which statement indicates learning has occurred? 1 approximately 10,000 individuals will commit suicide each year 2 almost 95% of all individuals who commit or attempt suicide have a diagnosed mental disorder 3 suicide is the 8th leading cause of death among 15-24 year olds 4 depressive disorders account for 70% of all individuals who commit or attempt suicide

2

A pt denying suicide ideation comes to the ED complaining about insomnia, irritability, anorexia, and depressed mood. Which intervention should the nurse employ first? 1 request a psychiatric consultation 2 complete a thorough physical assessment including lab tests 3 remove all hazardous materials from the environment 4 place the client on a one to one observation

2

A pt with major depressive disorder is prescribed phenelzine (Nardil) Which teaching should the nurse prioritize? 1 remind the pt that the medication takes 6-8 weeks to take full effect 2 instruct the pt and family about the many food-drug and drug-drug interactions 3 teach the pt about the possible sexual side effects and insomnia that can occur 4 educate the pt about taking the medication prescribed even after symptoms improve

2

Which client would the charge nurse assign to an agency nurse working on the in patient unit for the first time? 1 a client experiencing passive suicide ideations with a pasts history of attempt 2 a client rating mood 3/10 and attending but not participating in group therapy 3 a client lying in bed all day long in a fetal position and refusing all meals 4 a client admitted for the first time with major depressive disorder

2

Which situation would place a client at high risk for a life threatening hypertensive crisis? 1 a client is prescribed tranylcypromine (Parnate) and eats chicken salad 2 a client is prescribed isocarboxazid (Marplan) and drinks hot chocolate 3 a client is prescribed venlafazine (effexor) and drinks wine 4 a client is prescribed phenelzine (Nardil) and eats fresh roasted chicken

2

major depressive disorder would be most difficult to detect in which of the following clients? 1 5 yo girl 2 13 yo boy 3 25 yo woman 4 75 yo man

2

The client with a major depressive disorder taking the selective serotonin reuptake inhibitor (SSRI) fluoxetine (Prozac) calls the psychiatric clinic and reports feeling confused and restless and having an elevated temperature. Which action should the psychiatric nurse take? 1. Determine if the client has flulike symptoms. 2. Instruct the client to stop taking the SSRI. 3. Recommend the client take the medication at night. 4. Explain that these are expected side effects.

2 Confusion and restlessness would not indi- cate the flu. The elevated temperature should make the nurse suspect a possible serious complication of SSRIs. Serotonin syndrome is a serious compli- cation of SSRIs that produces mental changes (confusion, anxiety, and restless- ness), hypertension, tremors, sweating, hyperpyrexia (elevated temperature), and ataxia. Conservative treatment includes stopping the SSRI and supportive treat- ment. If untreated, ESE can lead to death. Taking the medication at night will not treat serotonin syndrome. These are not expected side effects. They require nursing intervention.

The client with major depressive disorder is prescribed nefazodone (Serzone), an atypical antidepressant. The client tells the nurse, "I am going to take my medication at night instead of in the morning." Which statement would be the nurse's best response? 1. "You really should take the medication in the morning for the best results." 2. "It is all right to take the medication at night. It may help you sleep at night." 3. "The medication should be taken with food so you should not take it at night." 4. "Have you discussed taking the medication at night with your psychiatrist?"

2 This medication does not need to be taken in the morning to be more effective. Antidepressants may cause central nerv- ous depression, which causes drowsi- ness. Therefore, taking the medication at night may help the client sleep at night and relieve daytime sedation. This is the nurse's best response. Antidepressants do not need to be taken with food because they do not cause gastrointestinal distress. The nurse can provide factual information to the client without contacting the HCP. Taking antidepressants at night is not contraindicated; therefore, the nurse can share this information with the client.

A pt with major depressive disorder is newly prescribed sertaline (Zoloft). Which of the following teaching points would the nurse review? SATA 1 ask the pt about suicidal ideations related to depressed mood 2 discuss the need to take meds, even when symptoms improve 3 instruct the pt about the risks of abruptly stopping the medication 4 alert the client to the risks of dry mouth, sedation, nausea, and sexual side effects 5 remind the pt that the medications full effect may not occur for 6-8 weeks

2, 3, 4, 5

A nurse is caring for a client with a terminal illness. The client asks the nurse to help him end his own life to alleviate his suffering and that of his family. When responding to the client, the nurse integrates knowledge of which of the following? a) Nurses may administer medications prescribed by physicians to hasten end of life. b) Participating in assisted suicide violates the Code of Ethics for Nurses. c) Most states have enacted laws that allow for physician-assisted suicide. d) A client has the right to make independent decisions about the timing of his or her death.

Participating in assisted suicide violates the Code of Ethics for Nurses. Correct Explanation: The American Nurses Association Position Statement on Assisted Suicide acknowledges the complexity of the assisted suicide debate but clearly states that nursing participation in assisted suicide is a violation of the Code for Nurses. Legally, nurses are not allowed to administer medications even if prescribed by a physician if that medication will hasten the client's end of life. Proponents of physician-assisted suicide argue that terminally ill people should have a legally sanctioned right to make independent decisions about the value of their lives and the timing and circumstances of their deaths. However, this is not the case at the present time. Two states have enacted legislation for physician-assisted suicide. These laws provide access to physician-assisted suicide by terminally ill clients under very controlled circumstances.

A nurse obtains an order for a bed alarm for a confused client. This is an example of which of the following ethical principles? a) Conflict b) Paternalism c) Confidentiality d) Deception

Paternalism Correct Explanation: Making a decision for a client who is confused to prevent an injury is an example of paternalism. Deception and conflict would not promote the safety of a client who is unable to make safe decisions and confidentiality does not apply in this instance

The physician has recommended an amniocentesis for an 18-year-old woman. The patient is 34 weeks' gestation and does not want this procedure. The physician is insistent the patient has the procedure. The physician arranges for the amniocentesis to be done. What is this would be an example of? a) Veracity b) Paternalism c) Beneficence d) Autonomy

Paternalism Correct Explanation: Paternalism is the act of acting for patients without their consent to secure good or prevent harm.

Upon entering the hospital system, the nurse discusses the patient's rights and responsibilities that he is entitled to in the institution. The information the nurse discusses is commonly referred to as: a) The Bill of Rights for Registered Nurses. b) Standards of Clinical Nursing Practice. c) Patient's Bill of Rights. d) The Code of Ethics for Nurses.

Patient's Bill of Rights. Explanation: The American Hospital Association developed A Patient's Bill Of Rights (revised in 2003 as The Patient Care Partnership). The Bill of Rights includes the rights and responsibilities of the patient while receiving care in the hospital, and range from "the right to considerate and respectful care" to "the right to be informed of hospital policies and practices that relate to patient care, treatment and responsibilities.

A nurse believes that abortion is an acceptable option if a pregnancy results from a situation of rape. What is the best description of this belief? a) Ethics b) Legal obligations c) Personal morality d) Professional values

Personal morality Correct Explanation: Personal morality is the set of beliefs about the standards of right and wrong that helps a person determine the correct or permissible action in a given situation.

A nurse is of the Catholic faith and votes pro-life. He is considered to have a) Personal values b) Personal morality c) Legal obligations d) Ethics

Personal values Correct Explanation: Personal values are ideas or beliefs a person considers important and feels strongly about.

A nurse researcher is studying female patients who have survived breast cancer. The nurse asks each patient to describe her experience and then analyzes the data for the meaning of the experience within each person's own reality. This nurse has used what type of qualitative research method? a) Historical b) Phenomenology c) Grounded theory d) Ethnography

Phenomenology Explanation: The purpose of phenomenology is to describe experiences as they are lived by the subjects being studied. Studying a group of female patients who have survived breast cancer is an example of phenomenology. The basis of grounded theory is the discovery of how people describe their own reality and how their beliefs are related to their actions in a social scene. Ethnography research is based on examining issues of a culture that are of interest to nursing. Historical research examines events of the past to increase understanding of the nursing profession today

When caring for a client with fecal incontinence, the nurse knows that fecal incontinence is the result of which of the following reasons? a) Nature and amount of food eaten by client b) Drinking and smoking habits of client c) Social and emotional setting of client d) Physiologic or lifestyle changes in client

Physiologic or lifestyle changes in client Correct Explanation: Fecal incontinence mainly results from physiologic or lifestyle changes that impair muscle activity and sensation of the gastrointestinal tract. Particularly in the older adult, the weakening of the intestinal walls and decreased muscle tone can lead to bowel incontinence.

Which of the following glands is considered the master gland? a) Thyroid b) Adrenal c) Pituitary d) Parathyroid

Pituitary Correct Explanation: Commonly referred to as the master gland, the pituitary gland secretes hormones that control the secretion of additional hormones by other endocrine glands. The thyroid, parathyroid, and adrenal glands are not considered the master gland.

Vision and visual fields are altered in disorders of which of the following endocrine glands? a) Thyroid b) Pituitary c) Parathyroid d) Pancreas

Pituitary Explanation: The pituitary gland is located close to the optic nerves and hence causes pressure on these nerves; thus, changes in the vision and the visual fields may occur.

A 36-year-old mother of six has been recently diagnosed with type 2 diabetes. She reports increased hunger and food consumption while continuing to lose weight. What is the term used to describe this condition? a) Polyuria b) Anorexia c) Polydipsia d) Polyphagia

Polyphagia Correct Explanation: While the needed glucose is being wasted, the body's requirement for fuel continues. The person with diabetes feels hungry and eats more (polyphagia). Despite eating more, he or she loses weight as the body uses fat and protein to substitute for glucose. Polyphagia describes a medical symptom that means excessive hunger or appetite

Which of the following would a nurse least likely assess in a client experiencing anxiety? a) Muscle tension b) Sleeping difficulties c) Positive self-talk d) Irritability

Positive self-talk Correct Explanation: Anxiety would be manifested by negative self-talk, sleeping difficulties, irritability, and muscle tension.

A pt plans and follows through with the wake and burial of a child lost in an auto accident. Using Engels model of normal grief response, which stage would the pt fall 1 resolution of the loss 2 recovery 3 restitution 4 developing awareness

3

A pt with major depressive disorder has an outcome that states "The pt will verbalize a measure of hope about the future by day 3." Which client statement indicates this outcome was successful? 1 i don't want to die because it would hurt my family 2 i need to go to group and get out of this room 3 i think I'm going to talk to my boss about conflicts at work 4 thank you for your compassionate care

3

Which is the key to understanding whether a child or adolescent is experiencing an underlying depressive disorder? 1 irritability with authority 2 being uninterested in school 3 a change in behaviors over a 2 week period 4 feeling insecure at a social gathering

3

a client admitted with major depressive disorder has a nursing dx of ineffective sleep pattern r/t aches and pains. Which is an appropriate short term outcome? 1 the client will express feeling rested upon awakening 2 the client will rate pain level at or below 4/10 3 the client will sleep 6-8 hrs a night by day 5 4 the client will maintain a steady sleep pattern while hospitilized

3

The nurse is assessing a patient who is bedridden. For which condition would the nurse consider this patient to be at risk? a) Increase in circulating fibrinolysin b) Increase in the movement of secretions in the respiratory tract c) Predisposition to renal calculi d) Increased metabolic rate

Predisposition to renal calculi Correct Explanation: In a bedridden client, the kidneys and ureters are level, and urine remains in the renal pelvis for a longer period of time before gravity causes it to move into the ureters and bladder. Urinary stasis favors the growth of bacteria that, when present in sufficient quantities, may cause urinary tract infections. Poor perineal hygiene, incontinence, decreased fluid intake, or an indwelling urinary catheter can increase the risk for urinary tract infection in an immobile patient. Immobility also predisposes the patient to renal calculi, or kidney stones, which are a consequence of high levels of urinary calcium; urinary retention and incontinence resulting from decreased bladder muscle tone; the formation of alkaline urine, which facilitates growth of urinary bacteria; and decreased urine volume. The client would be at risk for decreased movement of secretion in the respiratory tract, due to lack of lung expansion. The client would suffer from decreased metabolic rate due to being bedridden. The client would not have an increase in circulating fibrinolysin

The client diagnosed with a major depressive disorder asks the nurse, "Why did my psychiatrist prescribe an SSRI medication rather than one of the other types of anti- depressants?" Which statement by the nurse would be most appropriate? 1. "Probably it is the medication that your insurance will pay for." 2. "You should ask your psychiatrist why the SSRI was ordered." 3. "SSRIs have fewer side effects than the other classifications." 4. "The SSRI medications work faster than the other medications."

3 Confusion and restlessness would not indi- cate the flu. The elevated temperature should make the nurse suspect a possible serious complication of SSRIs. Serotonin syndrome is a serious compli- cation of SSRIs that produces mental changes (confusion, anxiety, and restless- ness), hypertension, tremors, sweating, hyperpyrexia (elevated temperature), and ataxia. Conservative treatment includes stopping the SSRI and supportive treat- ment. If untreated, ESE can lead to death. Taking the medication at night will not treat serotonin syndrome. These are not expected side effects. They require nursing intervention. The cost of the medication or the type of insurance should not be a reason why one medication is prescribed over another. This is passing the buck, and the psychiatric nurse should be knowledgeable about medications. SSRIs have the same efficacy as MAO inhibitors and tricyclics, but SSRIs are safer because they do not have the sympathomimetic effects (tachycardia and hypertension) and anticholinergic effects (dry mouth, blurred vision, urinary retention, and constipation) of the MAO inhibitors and tricyclics. All antidepressant medications take at least 14 to 21 days to become effective.

The nurse's comprehensive assessment of a patient includes inspection for signs of oral cancer. What assessment finding is most characteristic of oral cancer in its early stages? a) Presence of a painless sore with raised edges b) Areas of tenderness that make chewing difficult c) Diffuse inflammation of the buccal mucosa d) Dull pain radiating to the ears and teeth

Presence of a painless sore with raised edges Correct Explanation: Malignant lesions of the oral cavity are most often painless lumps or sores with raised borders. Because they do not bother the patient, delay in seeking treatment occurs frequently, and negatively affects prognosis. Dull pain radiating to the ears and teeth is characteristic of malocclusion. Inflammation of the buccal mucosa causes discomfort and often occurs as a side effect of chemotherapy. Tenderness resulting in pain on chewing may be associated with gingivitis, abscess, irritation from dentures, and other causes. Pain related to oral cancer is a late symptom.

A nursing instructor is teaching a class for a group of nursing students about the ethical principles. The instructor determines that the teaching was successful when the students give which of the following as an example of nonmaleficence? a) Providing emotional support to clients who are anxious b) Protecting clients from a chemically impaired practitioner c) Administering pain medications to a client in pain d) Performing dressing changes to promote wound healing

Protecting clients from a chemically impaired practitioner Correct Explanation: Protecting clients from a chemically impaired practitioner is an appropriate example of nonmaleficence. Nonmaleficence means to avoid doing harm, to remove from harm, and to prevent harm. Performing dressing changes to promote wound healing, providing emotional support to clients who are anxious, and administering pain medications to a client in pain are examples of beneficence, which means doing or promoting good

A patient in a clinical research study has given informed consent. This means that the patient has certain rights. These rights include which of the following? Select all that apply. a) Free medical care b) Protection from harm c) Refusal to participate d) Confidentiality e) Guarantee of treatment

Protection from harm • Confidentiality • Refusal to participate Correct Explanation: Informed consent in the research process protects the patient's rights to consent knowledgeably, to participate in a study without coercion, to refuse to participate without jeopardizing the care received, to expect confidentiality, and to be protected from harm. It does not give the patient the right to free care

A nurse takes informed consent from a client scheduled for abdominal surgery. Which of the following is the most appropriate principle behind informed consent? a) Helps the client refuse treatment that he or she does not wish to undergo. b) Provides the client with in-depth knowledge about the treatment options available. c) Protects the client's right to self-determination in health care decision making. d) Helps the client to make a living will regarding future health care required.

Protects the client's right to self-determination in health care decision making. Explanation: Informed consent protects the client's right to self-determination in health care decision making. Informed consent helps the client to refuse a treatment that the client does not wish to undergo and helps the client to gain in-depth knowledge about the treatment options available, but the most important function is to encourage shared decision making. Informed consent does not help the client to make a living will.

A client experiencing suicide ideations with a plan to overdose on medications is admitted to an in patient psych unit. Mirtazapine (Remeron) is prescribed. Which nursing intervention takes priority? 1 remind the pt that medication effectiveness may take 2-3 weeks 2 teach the pt to take the medication with food to avoid nausea 3 check the pts BP every shift to monitor for hypertension 4 monitor closely for signs that the client might be "checking" medications

4

A newly admitted client has been diagnosed with major depressive disorder. Which dx takes priority 1 social isolation 2 self care deficit 3 anxiety 4 risk for self directed violence

4

A nurse is planning to teach about appropriate coping skills. The nurse would expect which client to be at the highest level of readiness to participate in this instruction? 1 a newly admitted client with an anxiety level of 8/10 and racing thoughts 2 a client admitted 6 days ago for a manic episode refusing to take medications 3 a newly admitted client experiencing suicidal ideations with plan to OD 4 a client admitted 6 days ago for suicide ideations following a depressive episode

4

A pt diagnosed with major depressive disorder has a nursing diagnosis of low self esteem r/t negative view of self. Which cognitive nursing intervention would be appropriate to deal with the clients problem? 1 promote attendence in group therapy to assist the client in socializing 2 teach assertiveness skills by role playing situations 3 encourage the client to journal to uncover underlying feelings 4 focus on strengths and accomplishments to minimize failures

4

A pt with major depressive disorder and experiencing suicide ideation is showing signs of anxiety. Alprazolam (Xanax) is prescribed. Which assessment should be prioritized? 1 monitor for s/s of physical and psychological withdrawal 2 teach the client about side effects of the medication, and how to handle these side effects 3 assess for nasea, and give the meds with food if it occurs 4 ask the client to rate his or her mood on a mood scale, and monitor for suicide ideation

4

A student is studying major depressive disorder. which statement indicates learning has occurred? 1 1 % of the population is affected by depression yearly 2 2-5% of women experience depression in their lifetimes 3 1-3% of men become clinically depressed 4 major depression is the leading cause of disibility in the US

4

Which charting entry most accurately documents a clients mood? 1 the client expresses an elevation in mood 2 the client appears euthymic and is interacting with others 3 the client isolates self and is tearful most of the day 4 the client rates mood at a 2 out of 10

4

a pt has a dx of dysfunctional grieving r/t loss of a job AEB inability to seek employment because of sad mood. Which would support a resolution of the pts problem? 1 the pt reports anxiety 2/10 and denies suicide ideations 2 the pt exhibits trusting behaviors toward the treatment team 3 the pt is noted to be in the denial stage of the grief process 4 the pt recognizes and accepts the role he or she played in the loss of a job

4

during an intake assessment, which client statement is evidence of the etiology of major depressive disorder from an object-loss theory perspective? 1 a am so angry all the time and seem to take it out on myself 2 my grandmother and great grandfather also had depression 3 i just don't think my life is ever going to get better, i keep messing up 4 i don't know about my biologic family, i was in foster care as an infant

4

An admitting nurse on a rehabilitation unit notices that an elderly client with a fractured hip and severe hypothyroidism is dirty and disheveled and that his personal hygiene is very poor. As the nurse gathers admission data, she further notes that the client has few personal connections, is depressed, and doesn't seem to care about his appearance. How should the nurse improve the client's performance of self-care activities? a) Provide initial and routine hygienic care, then evaluate the client daily as treatment progresses. b) Provide complete hygienic care and make an appointment for the client to see the hospital barber. c) Offer to take the client to the shower and help him fix his hair. d) Ask the physician to refer the client to social services for a full evaluation and follow-up.

Provide initial and routine hygienic care, then evaluate the client daily as treatment progresses. Explanation: Low thyroid levels can cause depression, which can explain many of this client's symptoms. Rather than assuming the client doesn't care about his appearance, the nurse should provide supportive hygienic care and observe for mood changes as the client's thyroid levels improve. Offering to escort the client to the shower and help with his hair, providing complete hygienic care and making an appointment with the hospital barber, and asking the physician to refer the client to social services are appropriate interventions to take if the client's behavior doesn't improve as thyroid level improves.

A patient with HIV is visiting the health care center for a regular checkup. His symptoms indicate multiorgan infections and he reports extreme weakness and says that he is depressed most of the time, as his friends and colleagues has distanced themselves from him. Which are the primary areas of concern for the nurse attending the patient? Select all that apply. a) Instruct the patient to take frequent rest periods. b) Provide patient education related to multiorgan infections. c) Diagnose opportunistic infections. d) Prescribe medications based on viral load. e) Refer patient to seek psychosocial counseling.

Provide patient education related to multiorgan infections. • Instruct the patient to take frequent rest periods. • Refer patient to seek psychosocial counseling. Correct Explanation: Independent HIV nursing management involves managing the patient's psychosocial and educational needs. Diagnosis of opportunistic infections and medication prescription based on viral load are areas of interdependent concerns in disease management

Dry, rough, scaly skin with the presence of itching is best described as: a) Pruritus b) Seborrhea c) Candidiasis d) Shingles

Pruritus Correct Explanation: Pruritus (itching) is one of the most common symptoms of patients with dermatologic disorders. Itch receptors are unmyelinated, penicillate (brush-like) nerve endings that are found exclusively in the skin, mucous membranes, and cornea

Which of the following symptoms characterizes Korsakoff syndrome? a) Psychosis, disorientation, delirium, insomnia, and hallucinations b) Tremor, rigidity, and bradykinesia c) Choreiform movement and dementia d) Severe dementia and myoclonus

Psychosis, disorientation, delirium, insomnia, and hallucinations Correct Explanation: Korsakoff syndrome is a personality disorder characterized by psychosis, disorientation, delirium, insomnia, and hallucinations. Creutzfeldt-Jacob disease results in severe dementia and myoclonus. The three cardinal signs of Parkinson's disease are tremor, rigidity, and bradykinesia. Huntington disease results in progressive involuntary choreiform (dancelike) movements and dementia

A nurse recognizes an error in documentation regarding the site of a wound. What actions by the nurse are appropriate? (Select all that apply.) a) Write the words "mistaken entry" above the incorrect entry. b) Put a single line through the incorrect entry. c) Have the charge nurse co-sign the incorrect entry. d) Use correction fluid to cover the incorrect entry. e) Make a different entry explaining the incorrect entry.

Put a single line through the incorrect entry. • Write the words "mistaken entry" above the incorrect entry. Correct Explanation: The nurse should put a single line through the incorrect entry, writing the words "mistaken entry" above or next to the entry and sign. Correction fluid should never be used in a client record. There is no need to explain the incorrect entry or to have the charge nurse co-sign the document.

The client diagnosed with depression is prescribed phenelzine (Nardil), a monoamine oxidase (MAO) inhibitor. Which statement by the client indicates to the nurse the medication teaching is effective? 1. "I am taking the herb ginseng to help my attention span." 2. "I drink extra fluids, especially coffee and iced tea." 3. "I am eating three well-balanced meals a day." 4. "At a family cookout I had chicken instead of a hotdog."

4 The client should use herbs cautiously because ginseng causes headaches, tremors, mania, insomnia, irritability, and visual hallucinations. The nurse should ask if the client has a plan to commit suicide. As the client begins to recover from both psychologi- cal and physical depression, the client's energy level increases, making the client more prone to commit suicide during this time. It takes 2-6 weeks for thera- peutic effects of tricyclic antidepressants to be effective. As the depression gets better, the client will start sleeping better, which indicates the medication is effective, but this is not a priority intervention because the client is suicidal. The family is an excellent resource to determine how the client is tolerating the medication and if it is effective, but the nurse should ask the client directly, not the family members, if he or she has thoughts of suicide. If the client seriously wants to commit suicide, usually the client will not show objective signs of wanting to kill themselves. The nurse must directly ask the client if he or she has a plan to commit suicide. 2. The client should refrain from drinking too many beverages containing caffeine. 3. Eating three balanced meals a day is not information that the nurse would teach about MAO inhibitors. 4. Taking MAO inhibitors requires adher- ence to strict dietary restrictions concerning tyramine-containing foods, such as processed meat (hot dogs, bologna, and salami), yeast products, beer, and red wines. Eating these foods can cause a life-threatening hypertensive crisis.

The client with major depressive disorder has been taking amitriptyline (Elavil), a tricyclic antidepressant, for more than 1 year. The client tells the psychiatric clinic nurse that the client wants to quit taking the antidepressant. Which intervention is most important for the nurse to discuss with the client? 1. Ask questions to determine if the client is still depressed. 2. Ask the client why he or she wants to stop taking the medication. 3. Tell the client to notify the HCP before stopping medication. 4. Explain the importance of tapering off the medication.

4 The nurse should discuss what behavior led to the client being prescribed antidepres- sants and determine if the client is still depressed, but the most important thing to discuss with the client is that the antidepres- sant medication should not be discontinued abruptly. The nurse should discuss why the client wants to stop taking the medication, but the most important intervention is to teach the client that the medication must be tapered. The client could quit taking medication without telling an HCP; therefore, teaching safety is priority. The client should notify the HCP before stopping the medication, but the most important intervention is to keep the client safe and inform the client to taper off the medication. The client must first know the impor- tance of needing to taper off the medica- tion because rebound dysphoria, irritability, or sleepiness may occur if the medication is discontinued abruptly. Then the client should see the HCP to determine what action should be taken because the client doesn't want to take the medication.

The client prescribed an antidepressant 1 week ago tells the psychiatric clinic nurse, "I really don't think this medication is helping me." Which statement by the psychi- atric nurse would be most appropriate? 1. "Why do you think the medication is not helping you?" 2. "You think your medication is not helping you." 3. "You need to come to the clinic so we can discuss this." 4. "It takes about 3 weeks for your medication to work."

4 The nurse should realize this medication takes at least 3 weeks to work; therefore, this question is not helpful to the client. This is a therapeutic response to encour- age the client to ventilate feelings, but the client needs factual information. The nurse should realize this medication takes at least 3 weeks to become effective, and the client does not need to come into the clinic to be told that fact. The client probably was told this infor- mation but may have forgotten it, or the client may not have been told, but the most appropriate response is to provide information so that the client realizes it takes 3 weeks for the medication to work and that he or she may not feel better until that time has elapsed.

A client is diagnosed with multiple site cancers and has received whole-body irradiation. The nurse is concerned about a compromised immune system in this client for which of the following reasons? a) Radiation causes an excess of circulating lymphocytes. b) Radiation causes a deficiency of circulating hemoglobin. c) Radiation causes an excess of circulating hemoglobin. d) Radiation destroys lymphocytes.

Radiation destroys lymphocytes. Correct Explanation: Radiation destroys lymphocytes and decreases the ability to mount an effective immune response. Radiation is not associated with an excess of lymphocytes or an excess or deficiency of hemoglobin

Which of the following liver function studies is used to show the size of the liver and hepatic blood flow and obstruction? a) Magnetic resonance imaging (MRI) b) Electroencephalogram (EEG) c) Angiography d) Radioisotope liver scan

Radioisotope liver scan Explanation: A radioisotope liver scan assesses liver size and hepatic blood flow and obstruction. An MRI is used to identify normal structures and abnormalities of the liver and biliary tree. Angiography is used to visualize hepatic circulation and detect the presence and nature of hepatic masses. An EEG is used to detect abnormalities that occur with hepatic coma.

The nurse has instructed the client in self-catheterization, but the client is unable to perform a return demonstration. What is the nurse's most appropriate plan of action? a) Revise the plan to include the inclusion of a support group. b) Teach the content again utilizing the same method. c) Report the client?s inability to learn to the case manager. d) Reassess the appropriateness of the method of instruction.

Reassess the appropriateness of the method of instruction. Correct Explanation: It is the nurse's responsibility to revise the plan of care if an intervention is not successful. The most appropriate action of the nurse would be to determine if the initial education was the most effective for this client. Simply teaching the content again without reassessing the client's needs would not necessarily be effective. A support group might be helpful, but not until the client's needs are evaluated. The case manager is not responsible for the client's learning

A nurse arriving for duty notes that a nursing assistant (or unregulated care provider [UCP]) has been assigned to a complex client with treatments involving sterile technique. What is the responsibility of the nurse regarding the assignment of the UCP? a) Reassign the UCP to a client requiring basic tasks that the UCP has mastered. b) Make sure the UCP has practiced sterile technique on at least one other occasion. c) Provide the UCP with a list of resources to guide the implementation of care. d) Supervise the UCP during the treatments involving sterile technique.

Reassign the UCP to a client requiring basic tasks that the UCP has mastered. Correct Explanation: The nurse is accountable for the delegation of tasks to UCPs. The nurse delegates tasks to UCPs consistent with their level of expertise and education, the job description, agency policy, legislation, and personal need. UCPs should not be assigned to clients who are complex or require skills that involve a higher level of knowledge. Based on the choices offered, if the nurse is confident that the UCP has the appropriate knowledge regarding basic tasks, the tasks can be delegated. The other options are incorrect, as they do not ensure that the UCP has the knowledge and skill to provide the care or carry out the task

Which nursing action would be therapeutic for the client being admitted to the unit with panic disorder? Select all that apply. a) Confront the client's dysfunctional coping behaviors. b) Touch the client to provide contact with reality. c) Reassure the client of safety. d) Respect the client's personal space. e) Support the client's attempts to discuss feelings.

Reassure the client of safety. • Respect the client's personal space. • Support the client's attempts to discuss feelings. Correct Explanation: Supporting the client's attempts to discuss feelings conveys empathy and is a therapeutic response. Respecting personal space demonstrates caring and helps to prevent escalation of anxiety. Reassuring the client about safety promotes a therapeutic nurse-client relationship and prevents escalation of anxiety. Touching the client or confronting dysfunctional coping behaviors or defense mechanisms will most likely be viewed as a threat and will increase anxiety.

A nursing instructor is discussing the diagnosis of intermittent claudication with students. To determine if the students understand the pathophysiology of the disease, the instructor asks, "What percentage of the arterial lumen must be obstructed before intermittent claudication is experienced?" a) 50 b) 40 c) 20 d) 30

50 Correct Explanation: Typically, about 50% of the arterial lumen or 75% of the cross-sectional area must be obstructed before intermittent claudication is experienced.

The nurse is assessing an adolescent with an annual physical. The mother reports that she has noticed a change in the child's behavior lately including mood swings, withdrawal from the family, and failing school grades. The mother does not know what to do and asks the nurse for guidance. What is the most appropriate guidance from the nurse? a) ?These could be signs of substance abuse. Open communication and a referral to a counselor that specializes in substance abuse would be beneficial.? b) ?Let?s admit your child to an acute care facility so that we can run more tests.? c) ?Adolescents are generally difficult children. Sometimes punishment is necessary to make them change their attitudes.? d) ?This is typical adolescent behavior. Ignore it and it will improve.?

?These could be signs of substance abuse. Open communication and a referral to a counselor that specializes in substance abuse would be beneficial.? Correct Explanation: Some signs of substance abuse in adolescents include mood swings, withdrawal from the family, and failing school grades. The other statements are inappropriate generalizations and do not address the problem. There is not enough evidence to suggest a need for hospital admission

A client with mania begins dancing around the day room. When she twirled her skirt in front of the male clients, it was obvious she had no underpants on. The nurse distracts her and takes her to her room to put on underpants. The nurse acted as she did to A minimize the client's embarrassment about her present behavior. B keep her from dancing with other clients. C avoid embarrassing the male clients who are watching. D teach her about proper attire and hygiene.

A

What are the most common types of side effects from SSRIs? A Dizziness, drowsiness, and dry mouth B Convulsions and respiratory difficulties C Diarrhea and weight gain D Jaundice and agranulocytosis

A

Which of the following clients would the nurse expect to have negative coping skills? a) A 13-year-old diabetic who joins a softball league b) A 37-year-old factory worker who is laid off for the summer c) A 72-year-old retiree who needs to take an expensive new chemotherapeutic agent d) A 19-year-old diagnosed with schizophrenia who is heading off to college

A 19-year-old diagnosed with schizophrenia who is heading off to college Correct Explanation: All of these clients will be facing stressors and need to develop adaptation skills. The client with schizophrenia, a chronic but manageable psychiatric illness, is most at risk for reacting negatively to the new life stressor. Clients with mental illness may have decreased physiologic reserve with which to effectively cope and adapt.

A registered nurse (RN) has been paired with a licensed practical nurse (LPN) for the shift. Whose care should the RN delegate to the LPN? a) A 19-month-old infant who had surgery for a fractured tibia 12 hours ago b) A 2-year-old child who nearly drowned 2 days earlier c) A 17-month-old infant who lost consciousness 2 hours earlier because of a head injury d) A 6-month-old infant who has gastroenteritis and vomits every 30 minutes

A 2-year-old child who nearly drowned 2 days earlier Correct Explanation: The nurse can delegate care of the near-drowning victim to an LPN. Children recover quite quickly from near-drowning experiences; acute care isn't necessary. The infant who has undergone surgery is still under the effects of anesthesia and requires close observation for dehydration, pain, and signs of adverse reactions. The infant with gastroenteritis also requires close monitoring for signs of dehydration. The infant who lost consciousness will need to be monitored most closely. His status could quickly become very critical.

A registered nurse (RN) has been paired with a licensed practical nurse (LPN) for the shift. Whose care should the RN delegate to the LPN? a) A 2-year-old child who nearly drowned 2 days earlier b) A 17-month-old infant who lost consciousness 2 hours earlier because of a head injury c) A 19-month-old infant who had surgery for a fractured tibia 12 hours ago d) A 6-month-old infant who has gastroenteritis and vomits every 30 minutes

A 2-year-old child who nearly drowned 2 days earlier Correct Explanation: The nurse can delegate care of the near-drowning victim to an LPN. Children recover quite quickly from near-drowning experiences; acute care isn't necessary. The infant who has undergone surgery is still under the effects of anesthesia and requires close observation for dehydration, pain, and signs of adverse reactions. The infant with gastroenteritis also requires close monitoring for signs of dehydration. The infant who lost consciousness will need to be monitored most closely. His status could quickly become very critical.

Which client is most likely to develop systemic lupus erythematosus (SLE)? a) A 27-year-old black female b) A 25-year-old white male c) A 25-year-old Jewish female d) A 35-year-old Asian male

A 27-year-old black female Explanation: SLE strikes nearly 10 times as many women as men and is most common in women between ages 15 and 40. SLE affects more black women than white women; its incidence is about 1 in every 250 black women, compared to 1 in every 700 white women

A nurse is counseling several clients for depression. Four of them do not seem to be improving, which leads the nurse to suggesting a referral to a psychiatric nurse practitioner. Which of these clients would be most likely to attend the scheduled appointment? a) A 36-year-old male; uses public transportation, unable to read, wants to confer with pastor. b) A 28-year-old female; works nights, willing to try, asking about insurance coverage of appointment. c) A 51-year-old male; walks to most places because of no transportation, low income, works days. d) A 45-year-old female; unsure of the benefit of psychiatric care, on a fixed income, good family support.

A 28-year-old female; works nights, willing to try, asking about insurance coverage of appointment. Correct Explanation: Cultural issues, poor family support, or lack of transportation may cause barriers to obtaining needed care. If a client must choose between work and keeping an appointment, scheduling is an important factor. The characteristics of a client who will keep appointments with specialists include those clients who have been educated well about the importance of the referral, understand the benefit to seeing a specialist, and those who will not have to struggle to keep the appointment. A client who agrees, who has input in scheduling, and shows curiosity or hope about the benefit of the referral is more likely to keep the appointment.

Which of the following clients requires increased sensory stimulation to prevent sensory deprivation? a) A 65-year-old client who has employment-induced presbycusis and advanced glaucoma b) A 60-year-old client who is blind, reads books through use of Braille, listens to the radio, and regularly takes walks around the unit c) An 84-year-old client who has hemiparesis and ambulates with a walker d) A 24-year-old client who has been admitted with an anxiety disorder and appears very agitated

A 65-year-old client who has employment-induced presbycusis and advanced glaucoma Explanation: There is more risk of sensory deprivation when the primary senses are impaired. This client is most at risk for sensory deprivation because of two sensory deficits: hearing and vision. These two are primary senses that help a person stay oriented and communicate with others. The 24-year-old client has senses intact but is experiencing an anxiety disorder. The 60-year-old client has one sensory deficit, blindness, but is compensating by reading books and listening to tapes. An elderly person with mobility problems, who is using mobility aids, and hence can still socialize is not experiencing sensory deficits

Nurses use social media to share ideas, develop professional connections, access educational offerings and forums, receive support, and investigate evidence-based practices. What is an example of the proper use of social media by a nurse? a) A nurse posts pictures of a patient who accomplished a goal of losing 100 pounds and later deletes the photo. b) A nurse uses a disclaimer to verify that any views expressed on Facebook are his or hers alone and not the employer's. c) A nurse describes a patient on Twitter by giving the room number rather than the name of the patient. d) A nurse describes a patient on Twitter by giving the patient's diagnosis rather than the patient's name.

A nurse uses a disclaimer to verify that any views expressed on Facebook are his or hers alone and not the employer's. Correct Explanation: A proper use of social media by a nurse would be the use of a disclaimer to verify that any views expressed on Facebook are the nurse as an individual and not the employer's. The nurse should not use social media in any way to describe a client by room number, medical diagnosis, or by accomplished medical goal of any type. Serious consequences can result from a nurse not using social media correctly.

Which of the following best describes the health-illness continuum? a) A person with chronic illness is at the far end of the continuum reflecting illness. b) A person may be considered neither completely healthy or completely ill. c) A person on the continuum remains at the point based on his or her initial state of health. d) A person with high-level wellness is free of any disease or infirmity.

A person may be considered neither completely healthy or completely ill. Explanation: The health-illness continuum views a person as being neither completely healthy nor completely ill. Use of the continuum makes it possible to view a person as simultaneously possessing degrees of both health and illness. People with chronic illness or disability may attain a high-level of wellness if they are successful in meeting their health potential withing the limits of their condition. A person's state of health is ever-changing and can fall anywhere along the continuum at any time. High-level wellness does not necessarily mean that a person is free of disease or infirmity

The nurse caring for a client with obesity would like to address the possible health problems that can develop related to obesity. To plan care for this client, what type of nursing diagnosis would the nurse formulate? a) A wellness diagnosis b) An actual nursing diagnosis c) A risk nursing diagnosis d) A possible nursing diagnosis

A risk nursing diagnosis Correct Explanation: Since the nurse is trying to address health problems that the client is at risk for because of obesity, the appropriate diagnosis is a risk nursing diagnosis. The nurse is not addressing a health problem that the client has or a health problem that the nurse needs more information to validate, so an actual or possible nursing diagnosis is not appropriate. The client is not seeking health information, so a wellness diagnosis in inappropriate.

Which of the following activities would be appropriate for a client with mania? A Drawing a picture B Modeling clay C Playing bingo D Playing table tennis E Stretching exercises F Stringing beads

A, B, E

Which of the following would indicate an increased suicidal risk? A An abrupt improvement in mood B Calling family members to make amends C Crying when discussing sadness D Feeling overwhelmed by simple daily tasks E Statements such as "I'm such a burden for everyone" F Statements such as "Everything will be better soon"

A, B, F

A patient who stopped smoking 3 years ago has not resumed smoking and actively encourages other family members to stop smoking. This patient is demonstrating which step in the process of valuing? a) Prizing b) Choosing c) Treasuring d) Acting

Acting Explanation: This patient is demonstrating the consistency and regularity on the value that is consistent with the acting phase of the process of valuing. Choosing involves freely selecting from alternatives after careful consideration of the consequences of each alternative. Prizing and treasuring involve pride, happiness, and public affirmation

A patient who stopped smoking 3 years ago has not resumed smoking and actively encourages other family members to stop smoking. This patient is demonstrating which step in the process of valuing? a) Acting b) Treasuring c) Choosing d) Prizing

Acting Explanation: This patient is demonstrating the consistency and regularity on the value that is consistent with the acting phase of the process of valuing. Choosing involves freely selecting from alternatives after careful consideration of the consequences of each alternative. Prizing and treasuring involve pride, happiness, and public affirmation.

A nurse is caring for a patient who is a practicing Jehovah's Witness. The physician orders 2 units of packed cells based on his low hemoglobin and hematocrit levels. The nurse states to the surgeon that it is unethical to go against the patient's beliefs even though his blood counts are very low. What is the best description of the nurse's intentions? a) Observing institutional policies b) Being legally responsible c) Acting in the patient's best interest d) Siding with the patient over the surgeon

Acting in the patient's best interest Correct Explanation: Nurses' ethical obligations include acting in the best interest of their patients not only as individual practitioners but also as members of the nursing profession, the healthcare team, and the community at large.

An emergency department nurse is awaiting the arrival of multiple persons exposed to botulism at the local shopping mall. What should the nurse do? a) Implement contact precautions with all exposed victims. b) Separate those exposed to botulism from those who were not exposed. c) Notify community agencies of the incident. d) Activate the facility's emergency disaster plan.

Activate the facility's emergency disaster plan. Correct Explanation: The nurse should activate the facility's emergency disaster plan to ensure availability of sufficient personnel and supplies. The Centers for Disease Control and Prevention and local health departments are the designated agencies responsible for collaboration with hospital agencies regarding community biochemical exposures. The Public Health Agency of Canada and the the Department of Public Safety and Emergency Preparedness are the designated agencies responsible for disaster coordination in Canada and should notify all community agencies of the incident. The nurse must implement standard precautions, not contact precautions, to prevent botulism transmission from one person to another. It is not necessary to quarantine exposed individuals.

A nurse is caring for a client who began taking the antidepressant paroxetine (Paxil) 2 weeks ago. The client recently began giving away prized possessions and tells the nurse, ?My mind is made up, I can do this.? What is the best action by the nurse to incorporate this information into the plan of care? a) Tell another nurse about this client statement. b) Add the nursing diagnosis: Risk for self-harm. c) Encourage the client to join a therapy group. d) Document the depression has resolved.

Add the nursing diagnosis: Risk for self-harm. Explanation: Ongoing planning helps to resolve health problems and promote function. The nurse uses new data to make the plan more specific and effective. For this client, assessment indicates risk for self-harm, not resolution of the depression. The nurse will perform additional assessment and add the new nursing diagnosis to the care plan. Changing the care plan to incorporate this new data makes it the most effective for treating the client. Telling another nurse could assist in treatment, but is less formal and effective

A patient is being treated in a substance abuse unit of a local hospital. The nurse understands that when a patient has compulsive behavior to use a drug for its psychic effect, the patient needs to be monitored for which of the following? a) Placebo effect b) Dependence c) Tolerance d) Addiction

Addiction Explanation: Addiction is a behavioral pattern of substance use characterized by a compulsion to take the substance primarily to experience its psychic effects. Placebo effect is analgesia that results from the expectation that a substance will work, not from the actual substance itself. Dependence occurs when a patient who has been taking opioids experiences a withdrawal syndrome when the opioids are discontinued. Tolerance occurs when a person who has been taking opioids becomes less sensitive to their analgesic properties

Which of the following nursing interventions is most likely to be allowed within the parameters of a protocol or standing order? a) Administering a beta-adrenergic blocker to a new patient whose blood pressure is high on admission assessment b) Changing a patient's advance directive after his prognosis has significantly worsened c) Changing a patient's intravenous (IV) fluid from normal saline to 5% dextrose d) Administering a glycerin suppository to a constipated patient who has not responded to oral stool softeners

Administering a glycerin suppository to a constipated patient who has not responded to oral stool softeners Correct Explanation: Standing orders and protocols often surround the management of bowel elimination. Modification of a patient's IV fluid or administration of a new antihypertensive are patient-specific interventions that are physician initiated. The care team cannot independently change a patient's advance directive.

Patients who have received corticosteroids preoperatively are at risk for which type of insufficiency? a) Pituitary b) Parathyroid c) Adrenal d) Thyroid

Adrenal Correct Explanation: Patients who have received corticosteroids are at risk of adrenal insufficiency. Insufficiency related to corticosteroids does not occur to the pituitary, thyroid, or parathyroid glands.

Which of the following words is best described by the following: the protection and support of another's rights? a) Advocacy b) Ethics c) Paternalism d) Autonomy

Advocacy Correct Explanation: Advocacy is the protection and support of another's rights. Nurses who value patient advocacy make sure their loyalty to an employing institution or colleague does not compromise their primary commitment to the patient; give priority to the good of the individual patient rather than to the good of society in general; and carefully evaluate the competing claims of the patient's autonomy and patient well-being. Paternalism is acting for patients without their consent to secure good or prevent harm. Autonomy is respecting the rights of patients or their surrogates to make healthcare decisions; it is also known as self-determination. Ethics is the formal, systematic inquiry into principles of right and wrong conduct, of virtue and vice, and of good and evil as they relate to conduct and human flourishing.

A student nurse is assisting an elderly patient to ambulate following hip replacement surgery, and the patient falls and reinjures the hip. Who is potentially responsible for the injury to this patient? a) The hospital b) The student nurse c) The nurse instructor d) All of the above

All of the above Correct Explanation: As a student nurse, you are responsible for your own acts, including any negligence that may result in patient injury. A hospital may also be held liable for the negligence of a student nurse enrolled in a hospital-controlled program because the student is considered an employee of the hospital. The status of students enrolled in college and university programs is less clear, as is the liability of the educational institution in which they are enrolled and the health care agency offering a site for clinical practice. Nursing instructors may share responsibility for damages in the event of patient injury if an assignment called for clinical skills beyond a student's competency or the instructor failed to provide reasonable and prudent clinical supervision.

A nurse has completed 4 hours of his 8-hour shift on a medicalsurgical unit when he receives a phone call from the nursing supervisor. The nursing supervisor informs the nurse that he needs to give a report to the other two nurses on the medicalsurgical unit and immediately report to the telemetry unit to assist with staff needs on that unit. The nurse informs the supervisor that he has been busy with his patient assignment and feels this will overwhelm the nurses on the medicalsurgical unit. The supervisor informs the nurse that the need is greater on the telemetry unit. This is an example of which type of ethical problem? a) Deception b) Advocacy in a market-driven environment c) Conflicts concerning new technology d) Allocation of scarce nursing resources

Allocation of scarce nursing resources Correct Explanation: This is a clear example of ethical issues surrounding the allocation of scarce nursing resources and involves distributing nursing care.

A mother tells the nurse that her 4-year-old boy has developed some strange eating habits, including not finishing meals and eating the same food for several days in a row. She would like to develop a plan to correct this situation. When developing such a plan, what should the nurse and mother do? a) Decide on a good reward for finishing the meal. b) Restrict the availability of foods to those served at meal times. c) Allow him to make some decisions about the foods he eats. d) Do not allow him to leave the table until he has eaten the food.

Allow him to make some decisions about the foods he eats. Correct Explanation: Allowing a child to make some decisions about the foods he eats and not insisting that he finish meals can avoid power struggles. Refusing to finish meals and to eat certain foods is normal behavior for a preschool-aged child. It is important to avoid tension at mealtime and to avoid confrontation about food, which should not be used as a bribe or a reward. Rewarding a child for what is eaten can lead to power struggles between the parent and child over food. Restricting foods should be avoided; restriction can provoke power struggles and confrontation, thereby increasing tension. Not allowing the child to leave the table until finished can provoke power struggles and confrontation, thereby increasing tension.

Which type of cell secretes glucagon and promotes gluconeogenesis? a) Beta b) Delta c) Alpha d) Omega

Alpha Explanation: The alpha cells of the pancreas secret the hormone glucagon. It promotes gluconeogenesis, thus increasing the blood glucose level. The beta cells of the pancreas secrete insulin. Delta cells secrete somatostatin, which reduces the rate at which food is absorbed from the gastrointestinal tract

What is the definition of wellness? a) Maximizing the state in which you live. b) A desire to be without disease. c) An active state of being healthy. d) Being without disease

An active state of being healthy. Explanation: Wellness, a reflection of health, is an active state of being healthy by living a lifestyle that promotes good physical, mental, and emotional health.

A nurse caring for the skin of patients of different age groups should consider which accurately described condition? a) Secretions from skin glands are at their maximum from age 3 on. b) The skin becomes thicker and more leathery with aging and is prone to wrinkles and dryness. c) An infant's skin and mucous membranes are protected from infection by a natural immunity. d) An adolescent's skin ordinarily has enlarged sebaceous glands and increased glandular secretions.

An adolescent's skin ordinarily has enlarged sebaceous glands and increased glandular secretions. Correct Explanation: Adolescents have enlarges sebaceous glands and increased glandular secretions, which predisposes them to acne. Infants have natural immunities, but not pertaining to the mucous membranes. Secretions from skin glands occur later than age 3 months. While the skin may have more wrinkles as a person ages, the skin actually becomes thinner with age

Which of the following best describes feminist ethics? a) A combination of elements of utilitarian and deontologic theories that offers specific action guides for practice. b) Attention directed to the specific situation of individual patients viewed within the context of their life narratives. c) An approach critiquing existing patterns of oppression and domination in society. d) The formal study of ethical issues that arise in the practice of nursing.

An approach critiquing existing patterns of oppression and domination in society. Correct Explanation: Feminist ethics offers an approach critiquing existing patterns of oppression and domination in society, especially as they affect women and the poor. The principle-based approach to ethics combines elements of utilitarian and deontologic theories and offers specific action guides for practice. The care-based approach directs attention to the specific situation of individual patients viewed within the context of their life narratives. And the formal study of ethical issues that arise in the practice of nursing describes nursing ethics

A patient with HIV is visiting the health care center for a regular checkup. His symptoms indicate multiorgan infections and he reports extreme weakness and says that he is depressed most of the time, as his friends and colleagues has distanced themselves from him. Which are the primary areas of concern for the nurse attending the patient? Select all that apply. a) Refer patient to seek psychosocial counseling. b) Prescribe medications based on viral load. c) Diagnose opportunistic infections. d) Instruct the patient to take frequent rest periods. e) Provide patient education related to multiorgan infections.

Refer patient to seek psychosocial counseling. • Provide patient education related to multiorgan infections. • Instruct the patient to take frequent rest periods. Correct Explanation: Independent HIV nursing management involves managing the patient's psychosocial and educational needs. Diagnosis of opportunistic infections and medication prescription based on viral load are areas of interdependent concerns in disease management

A nurse has a duty of nonmaleficence. Which of the following would be considered a contradiction to that duty? a) Provide comfort measures for a terminally ill patient. b) Assist the patient with ADLs. c) Refuse to administer pain medication as ordered. d) Provide all information related to procedures.

Refuse to administer pain medication as ordered. Correct Explanation: The duty not to inflict as well as prevent and remove harm is termed nonmaleficence. Providing comfort measures for a terminally ill patient, assisting a patient with ADLs and providing information related to procedures would not be considered a contradiction to the nurse's duty of nonmaleficence

A nurse is assigned to be the circulating nurse during a surgical procedure. The nurse would be responsible for which activity? a) Preparing the sterile tables in the operating room before surgery b) Anticipating the needs of other members of the surgical team c) Coordinating care activity d) Providing sponges and drains to the surgical team in the operating room

Anticipating the needs of other members of the surgical team Explanation: The circulating nurse identifies and assesses the client on admission to the operating room, collaborates with multiple surgical team members to provide safe client care, including carefully positioning the client on the OR bed, using an approved antimicrobial agent to prepare the skin at the surgical site (prep), placing a Foley catheter (if indicated), assisting with monitoring the client during surgery, providing additional supplies, anticipating needs of the surgical team to facilitate the procedure, maintaining environmental safety, and counting the number of instruments, sharp items such as needles, and soft goods such as sponges used during the surgery to prevent the accidental loss of an item in the surgical site.

Which of the following classes of antineoplastic agents is cell cycle-specific? a) Antimetabolites (5-FU) b) Nitrosoureas (carmustine) c) Antitumor antibiotics (bleomycin) d) Alkylating agents (cisplatin)

Antimetabolites (5-FU) Correct Explanation: Antimetabolites are cell cycle-specific (S phase). Antitumor antibiotics are cell cycle-nonspecific. Alkylating agents are cell-cycle nonspecific. Nitrosoureas are cell cycle-nonspecific

A client continually reports of pain after the administration of an oral analgesic. The physician writes an order for the nurse to administer a placebo to the client the next time the client reports of pain. The doctor states, "Tell the client it is a stronger analgesic." What would be the appropriate action by the nurse? a) Refuse to administer the placebo to the client. b) Give the placebo but do not tell the client it is a stronger medication. c) Give the placebo as ordered by the physician. d) Consult with the pharmacist to discuss the dosage of the placebo.

Refuse to administer the placebo to the client. Correct Explanation: The nurse should refuse to give the placebo and should also refuse to misinform the client. The nurse has a responsibility to explain the client's medications to the client. The client can then make an informed decision about accepting or refusing the medication. The other options are incorrect because the nurse would be misinforming the client about the medication that is being administered. The client would not be able to provide informed consent

A nursing instructor is discussing a nursing student's Facebook post about a very interesting client situation that happened during clinical. The student states, "I didn't violate client privacy because I didn't use the client's name." What response by the nursing instructor is most appropriate? a) "Any information that can identify a person is considered a breach of client privacy." b) "You may continue to post about client you cared for during clinicals, as long as you do not use the client's name." c) "The information being posted on Facebook is inappropriate. Make sure to discuss information about client's privately with friends and family." d) "All aspects of the clinical experience are confidential and should not be discussed."

Any information that can identify a person is considered a breach of client privacy." Correct Explanation: Any information that can identify a person is considered confidential. A medical condition may identify a client that was cared for, especially if the location of the clinical site and unit was disclosed in the post. Discussion of clinical experience can be used for teaching purposes or seeking advice on care. No care should be discussed, even privately, with friends and family

A client has been diagnosed with colon cancer with metastasis to the lymph nodes. When the nurse enters the room, the client says life is "not worth living." What is the nurse's best therapeutic response? a) Approach the client and ask if there are questions about the condition. b) Explain that the condition is complicated and ask a physician to come speak with the client. c) Ask the client if calling the family would be helpful. d) Assure the client that everything will work out fine.

Approach the client and ask if there are questions about the condition. Correct Explanation: This is the best therapeutic response that is client focused. The other answers do not demonstrate therapeutic response: nurses should not offer false assurances, and calling the family is not addressing the problem between nurse and client.

A client has not had a bowel movement for 2 days and is feeling uncomfortable. The physician writes an order that states, "laxative of choice." How should the nurse proceed with this order? a) Ask the client what type of laxative he/she would like to have. b) Give mineral oil because it does not require a physician's order. c) Ask the physician to prescribe a specific laxative. d) Ask the client if he/she would prefer to have an enema administered.

Ask the physician to prescribe a specific laxative. Correct Explanation: The physician's order leaves the nurse in the position of prescribing a medication. To be a complete order, the physician must write the drug, dose, frequency, route, and purpose or reason for the drug. The other options are incorrect because they put the nurse in the position of prescribing a medication and not following established professional standards for the administration of medication

The nurse observes a physician leave the room of a patient in isolation for Clostridium difficile (C. difficile). The physician uses the alcohol-based hand sanitizer hanging on the wall to wash her hands and leaves the door open. Which of the following actions should the nurse take? a) Ask the physician to wash her hands with soap and water. b) Report the observation to the infection control department. c) No action is needed. The physician was following isolation protocol. d) Close the door to the room.

Ask the physician to wash her hands with soap and water. Correct Explanation: C. difficile is resistant to alcohol-based and other hand sanitizers; therefore physicians should be instructed to wash their hands with soap and water. The nurse could report the observation to the infection control department, but that does not address the immediate concern of the physician contaminating other patients. There is no need for the room door to be closed. The nurse must take action to ensure the safety of other patients

A client who fell at home is hospitalized for a hip fracture. The client is in Buck's traction, anticipating surgery, and reports pain as "2" on a pain intensity scale of 0 to 10. The client also exhibits moderate anxiety and moves restlessly in the bed. The best nursing intervention to address the client's anxiety is to a) Administer the prescribed alprazolam (Xanax). b) Assist the client out of bed and into a chair. c) Administer the prescribed dose of morphine. d) Assess the reason for the client's anxiety.

Assess the reason for the client's anxiety. Correct Explanation: Following the steps of the nursing process, the nurse needs to assess the reason for the client's anxiety. The client could be anxious about impending surgery, an unattended pet, a sick family member, etc. Then, the nurse intervenes appropriately by obtaining the assistance the client may need or administering anti-anxiety medication. The question is asking about treatment for anxiety. Pain medication should not be administered for anxiety. The nurse will not assist the client to a chair, because the client is on bedrest and in Buck's traction

The registered nurse (RN) is preparing to delegate tasks to an unlicensed assisted personnel (UAP). Which tasks would be appropriate for the RN to delegate to the UAP? a) Assessing oxygen (O2) saturation b) Calculating fluid volume resuscitation c) Monitoring intracranial pressure (ICP) d) Measuring intravenous therapy intake

Assessing oxygen (O2) saturation Explanation: When delegating the RN must determine the skill level, education of the UAP, client?s condition, complexity of the client, and potential for harm. Assessing O2 saturation is within the scope of an UAP. Monitoring ICP, measuring intravenous therapy intake, and calculating fluid volume resuscitation should be performed by a licensed professional.

A nurse volunteers to serve on the hospital ethics committee. Which of the following indicates that the nurse knows what the purpose of an ethics committee is? a) Assist in decision-making based on the client's best interests b) Present options about the type of care c) Decide the care for a client who is unable to voice their opinion d) Convince the family to choose a specific decision

Assist in decision-making based on the client's best interests Explanation: An ethics committee will meet when a client is unable to make an end-of-life decision and the family cannot come to a consensus. The committee members are there to advocate for the best interest of the client. The committee would not convince, decide, or present options about the type of care. This is not the role of an ethics committee

A nurse can best help a client who is undergoing chemotherapy and using guided imagery with this by doing which of the following? a) Promoting the client?s use of imagery only after a stressful event occurs. b) Explaining to the client that it is not a good idea to record their own imagery tape. c) Helping the client learn about the different poses that can be performed. d) Assisting the client to find an appropriate imagery tape to use.

Assisting the client to find an appropriate imagery tape to use. Correct Explanation: Clients use imagery to help with relaxation and this can use all five senses. Many times, clients use a guided imagery tape that includes a script to help with imagery; clients also can record their own tape. Clients do not necessarily use poses with this; this would be more related to use of yoga. Guided imagery can help both during and after a stressful event to help the client relax

A nursing instructor is describing values and how they can reflect a person's age and stage of development. At which stage would the instructor explain that values typically evolve into guiding principles in life? a) At infancy b) At old age c) At adulthood d) At childhood

At adulthood Correct Explanation: At adulthood, the values of a person develop into his or her guiding principles in life. This does not occur at infancy, childhood, or in old age. At infancy, a person learns to establish trust. At childhood, a person learns to distinguish between right and wrong. At old age, a person focuses on personal integrity and the wisdom that he or she has accumulated over a lifetime

A 46-year-old patient has been diagnosed with cancer. He has met with the oncologist and is now weighing his options to undergo chemotherapy or radiation as his treatment. This patient is utilizing which ethical principle in making his decision? a) Autonomy b) Beneficence c) Justice d) Confidentiality

Autonomy Correct Explanation: Autonomy entails the ability to make a choice free from external constraints. Beneficence is the duty to do good and the active promotion of benevolent acts. Confidentiality relates to the concept of privacy. Justice states that like cases should be treated alike.

An 83-year-old woman who has suffered a cerebrovascular accident and is unable to swallow refuses the insertion of a feeding tube. This is an example of what ethical principle? a) Veracity b) Justice c) Autonomy d) Nonmaleficence

Autonomy Correct Explanation: Autonomy essentially means independence and the ability to be self-directed.

The use of patient restraints limits which of the following ethical principles? a) Beneficence b) Autonomy c) Trust d) Justice

Autonomy Correct Explanation: It is important to weigh carefully the risk of limiting a person's autonomy and increasing the risk of injury by using restraints against the risk of not using restraints.

To practice ethically, the nurse should a) Avoid allowing her judgment to guide practice b) Review past cases before guiding practice c) Ask the family their views on caring d) Allow a committee to guide her practice

Avoid allowing her judgment to guide practice Explanation: Personal convictions apply only to situations and decisions pertaining to the individual. In ethical practice, nurses avoid allowing personal judgments to bias their treatment of patients.

Identify the serum lithium level for maintenance and safety. A 0.1 to 1.0 mEq/L B 0.5 to 1.5 mEq/L C 10 to 50 mEq/L D 50 to 100 mEq/L

B

The nurse hears a nursing assistant discussing a client's allergic reaction to a medication with another nursing assistant in the cafeteria. What is the highest priority nursing action? a) Document the nursing assistant?s conversation. b) Remind the nursing assistant about the client's right to privacy. c) Report the nursing assistant to the nurse manager. d) Notify the client relations department about the breech of privacy.

Remind the nursing assistant about the client's right to privacy. Correct Explanation: The nurse should first remind the nursing assistant about the client's right to privacy. All other actions are appropriate, but do not immediate protect the client's privacy.

Which of the following hormones is secreted by the juxtaglomerular apparatus? a) Aldosterone b) Antidiuretic hormone (ADH) c) Calcitonin d) Renin

Renin Explanation: Renin is a hormone directly involved in the control of arterial blood pressure; it is essential for proper functioning of the glomerulus. ADH, also known as vasopressin, plays a key role in the regulation of extracellular fluid by excreting or retaining water. Calcitonin regulates calcium and phosphorous metabolism

Nurses who prize their role in securing patient well-being are sensitive to the ethical and legal implications of nursing practice. What are examples of these ethical/legal skills? (Select all that apply.) a) Working collaboratively with the health care team as a respected and credible colleague to reach valued goals b) Being trusted to act in ways that advance the interests of patients c) Selecting nursing interventions that are most likely to yield the desired outcomes d) Being accountable for practice to oneself, the patient, the caregiving team, and society e) Using technical equipment with sufficient competence and ease to achieve goals with minimal distress to patients f) Acting as an effective patient advocate

Being trusted to act in ways that advance the interests of patients • Being accountable for practice to oneself, the patient, the caregiving team, and society • Acting as an effective patient advocate Explanation: Skills necessary in being proficient in legal/ethical competencies include: Self-motivated to act in ways that advance the interests of patients (consistently trustworthy); Accountable for practice to self, patients served, the caregiving team, and society; Consistently serve as effective patient advocate; Skilled in mediating ethical conflict among the patient, significant others, health care team, and other interested parties; Practice nursing faithful to the tenets of professional codes of ethics; and Use legal safeguards that reduce the risk of litigation. Being respected and viewed as credible meets competency needs of the interpersonal skills competency. Using technical equipment with competence to meet the needs of the patient is an example of proficiency in the technical competency category. Selecting interventions that yield desired outcomes demonstrates cognitive competency

Which ethical principle refers to the obligation to do good? a) Nonmaleficence b) Fidelity c) Veracity d) Beneficence

Beneficence Correct Explanation: Beneficence is the duty to do good and the active promotion of benevolent acts. Fidelity refers to the duty to be faithful to one's commitments. Veracity is the obligation to tell the truth. Nonmaleficence is the duty not to inflict, as well as to prevent and remove, harm; it is more binding than beneficence

A nurse is caring for a 28-year-old woman who has delivered a baby by cesarean section. She describes her pain as a 9. The nurse medicates her for pain. This is an example of which of the following ethical frameworks? a) Beneficence b) Justice c) Nonmaleficence d) Fidelity

Beneficence Correct Explanation: Beneficence means doing or promoting good. The treatment of the patient's pain is the nurse's act of doing good.

Prioritizing client care is an ongoing process within the art of nursing. Abraham Maslow proposed five levels of need and grouped them according to significance. Which client need is of primary importance? a) Being safe from falling b) Breathing easily c) Liking one's roommate d) Being able to keep up with current events while ill

Breathing easily Correct Explanation: The first-level needs, sometimes called baseline survival needs, have the highest priority. These activities, such as eating, breathing, and drinking, sustain life. Maslow believed humans could not or would not seek to fulfill higher level needs until basic physiologic needs were met. Safety and security are a secondary need, not primary. Needs related to getting along with others are important, but they are not primary needs. Needs related to feeling connected to the larger society are important, but they are not primary needs.

The nurse observes that a client with depression sat at a table with two other clients during lunch. The best feedback the nurse could give the client is which of the following? A "Do you feel better after talking with others during lunch?" B "I'm so happy to see you interacting with other clients." C "I see you were sitting with others at lunch today." D "You must feel much better than you were a few days ago."

C

Which of the following vitamins enhance the absorption of iron? a) D b) C c) A d) E

C Explanation: Vitamin C facilitates the absorption of iron. Therefore, iron supplements should be taken with a glass of orange juice or a vitamin C tablet to maximize absorption.

The instructor provides corrective information to the nursing student when the student refers to the client as the a) Woman who has diabetes b) COPDer in 216 c) Patient who is disabled d) Man with an MI

COPDer in 216 Correct Explanation: "People-first" language means referring to the person first. Examples include patient who is disabled, man with an MI, and woman who has diabetes. Using "COPDer in 216" conveys that the illness or disability is of greater importance than the person

A nurse was informed that a family member was involved in a car accident and transported to the emergency department in the same facility. What action by the nurse best demonstrates understanding of client privacy? a) Calling the client information desk to find out the room number of the family member b) Finding the emergency medical technicians that transported the family members about the injuries c) Accessing the electronic medical record of the family member to find out extent of injury d) Asking the emergency department nurse for information on the family member

Calling the client information desk to find out the room number of the family member Correct Explanation: Getting information from other health care providers violates client privacy. Health care workers must follow the same guidelines to accessing health information on people not assigned to their care.

For a hospitalized client, the physician orders morphine, 4 mg I.V., every 2 hours as needed for pain. However, the client refuses to take injections. Which nursing action is most appropriate? a) Withholding the medication until the client understands its importance b) Calling the physician to request an oral pain medication c) Administering the medication as ordered d) Explaining that no other medication can be given until the client receives the pain medication

Calling the physician to request an oral pain medication Correct Explanation: The most appropriate action is to call the physician to request an oral pain medication. By doing so, the nurse is adhering to the client's wishes. Administering an I.V. injection without client consent is considered battery and may result in a lawsuit. Withholding medication without providing an alternative and attempting to manipulate the client into taking the medication would violate the standards of care

Which of the following theories of ethics most highly prioritizes the nurse's relationship with patients and the nurse's character in the practice of ethical nursing? a) Principle-based ethics b) Utilitarianism c) Deontology d) Care-based ethics

Care-based ethics Correct Explanation: Central to the care-based approach to ethics is the nurse's relationships with patients and the nurse's "being," or character and identity. Deontology, utilitarianism, and principle-based ethics each prioritize goals and principles that exist beyond the particularities of the nursepatient relationship

A nurse is preparing to conduct a health assessment with a 78-year-old man who wears a hearing aid in his left ear. The patient is accompanied by his wife. Which of the following would be most appropriate? a) Have the patient sit at an angle to the nurse, with his good ear facing the nurse. b) Keep the examination door open during the interview so the nurse can ask for help if needed. c) Direct the interview to the patient's wife to ensure adequate information is obtained. d) Check to make sure that the patient has his hearing aid turned on and in place in his left ear.

Check to make sure that the patient has his hearing aid turned on and in place in his left ear. Correct Explanation: When obtaining information from an older adult with a hearing deficit, the nurse should make sure that the patient's hearing aid is turned on and in place in the appropriate ear. The patient should be positioned so that he can read lips and facial expressions to augment verbal communication. Distracting noises should be kept to a minimum. Keeping the door open would increase the chances of distracting noises and could violate privacy. There is no need to direct the interview to the patient's wife just because the patient has a hearing deficit.

During an emergency, a physician has asked for I.V. calcium to treat a client with hypocalcemia. The nurse should: a) Hand the physician the kind of calcium available on the unit. b) Hand the physician calcium gluconate for I.V. use. c) Hand the physician calcium chloride for I.V. use. d) Check with the physician for his complete order.

Check with the physician for his complete order. Correct Explanation: The nurse should first check with the physician for the complete order of calcium because calcium chloride has a concentration of 13.6 mEq (3.4 mmol/l) of calcium per gram and calcium gluconate has 4.65 mEq (1.2 mmol/l) of calcium per gram. The nurse can always offer the doctor the type of calcium available after the conversion in calcium has been made; otherwise, the error could be fatal

A home health care nurse who works in a low- income community assesses the risk factors for patients being serviced. What is an example of a community risk factor? a) Children are kept inside the home on a sunny summer day because of lack of recreational opportunities. b) An 80-year-old man is at risk for falls in his home due to clutter in his hallways and stairways. c) A child is born with severe mental retardation. d) A woman finds out that she is genetically inclined to develop crippling arthritis.

Children are kept inside the home on a sunny summer day because of lack of recreational opportunities. Explanation: A key component of the question is the term community. The most basic definition of a community is a specific population or group of people living in the same geographic area under similar regulations and having common values, interests, and needs. The only option above that addresses community is where children are kept inside the home on a sunny summer day because of lack of recreational opportunities. Each of the other options are focused on individuals which is not the direction of the question

Which of the following medication classifications increases aqueous fluid outflow in the patient with glaucoma? a) Alpha-adrenergic agonists b) Cholinergics c) Carbonic anhydrase inhibitors d) Beta blockers

Cholinergics Explanation: Cholinergics increase aqueous fluid outflow by contracting the ciliary muscle, causing miosis and opening the trabecular meshwork. Beta blockers decrease aqueous humor production. Alpha-adrenergic agonists decrease aqueous humor production. Carbonic anhydrase inhibitors decrease aqueous humor production

Which of the following is an accurate statement regarding cancer of the esophagus? a) Chronic irritation of the esophagus is a known risk factor. b) It is three times more common in women in the U.S. than men. c) It usually occurs in the fourth decade of life. d) It is seen more frequently in Caucasian Americans than in African Americans.

Chronic irritation of the esophagus is a known risk factor. Correct Explanation: In the United States, cancer of the esophagus has been associated with the ingestion of alcohol and the use of tobacco. In the United States, carcinoma of the esophagus occurs more than three times more often in men as in women. It is seen more frequently in African Americans than in Caucasian Americans. It usually occurs in the fifth decade of life.

The nurse is reviewing the following physician's order written for a postmenopausal woman: "calcitonin salmon nasal spray 200 IU, one spray every day." What is the appropriate action to be taken by the nurse regarding this order? a) Clarify with the physician that the spray should be given in only one nostril per day. b) Ask the physician why this medication was ordered for a postmenopausal woman. c) Inform the physician that the medication is not a nasally applied medication. d) Remind the physician that this medication can be purchased over-the-counter.

Clarify with the physician that the spray should be given in only one nostril per day. Correct Explanation: Calcitonin salmon nasal spray should be administered in only one nostril per day. Many preprinted order sheets automatically print "administer in both nostrils" when a nasal spray is ordered. Nurses must be familiar with the directions for each medication they give before administering medications. The other options are incorrect because calcitonin salmon nasal spray is prescribed to postmenopausal women for the treatment of osteoporosis and requires a physician's order.

Pentoxifylline (Trental) is a medication used for which of the following conditions? a) Thromboemboli b) Hypertension c) Elevated triglycerides d) Claudication

Claudication Correct Explanation: Trental and Pletal are the only medications specifically indicated for the treatment of claudication. Thromboemboli, hypertension, and elevated triglycerides are not indications for using Trental

A nurse suspects that a client may be experiencing post-traumatic stress disorder (PTSD). Which assessment finding would help support the nurse's suspicion? a) Client had a foot amputated due to diabetes. b) Client was a victim of a rape and beating. c) Client recently lost a spouse to a chronic illness. d) Client was just told that he has lung cancer

Client was a victim of a rape and beating. Correct Explanation: PTSD results from an overwhelming event that is outside the range of normal human experience. Rape and beating would be one such example. The diagnosis of lung cancer would most likely cause anxiety and possibly depression. Amputation of a body part or loss of a spouse would most likely lead to depression

A nurse administering medications accidentally gives a double dose of blood pressure medications. After ensuring the safety of the client, the nurse would document the error in which documents? a) Critical pathway and care plan b) Client's record and occurrence report c) Occurrence report and critical pathway d) Care plan and client's record

Client's record and occurrence report Correct Explanation: An occurrence report should be completed when a planned intervention is not implemented as ordered. The incident, with actions taken by the nurse, should also be included in the client's record. Critical pathways and care plans are not places to document occurrences.

The night nurse has completed the change of shift report. As the day nurse makes rounds on a postpartum client receiving magnesium sulfate, it is noted the client developed significantly elevated blood pressure during the past shift. Further assessment reveals the magnesium sulfate rate is infusing well below the prescribed rate. In addition to adjusting the infusion rate and notifying the health care provider (HCP), what is the most important action by the nurse? a) Ask the charge nurse if an incident report is necessary. b) Discuss the matter with the night nurse the next time she works. c) Complete an incident report. d) Evaluate the client's BP for 4 hours before making decision.

Complete an incident report. Correct Explanation: Safety is the highest priority, and a nursing error has occurred. If the day nurse decides to tell the night nurse, the timing of the notification will be up to the nurse initiating the incident report. The nurse should confer with the charge nurse concerning the incident, but completion of the report is required. Waiting for several hours to initiate the report based on changes in client data and assessment is not an ethical or professional decision and should not be considered; again, safety is the highest priority.

A nursing student is describing abstract ideas with a group of student peers. The student suggests that these ideas can be a group of objects and the relationships among them. The nursing instructor knows that the student is describing which of the following? a) Deductive reasoning b) Concepts c) Theory d) Inductive reasoning

Concepts Explanation: Concepts are abstract ideas or objects and their relationships to one another. Theory is a group of concepts and the relationships among them. Deductive reasoning provides a basis for theory development as one studies a general idea to more specific properties. Inductive reasoning is derived from consideration of specific properties to make conclusions about a general idea.

A nursing student understands which of the following is true of concepts? a) Concepts describe objects, properties, and events and relationships among them. b) Concepts provide a means of testing knowledge. c) Concepts differentiate nursing from other disciplines. d) Concepts are derived through deductive and inductive reasoning.

Concepts describe objects, properties, and events and relationships among them. Correct Explanation: Concepts describe objects, properties, and events and the relationships among them. Theories are derived through deductive and inductive reasoning and provide a mechanism by which knowledge can be tested, and nursing theory differentiates nursing from other disciplines

During the recuperation phase, a client with severe burns has become withdrawn. What concerns should the nurse explore? a) Concerns regarding how the client's family will respond b) Concerns regarding dependence and unwillingness to be discharged c) Concerns about body image and self esteem d) Concerns regarding coping abilities

Concerns about body image and self esteem Correct Explanation: During the recuperation phase, the client is likely to consider the body image implications of this injury. Sensitivity to body image and self-esteem issues are anticipated concerns. The client has suffered through the most difficult part. There are fewer concerns regarding dependence and coping abilities in the recuperation phase. The family will have been more concerned during the initial phase

The nurse is caring for a client in the hospital who has been taking an analgesic for pain related to a chronic illness and has developed a tolerance to the medication. What is the most appropriate action by the nurse? a) Consult with the physician regarding the need for an increased dose of the drug and not to reduce its dosage or frequency of administration. b) Suggest a consultation with a psychiatrist to treat the client's addiction. c) Inform the client that you will ask the physician to order a non-narcotic analgesic. d) Inform the client that he will not be able to receive more medication than the physician has ordered.

Consult with the physician regarding the need for an increased dose of the drug and not to reduce its dosage or frequency of administration. Correct Explanation: The most appropriate action by the nurse would be to consult with the physician regarding the need for an increased dose of the drug and not to reduce its dosage or frequency of administration. As a rule of thumb, an ineffective dose should be increased by 25% to 50%. Informing the client that he will not be able to receive more medication is not acting as a patient advocate nor acting in the best interest of the client. Suggesting a psychiatrist consultation would not be an appropriate action because the client has a chronic illness that requires medication. Taking a non-narcotic analgesic would not provide the client with the pain relief that he has

The nurse in the emergency department is administering a prescription for 20 mg intravenous furosemide, which is to be given immediately. The nurse scans the client's identification band and the medication barcode. The medication administration system does not verify that furosemide is prescribed for this client; however, the furosemide is prepared in the accurate unit dose for intravenous infusion. What should the nurse do next? a) Report the problem to the information technology team to have the barcode system recalibrated. b) Ask another nurse to verify the medication and the client so the medication can be given now. c) Contact the pharmacist immediately to check the order and the barcode label for accuracy. d) Administer the medication now, knowing the medication is labeled and the client is identified.

Contact the pharmacist immediately to check the order and the barcode label for accuracy. Correct Explanation: The nurse should contact the pharmacist first to be sure the medication is labeled for administration to this client. The nurse should not administer the drug until all safety precautions have been observed; the nurse should also not ask another nurse to verify the medication or client. Later, if the problem cannot be resolved with relabeling the medication, the nurse or pharmacist can contact the information technology team to check the barcode system

A nurse assesses a client with psychotic symptoms and determines that the client likely poses a safety threat and needs vest restraints. The client is adamantly opposed to this. What would be the best nursing action? a) Ask a family member to come in to supervise the client. b) Restrain the client with vest restraints. c) Apply wrist restraints instead of vest restraints. d) Contact the physician and obtain necessary orders.

Contact the physician and obtain necessary orders. Correct Explanation: If a nurse feels that a client needs to be restrained, the nurse should inform the physician and obtain necessary orders. The nurse should also discuss this with the client's family members and ask their opinion. Applying a wrist restraint instead of a vest restraint is inappropriate if a vest restraint is genuinely necessary. It would be inappropriate to delegate this aspect of care to a family member

A nurse notes that another nurse on the previous shift made an entry on the wrong client's health record. What are the most appropriate steps for the first nurse to take? a) Report to the nurse manager that the nurse needs guidance on documentation b) Contact the previous nurse requesting that the nurse correct the error c) Strike through the entry ensuring the original entry is still visible d) Rewrite the entry on the correct health record indicating who made the error

Contact the previous nurse requesting that the nurse correct the error Correct Explanation: The nurse who wrote the original record and performed the care must make the correction to health record. Nurses have a responsibility to ensure documentation is clear, accurate and concise to ensure continuity of care. The other options are incorrect because they do not follow established procedures for correcting legal medical records

The primary task of nursing research includes which of the following? a) Determining nursing diagnoses b) Managing the care of an entire caseload of patients c) Decreasing overall operating costs to the health care system d) Contributing to the scientific base of nursing practice

Contributing to the scientific base of nursing practice Correct Explanation: The primary task of nursing research is to contribute to the scientific base of nursing practice. Nursing research does not determine the appropriate nursing diagnoses or decrease the overall operating costs to the health care system. Case management is the management of care of an entire caseload of patients.

A parent informs the nurse that immunizations are against her cultural and religious beliefs and she does not want her child to receive immunizations. The nurse proceeds to inform the parent that the child will be ill all their life and will not be allowed to start school unless they are immunized. The nurse also informs the parent that she had all of her children vaccinated. What is the nurse's behavior an example of? a) Cultural conflict b) Cultural blindness c) Cultural imposition d) Stereotyping

Cultural imposition Correct Explanation: The nurse's behavior is an example of cultural imposition, defined as the tendency to impose one's cultural beliefs, practices, and values on a person from a different culture. Stereotyping is when one assumes that all members of a culture, ethnic group, or race act alike. Cultural blindness occurs when one ignores differences and proceeds as though they do not exist. Cultural conflict occurs when people become aware of cultural differences, feel threatened, and respond by ridiculing the beliefs and traditions of others to make themselves feel more secure in their own values

What is the rationale for a person taking lithium to have enough water and salt in his or her diet? A Salt and water are necessary to dilute lithium to avoid toxicity. B Water and salt convert lithium into a usable solute. C Lithium is metabolized in the liver, necessitating increased water and salt. D Lithium is a salt that has greater affinity for receptor sites than sodium chloride.

D

The nurse is evaluating a patient's social support network. The nurse evaluates that the network will assist the patient in coping with stress when which of the following is noted? a) Patient avoids situations exposing her to new people b) Son does not acknowledge his mother's diagnosis c) Patient's friends ask her for advice d) Daughter helps mother with laundry

Daughter helps mother with laundry Correct Explanation: Social networks assist in the management of stress when they provide material aid and tangible services, such as a daughter helping her mother with the laundry. Additionally, networks should provide a positive social identity and emotional support as well as access to information and new social contacts/social roles

A client from a Mediterranean country is admitted with thalassemia, jaundice, splenomegaly, and hepatomegaly. Which should be the primary focus of nursing care for this client? a) Offer foods that the client enjoys in order to increase the intake of calories. b) Decrease cardiac demands by promoting rest. c) Provide activities of daily living on the time schedule of the client's homeland. d) Listen to concerns about the hospitalization.

Decrease cardiac demands by promoting rest. Explanation: This client has clinical manifestations of thalassemia major, a disease found in descendants from the Mediterranean Sea area whose mother and father both possess a gene for thalassemia (i.e., the client is homozygous for the gene). The severe hemolytic anemia causes sequestration of red blood cells in the spleen and liver, which leads to engorgement of the organs and chronic bone marrow hyperplasia. Rest will decrease the demands on the heart due to the diminished hemoglobin level, a physiologic concern.

A nurse is caring for a client with a traumatic injury and developing tension pneumothorax. Which assessment data would be of concern? Select all that apply. a) Decreased cardiac output b) Flattened jugular veins c) Hypotension d) Tracheal deviation to the opposite side e) Tracheal deviation to the affected side f) Bradypnea

Decreased cardiac output • Hypotension • Tracheal deviation to the opposite side Explanation: Tension pneumothorax results when air in the pleural space is under higher pressure than air in the adjacent lung. The site of the rupture of the pleural space acts as a one-way valve, allowing the air to enter on inspiration but not to escape on expiration. The air presses against the mediastinum, causing a tracheal shift to the unaffected side and decreased venous return (reflected by decreased cardiac output and hypotension). Neck veins bulge with tension pneumothorax. This also leads to compensatory tachycardia and tachypnea

Nursing students are reviewing different types of mental health problems in the older adult population. They demonstrate understanding of this information when they identify which condition as the most common affective disorder? a) Schizophrenia b) Anxiety c) Depression d) Phobias

Depression Correct Explanation: Depression is the most common affective or mood disorder of old age. Although anxiety may be common, anxiety disorders including phobias are not as common as depression. Schizophrenia is a thought disorder and is less common than depression

A nurse who will be working at a local health fair is preparing a presentation about depression. When developing the presentation, which of the following would the nurse most likely include? a) Depression is more common in women than in men. b) Individuals with depression often seek treatment for it. c) Depression is a common diagnosis in health care today. d) Older adult clients frequently exhibit characteristic symptoms.

Depression is more common in women than in men. Explanation: Depression can occur at any age and is most frequently diagnosed in women than in men. It is often underdiagnosed and undertreated. Statistics reveal that only one in three people with depression is properly diagnosed and treated. Many people experience depression but seek treatment for somatic complaints, not for depression. Older adult patients may exhibit a wide range of symptoms. Nurses need to be aware that decreased mental alertness or withdrawal-type responses may indicate depression in older adults.

The client diagnosed with pneumonia is admitted to the medical unit. The nurse notes the client is taking an antidepressant medication. Which data best indicate the anti- depressant therapy is effective? 1. The client reports a "2" on a 1-10 scale, with 10 being very depressed. 2. The client reports not feeling very depressed today. 3. The client gets out of bed and completes activities of daily living. 4. The client eats 90% of all meals that are served during the shift.

Depression is subjective and the nurse does not know this client; therefore, asking the client to rate the depression on a scale best indicates the effectiveness of the medication. Any subjective data can be put on a scale to make it objec- tive. This is a very vague statement and it is not objective; therefore, it is not the best indica- tor of effectiveness of the medication. Completing ADLs indicates the client is not severely depressed, but it does not objec- tively support that the client's antidepres- sant medication is effective. Consuming 90% of the food may indicate the client is not depressed, but the nurse does not know how the client eats when severely depressed. Therefore, it is not the best indicator of the medication's effectiveness.

A nursing student is studying the principle of autonomy. Which of the following examples most accurately depicts this principle? a) Administering a morning dose of insulin before breakfast b) Changing a dressing on a wound as needed c) Describing surgery to a client before the consent is signed d) Transporting a client to a scheduled physical therapy appointment

Describing surgery to a client before the consent is signed Correct Explanation: Describing surgery to a client before a consent is signed provides the client with all of the information needed to make an informed decision, thus an autonomous one. The other choices are not reflective a client decision making

A nurse is developing a care plan for a client who is a single parent. The client is experiencing anxiety after the loss of a job and is verbalizing concerns regarding the ability to meet role expectations and financial obligations. Which of the following is most important for the nurse to include in the plan of care? a) Explore the client's obsessive thoughts that are resulting in high anxiety. b) Determine the client's ability to cope with the job loss and family obligations. c) Interview family members to determine the dynamics of the family relationships. d) Ask the client about plans for managing the financial obligations.

Determine the client's ability to cope with the job loss and family obligations. Correct Explanation: The client is experiencing stressors that are making it difficult to cope, resulting in anxiety. It is important to assess the client's coping abilities related to the job loss and meeting family obligations. This situation could become a crisis if it overwhelms the client's usual methods of coping. The client is a single parent, which also adds the burden of childcare. The client is not concerned about the family dynamics or obsessive thoughts. Responding to the client's cues is vital in addressing the client's concerns

Which of the following is a true statement regarding characteristics of chronic conditions? a) Keeping chronic conditions under control requires occasional adherence to therapeutic regimens. b) Managing chronic conditions is becoming less costly. c) Managing chronic conditions must be an individual process. d) Difficult ethical issues arise in the care of people with chronic conditions.

Difficult ethical issues arise in the care of people with chronic conditions. Explanation: Many ethical issues arise in the care of people with chronic conditions. Managing chronic conditions must be a collaborative process. Keeping chronic conditions under control requires persistent adherence to therapeutic regimens. The management of chronic conditions is becoming more costly

A nurse administers digoxin 0.125 mg to a client at 1400 instead of the prescribed dose of digoxin 0.25 mg. Which of the following statements should the nurse record in the medical record? a) Nurse accidentally gave digoxin 0.125 mg to the client at 1400. b) Digoxin 0.125 mg given at 1400 instead of prescribed dose of 0.25 mg. c) Digoxin 0.25 mg administered at 1400, physician notified. d) At 1400, wrong dose of digoxin given due to heavy workload.

Digoxin 0.125 mg given at 1400 instead of prescribed dose of 0.25 mg. Correct Explanation: The nurse should not include judgment statements, opinion, assumptions, or conclusions about what happened. The nurse should simply state the occurrence. The other options present judgment, blame, and conclusion

A client tells the visiting community health nurse that another client's name and phone number were seen on the call display after the previous day's nurse used the client's home phone. What should the nurse do in response to this conversation? a) Ask the client to visit the other client because he/she is lonely and would enjoy the company. b) Tell the client that the other client is on the nurse's list of clients, but do not disclose any further information. c) Tell the client that he/she should not be looking at the call display after the nurse leaves because it contains confidential information. d) Discuss the matter with the other nurse, reminding him/her not to use the client's phone because it has a call display feature.

Discuss the matter with the other nurse, reminding him/her not to use the client's phone because it has a call display feature. Correct Explanation: Leaving personal information in view of other people is a breach of confidentiality. The nurse should approach the other nurse and inform him/her of the incident. The other options are incorrect because they do not protect the client's privacy and do not address the behavior of the other nurse

A depressed client on a psychiatric unit asks the nurse to call the hospital lawyer to discuss writing out a will. What is the nurse's priority intervention? a) Call a lawyer as requested by the client. b) Offer the client medication for anxiety. c) Discuss thoughts and explore intent for suicide with the client. d) Inform the physician first, and place the client on suicide watch.

Discuss thoughts and explore intent for suicide with the client. Correct Explanation: Exploration of thoughts and intent are a priority based on lethality of plan for suicide. Calling the lawyer is incorrect because it does not explore the intent of the client's question. The nurse would discuss the intent with the client prior to calling the physician. Administering anxiety medication does not address the problem.

A client is brought to the emergency department (ED) by the son who states, "I am unable to care for my mother anymore." The nurses identifies this son's ethical problem as being which of the following? a) Uncertainty b) Dilemma c) Dissatisfaction d) Distress

Distress Correct Explanation: Ethical distress is when someone wants to do the right thing but is not able to. The son brings the mother to the ED to maintain her safety, although he needs to take care of her. The other choices may be part of the son's decision, however the immediate problem is one of distress

A 73-year-old man has been the primary caregiver for his wife, who has multiple sclerosis (MS). After 30 years with the disease she died and he has become increasingly withdrawn and refuses to leave the house. Which nursing diagnosis is most appropriate? a) Low self-esteem related to feeling unloved now that his spouse has passed away b) Risk for altered self-esteem related to the recent death of his spouse c) Disturbed body image related to death of spouse and loss of the role of caregiver d) Disturbed personal identity related to the unresolved crisis of his wife's death

Disturbed personal identity related to the unresolved crisis of his wife's death Correct Explanation: The client is experiencing disturbed personal identity as he is no longer a spouse or a caregiver. This is related to the recent death of his chronically ill spouse. Without her to care for he is unable to define who he is or what his role is without her. He does not have low self-esteem or disturbed body image

There are various levels of preparation for nursing practice. Which degree program will best prepare a nurse to work as a nurse researcher? a) Diploma b) Baccalaureate c) Doctoral d) Master's

Doctoral Explanation: Nurses with doctoral degrees meet requirements for academic advancement and are prepared to carry out research necessary to advance nursing theory and practice. A master's degree prepares advanced practice nurses to function in educational settings, managerial roles, as clinical specialists, and in various advanced practice areas. Baccalaureate programs provide a general educational base and incorporate the sciences and humanities. Graduates of a diploma program have a strong emphasis on clinical experiences in direct patient care and are prepared to work in acute, long-term, and ambulatory care facilities.

A client with cancer-related pain has been prescribed a narcotic analgesic to be given around the clock. The client is competent and has been actively involved in decisions regarding care. What should the nurse do if the client refuses the next dose of analgesia? a) Try to persuade the client to take the medication as ordered by the doctor. b) Document the client's choice and re-assess pain in 1 hour. c) Emphasize the rationale for taking the medication now as ordered. d) Ask the client's spouse wife to hold the client's hands while the nurse puts the pill under the tongue.

Document the client's choice and re-assess pain in 1 hour. Correct Explanation: A client has the right to choose whether to take medication. The nurse should assess the client's pain regularly and educate the client that taking the medication before the pain gets out of control will be a better pain management plan. The other options do not reflect an understanding of the client's right to choice including the refusal of pain medication.

What nurse theorist developed therapeutic touch? a) Martha Rogers b) Dolores Krieger c) Jean Watson d) Dorothea Orem

Dolores Krieger Explanation: Therapeutic touch, a healing method used by thousands of nurses and other professionals, was developed more than 30 years ago by Dora Kunz, a healer, and Dolores Krieger, an emeritus member of New York University's nursing faculty.

A nurse is being sued for malpractice in a court of law. What elements must be established to prove that malpractice or negligence has occurred? (Select all that apply.) a) Breach of duty b) Punitive damages c) Intent to harm d) Causation e) Fraud f) Duty

Duty • Breach of duty • Causation Explanation: The elements that must be established to prove that malpractice or negligence have occurred include duty, breach of duty, and causation. Intent to harm would be intentional torts. Fraud is willful and purposeful misrepresentation that could cause, or has caused, loss or harm to a person or property. Punitive damages are monetary compensation awarded in a legal case to the injured party

Which of the following terms refer to a method of recording, in graphic form, the electrical activity of the muscle? a) Electromyogram b) Electroencephalogram c) Electrocardiography d) Electrogastrography

Electromyogram Correct Explanation: Electromyogram is a method of recording, in graphic form, the electrical activity of the muscle. Electroencephalogram is a method of recording, in graphic form, the electrical activity of the brain. Electrocardiography is performed to assess the electrical activity of the heart. Electrogastrography is an electrophysiologic study performed to assess gastric motility disturbances.

The nurse assesses that her client has olfactory disturbances. Which of the following health topics would be important to teach the client? a) Eliminating disturbing odors with adequate ventilation b) Using earplugs when using loud machinery c) Protecting the skin from extremes in temperature d) Practicing oral care three times a day

Eliminating disturbing odors with adequate ventilation Explanation: Olfactory or smell disturbances can be aided by eliminating disturbing odors with adequate ventilation. Earplugs help those with auditory disturbances. Oral care is useful for those with taste disturbances. Protecting the skin is important for those with tactile disturbances.

You are caring for a client postoperatively. What nursing interventions help prevent venous stasis and other circulatory complications in a client who has undergone surgery? a) Place pillows under the client's knees or calves. b) Encourage the client to move legs frequently and do leg exercises. c) Apply pressure on the client's lower extremities. d) Maintain the client in a side-lying position.

Encourage the client to move legs frequently and do leg exercises. Correct Explanation: The nurse should encourage the client to move legs frequently and do leg exercises to prevent venous stasis and other circulatory complications. The nurse should not place pillows under the client's knees or calves unless ordered and should avoid placing pressure on the client's lower extremities. Placing the client in a side-lying position will not help prevent venous stasis and other circulatory complications in a client who has undergone surgery

The nurse is caring for a client in strict isolation and assesses that the client is apathetic and has a decreased attention span. Which of the following nursing actions should the nurse implement? a) Limit the client's use of the television. b) Encourage the client to share concerns and perceptions. c) Touch the client only when necessary. d) Provide earplugs to the client to block out noise.

Encourage the client to share concerns and perceptions. Correct Explanation: Sensory deprivation results when a person experiences decreased sensory input or input that is monotonous, unpatterned, or meaningless. A client in isolation would be in an environment with decreased stimuli and may exhibit cognitive disturbances such as a decreased attention span, and emotional disturbances, such as apathy. The nurse needs to encourage clients to share fears, concerns, and perceptions and reassure the client that misconceptions do occur with sensory deprivation. Nursing interventions focus on maintaining a sufficient level of arousal by increasing sensory stimuli. Providing earplugs and limiting television use and touch would decrease stimulation

A client tells the nurse that he is feeling depressed and low. Further assessment reveals that the client has difficulty verbalizing his feelings and needs, often feeling manipulated by others. Which of the following would the nurse suggest to the client to help relieve stress? a) Gain control over self-defeating thoughts b) Practice rephrasing thoughts that are negative or irrational c) Enroll in a class or workshop in assertiveness training d) Be realistic about how much you can accomplish

Enroll in a class or workshop in assertiveness training Explanation: The nurse should suggest that the client enroll in a class or workshop in assertiveness training. Assertive behavior enables people to act in their best interests, to stand up for themselves, to express their feelings openly and honestly, and to exercise their rights without infringing on the rights of others. Suggesting that the client practice rephrasing thoughts that are negative or irrational, be realistic about how much he can accomplish, and gain control over self-defeating thoughts are inappropriate coping strategies

You have an elderly home care patient who lives alone with no family nearby. On a recent visit, you notice that his clothes are very loose, and he has difficulty letting you leave when the visit is coming to an end. After talking with him you learn that he has not been cooking for himself and he can't get to the grocery store easily. What service could you suggest as an immediate response until a long-term plan can be formed? a) Suggest he go to an assisted living facility. b) Enroll the patient in Meals on Wheels. c) Have the social worker counsel him. d) Refer him to the dietician.

Enroll the patient in Meals on Wheels. Correct Explanation: The best option that can be arranged quickly is to enroll the patient in Meals on Wheels, a voluntary agency. The other responses may be helpful in later planning for a long-term situation

The nurse is working with a client with depression in a mental health clinic. During the interaction, the nurse uses the technique of self-disclosure. In order for this technique to be therapeutic, which of the following steps must be a priority for the nurse? a) Ensuring relevance to, and quickly refocusing upon, the client's experience b) Discussing the nurse's experience in detail c) Asking for the client's perception of what the nurse has revealed d) Allowing the client time to ask questions about the nurse's experience

Ensuring relevance to, and quickly refocusing upon, the client's experience Explanation: The nurse's self-disclosure should be brief and to the point so that the interaction can be refocused on the client's experience. Because the client is the focus of the nurse-client relationship, the discussion should not dwell on the nurse's own experience.

A client has been exposed to bacterial pneumonia. The client has been eating poorly, with limited intake of nutrients. This factor would predispose a client to the development of pneumonia. Poor nutritional intake is classified as the a) Environment b) Agent c) Host d) Disease

Environment Correct Explanation: The environment is factors that are physical, social, economic, emotional, or spiritual. They create the likelihood or the predisposition for the person to develop disease.

A patient with an esophageal disorder comes into the hospital with symptoms that include halitosis and a sour taste in the mouth. These symptoms are associated most directly with which of the following? a) Esophageal diverticula b) Hiatal hernia c) Gastroesophageal reflux d) Achalasia

Esophageal diverticula Explanation: Because the diverticula may retain decomposed food, halitosis and a sour taste in the mouth are frequent complaints. Achalasia presents as difficulty in swallowing both liquids and solids. Gastroesophageal reflux presents as burning in the esophagus, indigestion, and difficulty in, or pain upon, swallowing. Hiatal hernia presents as heartburn, regurgitation, and dysphagia in many patients, whereas at least 50% of patients are asymptomatic

A nurse is developing a plan of care for a patient diagnosed with post-traumatic stress disorder (PTSD). Which of the following would be the priority? a) Teaching coping skills for self-care b) Assisting the patient to work through the traumatic experience c) Establishing a trusting nurse-patient relationship d) Administering prescribed drug therapy

Establishing a trusting nurse-patient relationship Correct Explanation: The priority when caring for a patient with PTSD is establishing a trusting nurse-patient relationship, because the patient is physically compromised and struggling emotionally with situations that are not considered part of the normal human experience. Once trust is established, then the nurse can assist the patient in working through the traumatic experience, teach coping skills for recovery and self-care, and administer prescribed medications

Which of the following statements best conveys the concept of ethical agency? a) Ethical practice requires a skill set that must be conscientiously learned and nurtured. b) Individuals who enter the nursing profession often innately possess ethical characteristics. c) Ethical practice is best learned and fostered by surrounding oneself with people who exhibit ethical character. d) A nurse's understanding and execution of ethical practice is primarily a result of increased years of experience.

Ethical practice requires a skill set that must be conscientiously learned and nurtured. Correct Explanation: Ethical agency must be cultivated in the same way that nurses cultivate the ability to do the scientifically right thing in response to a physiologic alteration. It is inaccurate to assume that it will passively develop from the presence of other ethical practitioners or from years of experience and it is not an innate characteristic of personality

Several times you've had the opportunity to share personal prescriptions with family members when they were in need of pain medication or antibiotics. Which area of rules should govern your moral decision? a) Civil law b) Administrative law c) Common law d) Ethics

Ethics Correct Explanation: While all of the options may affect your decision, moral decisions are guided by ethics (moral principles and values that guide the behavior of honorable people).

Several times you've had the opportunity to share personal prescriptions with family members when they were in need of pain medication or antibiotics. Which area of rules should govern your moral decision? a) Common law b) Civil law c) Administrative law d) Ethics

Ethics Correct Explanation: While all of the options may affect your decision, moral decisions are guided by ethics (moral principles and values that guide the behavior of honorable people).

Using the nursing process to make ethical decisions involves following several steps. Which step is the nurse implementing when he or she reflects on the decision-making process and the role it will play in making future decisions? a) Evaluating b) Implementing c) Diagnosing d) Planning

Evaluating Correct Explanation: Evaluating an ethical decision involves reflecting on the process and evaluating those elements that will be helpful in the future. The nurse may also question how this experience can improve reasoning and decision making in the future. Diagnosing the ethical problem involves stating the problem clearly. Planning includes identifying the options and exploring the probable short-term and long-term consequences. Implementing includes the implementation of the decision and comparing the outcomes of the action with what was considered and hoped for in advance.

Using the nursing process to make ethical decisions involves following several steps. Which step is the nurse implementing when he or she reflects on the decision-making process and the role it will play in making future decisions? a) Evaluating b) Implementing c) Planning d) Diagnosing

Evaluating Explanation: Evaluating an ethical decision involves reflecting on the process and evaluating those elements that will be helpful in the future. The nurse may also question how this experience can improve reasoning and decision making in the future. Diagnosing the ethical problem involves stating the problem clearly. Planning includes identifying the options and exploring the probable short-term and long-term consequences. Implementing includes the implementation of the decision and comparing the outcomes of the action with what was considered and hoped for in advance.

A client with seizure disorder comes to the physician's office for a routine checkup. Knowing that the client takes phenytoin to control seizures, the nurse assesses for: a) Drowsiness b) Hypertension c) Tinnitus d) Excessive gum tissue growth

Excessive gum tissue growth Correct Explanation: Phenytoin can lead to excessive gum tissue growth, known as gingival hyperplasia. However, brushing the teeth two or three times daily helps retard such growth. Some clients may require excision of excessive gum tissue every 6 to 12 months. Phenytoin may cause central nervous system stimulation, leading to insomnia, nervousness, and twitching; it doesn't cause drowsiness. Other adverse reactions to phenytoin include hypotension, not hypertension; and visual disturbances, not tinnitus.

A client with acute mania exhibits euphoria, pressured speech, and flight of ideas. The client has been talking to the nurse nonstop for 5 minutes and lunch has arrived on the unit. What should the nurse do next? a) Tell the client he needs to stop talking because it is time to eat lunch. b) Do not interrupt the client, but wait for him to finish talking. c) Walk away, and approach the client in a few minutes before the food gets cold. d) Excuse oneself while telling the client to come to the dining room for lunch.

Excuse oneself while telling the client to come to the dining room for lunch. Correct Explanation: The nurse would request to be excused, showing respect and regard for the client, while telling the client to come to the dining room for lunch. Acutely manic clients need clear, concise comments and directions. Telling the client that he needs to stop talking because it is lunchtime is disrespectful and does not give the client directions for what he needs to do. Using the familiar skill of waiting without interrupting until the person pauses would not be effective with the very talkative, manic client. Walking away and approaching the client after a few minutes before the food gets cold is not helpful because the client would probably continue talking

The nurse is caring for two children in the same room. The parents of one child ask the nurse about the condition of the other child. What is the most appropriate response by the nurse? a) Share the child's information if the parents promise not to tell. b) Give the parents information about the child that is inaccurate. c) Explain that giving information would violate confidentiality. d) Provide a limited amount of information about the child.

Explain that giving information would violate confidentiality. Correct Explanation: Evoke confidentiality and stress that you would also not share information about their child to anyone inquiring who was not directly responsible for the care of that particular child. None of the other answer options would be appropriate or professional

A postpartum client's partner asks to read the client's medical record. He claims to be a nursing student and that reading it would be a learning opportunity. What action by the nurse is most appropriate? a) Tell the partner that he made a wise choice in choosing the nursing profession and allow him to review the record. b) Have the partner review the medical record in the presence of the nursing staff. c) Explain that, for client confidentiality reasons, he can't review the medical record. d) Review the medical record with the partner and teach him documentation principles.

Explain that, for client confidentiality reasons, he can't review the medical record. Correct Explanation: The nurse should explain that for confidentiality reasons he can't review the medical record. Some facilities permit family members to review the client's medical records in the presence of staff, but only with the client's permission and a physician's order.

Before seeing a newly assigned client with respiratory alkalosis, a nurse quickly reviews the client's medical history. Which condition is a predisposing factor for respiratory alkalosis? a) Opioid overdose b) Extreme anxiety c) Myasthenia gravis d) Type 1 diabetes mellitus

Extreme anxiety Correct Explanation: Extreme anxiety may lead to respiratory alkalosis by causing hyperventilation, which results in excessive carbon dioxide (CO2) loss. Other conditions that may set the stage for respiratory alkalosis include fever, heart failure, injury to the brain's respiratory center, overventilation with a mechanical ventilator, pulmonary embolism, and early salicylate intoxication. Type 1 diabetes may lead to diabetic ketoacidosis; the deep, rapid respirations occurring in this disorder (Kussmaul's respirations) don't cause excessive CO2 loss. Myasthenia gravis and opioid overdose suppress the respiratory drive, causing CO2 retention, not CO2 loss; this may lead to respiratory acidosis, not alkalosis

A parent teaches his or her children not to drink and drive; however, the parent does drink and drive. This action causes a) Conflict with society b) Disruption in consistency c) Failure to reflect own values d) Insecurity and lack of safety

Failure to reflect own values Correct Explanation: Many role models fail to reflect on their own values and, as a result, model conflict and confusion.

A patient rings the call bell to request pain medication. Upon performing the pain assessment, the nurse informs the patient that she will return with the pain medication. The nurse's promise to return with the pain medication is an example of which principle of bioethics? a) Nonmaleficence b) Fidelity c) Autonomy d) Justice

Fidelity Correct Explanation: Fidelity is keeping one's promises and never abandoning a patient entrusted to your care without first providing for the patient's needs. Autonomy respects the rights of patients or their surrogates to make healthcare decisions. Nonmaleficence is avoiding causing harm. Justice involves giving each his or her due and acting fairly

A patient rings the call bell to request pain medication. Upon performing the pain assessment, the nurse informs the patient that she will return with the pain medication. The nurse's promise to return with the pain medication is an example of which principle of bioethics? a) Autonomy b) Fidelity c) Justice d) Nonmaleficence

Fidelity Correct Explanation: Fidelity is keeping one's promises and never abandoning a patient entrusted to your care without first providing for the patient's needs. Autonomy respects the rights of patients or their surrogates to make healthcare decisions. Nonmaleficence is avoiding causing harm. Justice involves giving each his or her due and acting fairly.

A nurse states to the patient that she will keep her free of pain. However, her family wishes to try a treatment to prolong her life that may necessitate withholding pain medication. This factor will cause an ethical dilemma for the nurse in relation to which ethical principle? a) Fidelity b) Veracity c) Justice d) Autonomy

Fidelity Explanation: Fidelity means being faithful to one's commitments and promises.

A nurse is administering evening medications and notices that a medication was omitted during the day shift. Which of the following statements demonstrates the principle of accountability? a) Filling out an occurrence report and notifying the health care provider b) Documenting a narrative note in the chart about the occurrence c) Telling the client that the medication will be given the following morning d) Administering the medication with the other evening medications

Filling out an occurrence report and notifying the health care provider Correct Explanation: When an untoward occurrence occurs, the nurse will be considered to be accountable when the proper actions are taken. In this instance, an occurrence form is filled out for follow-up, and the provider is notified. The other choices are incorrect actions.

A child on a pediatric unit hits one of the other children and subsequently has video game privileges revoked for the rest of the day. The next day the same child plays with the other children without any problems in order to avoid losing video game privileges again. According to Kohlberg, the child is demonstrating what stage of development? a) First-level preconventional stage b) Trust versus mistrust c) Self-actualization d) Moralizing

First-level preconventional stage Explanation: As children progress to toddlerhood, morals and values development begins as they identify behaviors that elicit reward or punishment. Kohlberg refers to this process as the first-level preconventional stage when children learn to distinguish right from wrong and understand the choice between obedience and punishment.

A health care provider (HCP) has prescribed valproic acid for a client with bipolar disorder who has achieved limited success with lithium carbonate. Which information should the nurse teach the client about taking valproic acid? a) The extended-release tablet can be crushed if necessary for ease of swallowing. b) Tachycardia and upset stomach are common side effects. c) Consumption of a moderate amount of alcohol is safe if the medication is taken in the morning. d) Follow-up blood tests are necessary while on this medication.

Follow-up blood tests are necessary while on this medication. Correct Explanation: Valproic acid can cause hepatotoxicity, so regular liver function tests are needed. Other side effects include nausea and drowsiness. Extended-release tablets should not be split or crushed; doing so changes their absorption. Alcohol should never be mixed with this medication. There will be medication in the client's body at all times. Nausea and tachycardia are not common side effects of valproic acid.

A 76-year-old man presents to the ED complaining of "laryngitis." The triage nurse should ask if the patient has a past medical history that includes which of the following? a) Gastroesophageal reflux disease (GERD) b) Chronic obstructive pulmonary disease (COPD) c) Congestive heart failure (CHF) d) Respiratory failure (RF)

Gastroesophageal reflux disease (GERD) Explanation: The nurse should ask if the patient has a past medical history of GERD. Laryngitis in the older adults is common and may be secondary to GERD. Older adults are more likely to have impaired esophageal peristalsis and a weaker esophageal sphincter. COPD, CHF, and RF are not associated with laryngitis in the older adult

A 38-year-old female patient has been diagnosed with rheumatoid arthritis, an autoimmune disease. As the nurse completes the health history assessment she learns that the patient works as an aide at a facility that cares for children infected with AIDS, does moderate cardiovascular exercises every other day, takes no medication, has no allergies, and eats mainly a vegetarian diet with fish and chicken one to two times each week. Which of the following factors is the most important consideration in determining the status of the patient's immune system? a) Age b) Diet c) Environment d) Gender

Gender Explanation: There are differences in the immune system functions of men and women. For example, many autoimmune diseases have a higher incidence in females than in males, a phenomenon believed to be correlated with sex hormones. Autoimmune diseases tend to be more common in women because estrogen tends to enhance immunity. Androgen, on the other hand, tends to be immunosuppressive. Autoimmune diseases are a leading cause of death by disease in females of reproductive age.

During a follow-up visit 3 months following a new diagnosis of type 2 diabetes, a patient reports exercising and following a reduced-calorie diet. Assessment reveals that the patient has only lost 1 pound and did not bring the glucose-monitoring record. Which of the following tests will the nurse plan to obtain? a) Urine dipstick for glucose b) Glycosylated hemoglobin level c) Oral glucose tolerance test d) Fasting blood glucose level

Glycosylated hemoglobin level Correct Explanation: Glycosylated hemoglobin is a blood test that reflects average blood glucose levels over a period of approximately 2 to 3 months. When blood glucose levels are elevated, glucose molecules attach to hemoglobin in red blood cells. The longer the amount of glucose in the blood remains above normal, the more glucose binds to hemoglobin and the higher the glycated hemoglobin level becomes

79-year-old male patient who is postoperative day three following hip replacement surgery has been approached by a nurse researcher and asked to participate in a research study that will test a new rehabilitation strategy. What aspect of the nursing research process addresses the patient's understanding of the potential risks and benefits of this study? a) Meeting with the hospital's institutional review board (IRB) b) Giving the patient the opportunity to ask questions about the study c) Obtaining the patient's witnesses signature on a permission document d) Going through the informed consent process with the patient

Going through the informed consent process with the patient Correct Explanation: Informed consent is a process in which the details of the study and the patient's rights are explained and discussed in detail. This goes beyond simply obtaining the patient's signature or allowing him to ask questions. Meeting with the IRB is a necessary step in the research process but this does not directly affect the patient's understanding of risks and benefits.

A 79-year-old male patient who is postoperative day three following hip replacement surgery has been approached by a nurse researcher and asked to participate in a research study that will test a new rehabilitation strategy. What aspect of the nursing research process addresses the patient's understanding of the potential risks and benefits of this study? a) Obtaining the patient's witnesses signature on a permission document b) Meeting with the hospital's institutional review board (IRB) c) Giving the patient the opportunity to ask questions about the study d) Going through the informed consent process with the patient

Going through the informed consent process with the patient Explanation: Informed consent is a process in which the details of the study and the patient's rights are explained and discussed in detail. This goes beyond simply obtaining the patient's signature or allowing him to ask questions. Meeting with the IRB is a necessary step in the research process but this does not directly affect the patient's understanding of risks and benefits.

A client is typed and cross-matched for three units of packed cells. What are important precautions for the nurse to take before initiating the transfusion? Select all that apply. a) Warm the blood to room temperature. b) Take baseline vital signs. c) Initiate an IV with normal saline. d) Have two nurses check the blood type and identity. e) Initiate an IV with dextrose.

Have two nurses check the blood type and identity. • Initiate an IV with normal saline. • Take baseline vital signs. Correct Explanation: Prior to administrating blood, the unit must be checked by two registered nurses. Baseline vital signs are obtained before the transfusion is started so any changes would be identified. Blood is always transfused with normal saline as other IV fluids are incompatible with blood. Warming to room temperature is not necessary

A client asks the nurse about the use of healing touch. Which statement regarding healing touch is accurate? a) Aromatherapy is used in healing touch. b) Healing touch does not use injections. c) Supplemental music is used during healing touch. d) Healing touch is too expensive for most clients.

Healing touch does not use injections. Correct Explanation: Healing touch has been shown to be effective in helping clients relax and improve the healing process. It is not expensive because it involves no special equipment, including insertion of tubes or injections. Aromatherapy and supplemental music are not routinely used in healing touch

A nurse working in the community is involved in secondary prevention. Which activity would most likely apply? a) Health screening for diabetes risk b) Assisting with bowel retraining c) Teaching about a nutritious diet d) Providing rehabilitation exercises

Health screening for diabetes risk Correct Explanation: Secondary prevention focuses on health maintenance and is aimed at early detection and prompt intervention to prevent or minimize loss of function and independence, including interventions such as health screening and health risk appraisal. Primary prevention focuses on health promotion and prevention of illness and disease, including interventions such as teaching about healthy lifestyles, for example, a nutritious diet. Tertiary prevention focuses on minimizing deterioration and improving the quality of life, including rehabilitation such as bowel retraining and exercise to assist clients in achieveing their maximum potential

Which of the following phases of psychological reaction to rape is characterized by fear and flashbacks? a) Acute disorganization phase b) Denial phase c) Reorganization phase d) Heightened anxiety phase

Heightened anxiety phase Explanation: During the heightened anxiety phase, the patient demonstrates anxiety, hyperalertness, and psychosomatic reactions, in addition to fear and flashbacks. The acute disorganization phase is characterized by shock, disbelief, guilt, humiliation, and anger. The denial phase is characterized by an unwillingness to talk. The reorganization phase occurs when the incident is put into perspective. Some patients never fully recover from rape trauma

A 17-year-old male client is being admitted to the adolescent psychiatric unit. He was brought in by the police after beating up two male peers. The client says, "They said I was gay because I had sex with an older neighbor when I was 8 years old. I am not gay!" Which nursing interventions would be appropriate? Select all that apply. a) Discuss the client's attitude about going to jail after discharge. b) Assist the client in processing his feelings about the sexual abuse. c) Help the client express anger safely. d) Monitor the client's level of anger and potential aggression. e) Ask the client if he would like to attend a support group.

Help the client express anger safely. • Assist the client in processing his feelings about the sexual abuse. • Ask the client if he would like to attend a support group. • Monitor the client's level of anger and potential aggression. Correct Explanation: Safety of others is a priority, and the nurse must monitor the client's anger and potential for aggression. The nurse should also find safe ways for the client to express the client's anger and any other feelings about the abuse. A referral to a support group is appropriate because anger management groups are one way to assist the client in learning to manage anger. Nothing about jail is mentioned in the question. Discussion of jail does not help the client address the client's issues with anger and the abuse causing the anger.

A nurse is developing a plan of care for a family who is grieving. Which of the following would the nurse focus on first? a) Getting the family to acknowledge the pain of the loss b) Helping the family to learn to accept the loss c) Promoting the development of new activities d) Assisting the family to adapt after the loss

Helping the family to learn to accept the loss Explanation: A nurse would plan to work with a grieving family to accomplish four major tasks. The first task would be to help the family accept the loss. Next tasks would be acknolwedgement of the intensity of the pain of the loss, adaptation to life after the loss, and cultivation of new relationships and activities

Which term refers to the shifting of brain tissue from an area of high pressure to an area of low pressure? a) Herniation b) Autoregulation c) Cushing's response d) Monro-Kellie hypothesis

Herniation Correct Explanation: Herniation refers to the shifting of brain tissue from an area of high pressure to an area of lower pressure. Autoregulation is an ability of cerebral blood vessels to dilate or constrict to maintain stable cerebral blood flow despite changes in systemic arterial blood pressure. Cushing's response is the brain's attempt to restore blood flow by increasing arterial pressure to overcome the increased ICP. The Monro-Kellie hypothesis is a theory that states that, due to limited space for expansion within the skull, an increase in any one of the cranial contents causes a change in the volume of the others

Which of the following is a factor that inhibits fracture healing? a) History of diabetes b) Increased vitamin D and calcium in the diet c) Immobilization of the fracture d) Patient age of 35

History of diabetes Correct Explanation: Factors that inhibit fracture healing include diabetes, smoking, local malignancy, bone loss, extensive local trauma, age greater than 40, and infection. Factors that enhance fracture healing include proper nutrition, vitamin D and calcium, exercise, maximum bone fragment contact, proper alignment, and immobilization of the fracture

Which of the following is a modifiable risk factor for transient ischemic attacks and ischemic strokes? a) Advanced age b) Thyroid disease c) History of smoking. d) Social drinking

History of smoking. Correct Explanation: Modifiable risk factors for TIAs and ischemic stroke include hypertension, type 1 diabetes, cardiac disease, history of smoking, and chronic alcoholism. Hypertension, type 1 diabetes, and cardiac disease are modifiable risk factors for TIAs and ischemic stroke. Chronic alcoholism is a modifiable risk factor for TIAs and ischemic stroke. Advanced age, gender, and race are nonmodifiable risk factors for stroke.

The yin/yang theory of harmony and illness is rooted in which paradigm of health and illness? a) Religious b) Biomedical c) Holistic d) Scientific

Holistic Correct Explanation: One example of a naturalistic or holistic belief, held by many Asian groups, is the yin/yang theory, in which health is believed to exist when all aspects of a person are in perfect balance or harmony.

A patient's body uses physiologic mechanisms from within to respond to internal changes and maintain an essential balance.This process is known as what? a) Stress b) Self-regulation c) Homeostasis d) Fight-or-flight response

Homeostasis Correct Explanation: To maintain health, the body's internal environment must remain in a balanced state. Various physiologic mechanisms within the body respond to internal changes to maintain relative constancy in the internal environment, which is referred to as homeostasis. Stress is a condition in which the human system responds to changes in its normal balanced state, and results from a change in one's internal or external environment that is perceived as a challenge, a threat, or a danger. The Self-regulation is a mechanism that helps to maintain homeostasis. Fight-or-flight response occurs during the alarm reaction stage of the general adaptation syndrome

A client with cancer of the stomach tells the nurse, "I cannot bear the pain anymore. Please give me something to end all my suffering and this agonizing pain." The nurse faces a value conflict. Which of the following would be most important for the nurse to keep in mind in this situation? a) Human need may affect the values conflict. b) Values conflict is consistently destructive in nature. c) The responsbility is to focus solely on the values of the client. d) Value conflict has no effect on the client's compliance.

Human need may affect the values conflict. Correct Explanation: Human need may affect the values conflict. Though the client is refusing further treatment, the nurse should be aware that the client needs the treatment. The nurse should not consider only the values of the client. When faced with a values conflict, nurses should examine their own values regarding the conflict. Value conflict may affect the client's compliance. Values conflict is not always destructive in nature. At times, it may even be constructive

A nurse notes that a patient admitted to a long-term care facility sleeps for an abnormally long time. After researching sleep disorders, the nurse learns that which area of this patient's brain may have suffered damage? a) Midbrain b) Medulla c) Cerebral cortex d) Hypothalamus

Hypothalamus Explanation: The hypothalamus has control centers for several involuntary activities of the body, one of which concerns sleeping and waking. Injury to the hypothalamus may cause a person to sleep for abnormally long periods. The medulla and midbrain are part of the reticular activating system (RAS) which plays a part in the cyclic nature of sleep. The cerebral cortex does not have any role in the sleep process

Students are reviewing information about substance abuse and its effects on individuals and families. The students demonstrate understanding of this topic when they identify which of the following? a) Substance abuse applies primarily to the use of illegal drugs. b) Individuals use substances to enhance their decision-making ability. c) Individuals with substance abuse often have difficulty using adaptive behaviors. d) Substance abuse is most frequently seen in outpatient settings.

Individuals with substance abuse often have difficulty using adaptive behaviors. Correct Explanation: Substance abuse refers to the use of alcohol and illegally obtained, prescribed, or over-the-counter drugs alone or combined in ineffective attempts to cope with the pressures, strains, and burdens of life. Thus, individuals with substance abuse often have difficulty identifying and implementing adaptive behaviors. Substance abuse occurs in all settings. Individuals who abuse substances are unable to make healthy decisions and to solve problems effectively.

When completing the preoperative checklist on the nursing unit, the nurse discovers an allergy that the client has not reported. What should the nurse do first? a) Inform the anesthesiologist. b) Note this new allergy prominently on the medical record. c) Contact the scrub nurse in the operating room. d) Administer the prescribed preanesthetic medication.

Inform the anesthesiologist. Correct Explanation: The anesthesiologist who administers the anesthetic agent and monitors the client's physical status throughout the surgery must have knowledge of all known allergies for client safety. The completed record (with the preoperative checklist) must be available to all members of the surgical team, and any unusual last-minute observations that may have a bearing on anesthesia or surgery are noted prominently at the front of the medical record. The preanesthetic medication can cause light-headedness or drowsiness. The nurse in the scrub role provides sterile instruments and supplies to the surgeon during the procedure

A client who tested positive for human immunodeficiency virus (HIV) and has pancreatitis is admitted to the medical unit. The nurse director from another unit comes into the medical unit nurses' station and begins reading the client's chart. The staff nurse questions the director about reading the client's chart. The director states that the client is her neighbor's son. What action should the nurse take to protect the client's right to privacy? a) Inform the nurse director she's violating the client's right to privacy and ask her to return the chart. b) Remind the nurse director not to share the client's medical information with anyone because of his HIV status. c) Report the incident to the medical director. d) Ask the nurse director if she has permission to read the client's chart, and if she doesn't, tell her she needs to obtain it before further reading.

Inform the nurse director she's violating the client's right to privacy and ask her to return the chart. Explanation: Personal health information may not be used for purposes not related to health care. The nurse director found reading the chart isn't providing health care to the client and, therefore, doesn't require access to the chart. The nurse should confront the nurse director and ask her to return the client's chart. The director shouldn't have access to this client's health care information regardless of his HIV status. If she doesn't comply with the nurse's request, the nurse should report the incident to her nurse manager, so the infraction can be reported through the proper channels. The staff nurse shouldn't report the incident to the medical director. Asking the nurse director if she has permission to read the chart doesn't protect client confidentiality

A visitor to the surgical unit asks the nurse about another client on the unit. The visitor viewed the client's name on the computer screen of another nurse at the nurses' station and recognized the client as a relative. What is the first action of the nurse in relation to this situation? a) Confirm that the client is on the unit but offer no further details. b) Validate the relationship of the visitor to the client before discussing the client's status. c) Notify security that the visitor viewed confidential client information. d) Inform the other nurse that the viewed screen resulted in a breach of confidentiality.

Inform the other nurse that the viewed screen resulted in a breach of confidentiality. Correct Explanation: Nurses must protect the privacy of all client information, and this includes information on an electronic medical record. The computer screen at the nurses' station should not be in view of anyone other than the person accessing the record. The other answers are incorrect because they breach client confidentiality.

In the delivery of care, the nurse acts in accordance with nursing standards and the code of ethics and reports a medication error that she has made. The nurse is most clearly demonstrating which of the following professional values? a) Integrity b) Altruism c) Human dignity d) Social justice

Integrity Correct Explanation: The nurse is demonstrating integrity, which is defined as acting in accordance with an appropriate code of ethics and accepted standards of practice. Seeking to remedy errors made by self or others is an example of integrity. Altruism is a concern for the welfare and being of others. Social justice is upholding moral, legal, and humanistic principles. Human dignity is respect for the inherent worth and uniqueness of individuals and populations

In the delivery of care, the nurse acts in accordance with nursing standards and the code of ethics and reports a medication error that she has made. The nurse is most clearly demonstrating which of the following professional values? a) Social justice b) Integrity c) Altruism d) Human dignity

Integrity Correct Explanation: The nurse is demonstrating integrity, which is defined as acting in accordance with an appropriate code of ethics and accepted standards of practice. Seeking to remedy errors made by self or others is an example of integrity. Altruism is a concern for the welfare and being of others. Social justice is upholding moral, legal, and humanistic principles. Human dignity is respect for the inherent worth and uniqueness of individuals and populations

A client who is scheduled to have surgery for a hernia the next day is anxious about the whole procedure. The nurse assures the client that surgery for hernias is very common and that the prognosis is very good. What skills of the nurse are reflected here? a) Imaginal skills b) Systems skill c) Interpersonal skills d) Instrumental skills

Interpersonal skills Correct Explanation: The scenario reflects the nurse's interpersonal skills. It shows how a person relates with others. The nurse shows imaginal skills when he or she envisions a plan for adapting and personalizing client care. Instrumental skills are associated with basic physical and intellectual competencies. Systems skills are those that help the nurse see the whole picture and how various parts relate

A client scheduled to have a surgery for a hernia the next day is anxious about the whole procedure. The nurse assures the client that surgery for hernias is very common and that the prognosis is very good. What skills of the nurse are reflected here? a) Interpersonal skills. b) Instrumental skills. c) Systems skills. d) Imaginal skills.

Interpersonal skills. Correct Explanation: The scenario reflects the nurse's interpersonal skills. It shows how a person relates with others. The nurse shows imaginal skills when he or she envisions a plan for adapting and personalizing client care. Instrumental skills are associated with basic physical and intellectual competencies. Systems skills are those that help the nurse see the whole picture and how various parts relate

A patient is being seen in the ER following a motor vehicle accident (MVA). He is having severe back pain. The preferred route of administration of medication in the most acute care situations is which of the following routes? a) Intramuscular b) Intravenous c) Subcutaneous d) Epidural

Intravenous Correct Explanation: The IV route is the preferred parenteral route in most acute care situations because it is much more comfortable for the patient, and peak serum levels and pain relief occur more rapidly and reliably. Epidural administration is used to control postoperative and chronic pain. Subcutaneous administration results in slow absorption of medication. Intramuscular administration of medication is absorbed more slowly than intravenously administered medication.

A patient is being seen in the ER following a motor vehicle accident (MVA). He is having severe back pain. The preferred route of administration of medication in the most acute care situations is which of the following routes? a) Intramuscular b) Epidural c) Subcutaneous d) Intravenous

Intravenous Explanation: The IV route is the preferred parenteral route in most acute care situations because it is much more comfortable for the patient, and peak serum levels and pain relief occur more rapidly and reliably. Epidural administration is used to control postoperative and chronic pain. Subcutaneous administration results in slow absorption of medication. Intramuscular administration of medication is absorbed more slowly than intravenously administered medication

The client with recurring depression will be discharged from the psychiatric unit. Which suggestion to the family is most important to include in the plan of care? a) Encourage the client to sleep as much as possible. b) Discourage visitors while the client is at home. c) Provide for a schedule of activities outside the home. d) Involve the client in usual at-home activities.

Involve the client in usual at-home activities. Correct Explanation: It is best to involve the client in usual at-home activities as much as the client can tolerate them. Discouraging visitors may not be in the client's best interest because visits with supportive significant others will help reinforce supportive relationships, which are important to the client's self-worth and self-esteem. Providing for a schedule of activities outside the home may be overwhelming for the client initially. Involving the client in planning for outside activities would be appropriate. Encouraging the client to sleep as much as possible is nontherapeutic and promotes withdrawal from others.

You are the nurse caring for a client with an enlarged thyroid gland. You anticipate which nutritional deficiency is linked to the client's condition? a) Iodine b) Sodium c) Potassium d) Magnesium

Iodine Correct Explanation: A chronic deficiency of iodine can lead to goiter, which manifests as an enlargement of the thyroid gland.

The nurse in a free clinic when caring for clients uses the Health Belief Model, which is based on three components. What is the main focus for this model? a) It focuses on how health is a constantly changing state. b) It focuses on what people believe to be true about their health. c) It focuses on how people interact with their environments. d) It focuses on factors that predispose a person to infectious diseases.

It focuses on what people believe to be true about their health. Explanation: The Health Belief Model focuses on what people perceive or believe to be true about themselves in relation to their health. The Health Promotion Model focuses on how people interact with their environments, as they pursue health. The Health Illness Continuum Model focuses on health as a constantly ever-chaining state, while The Agent-Host-Enviornemnt Model explains how certain factors place a person at risk for an infectious disease

A registered nurse checks the ANA regulations prior to delegating tasks to UAPs on a burn unit. Which principles regarding the regulation, education, and use of the UAP are recommended by the American Nurses Association? (Select all that apply.) a) It is the role of the LPN to assign nursing duties to the UAP. b) It is the purpose of assistive personnel to work in a supportive role to the registered nurse. c) It is the health care institution that determines the scope of nursing practice. d) It is the LPN who supervises any assistant involved in providing direct patient care. e) It is the registered nurse who is responsible and accountable for nursing practice. f) It is the role of the assistive personnel to carry out tasks to enable the professional nurse to concentrate on nursing care for the patient.

It is the registered nurse who is responsible and accountable for nursing practice. • It is the purpose of assistive personnel to work in a supportive role to the registered nurse. • It is the role of the assistive personnel to carry out tasks to enable the professional nurse to concentrate on nursing care for the patient. Correct Explanation: The nurse must be familiar with the delegation guidelines when working as a registered nurse. The purpose of assistive personnel is to work in a supportive role to the registered nurse. It is the role of the assistive personnel to carry out tasks to enable the professional nurse to concentrate on nursing care for the client. It is the registered nurse who is responsible and accountable for nursing practice. The LPN does not supervise the assistant, the RN does. It is not the role of the LPN, rather the RN, to assign nursing duties

You work in a long-term care facility. One of your patients is an elderly man who is very confused. What ethical dilemma is posed when using restraints in a long-term care setting? a) It prevents self-directed care b) It threatens autonomy c) It limits personal safety d) It increases confusion

It threatens autonomy Correct Explanation: Because there are safety risks involved when using restraints on elderly confused patients, this is a common ethical problem in long-term care settings, as well as other health care settings. Restraints limit the individual's autonomy because they are perceived as imprisonment. Restraints should not limit personal safety. Oftentimes restraints increase confusion, and they prevent self-directed care

A nurse is providing care for three patients on a medical unit, two of whom are significantly more acute than the third. The nurse is making a concerted effort to ensure that the less acute patient still receives a reasonable amount of time, attention, and care during the course of the shift. Which of the following is the nurse attempting to enact? a) Beneficence b) Fidelity c) Justice d) Nonmaleficence

Justice Explanation: The ethical principle of justice includes an effort to fairly distribute benefits and to minimize discrimination, even when circumstances make this difficult to achieve. This is demonstrated by the nurse's efforts to fairly distribute his or her time and care.

A nurse is working with young adults and assessing their moral development. The nurse relates which of the following theorists who developed the Theory of Moral Development? a) Lawrence Kohlberg b) Sigmund Freud c) Erik Erikson d) Robert Havighurst

Lawrence Kohlberg Explanation: Lawrence Kohlberg developed the Theory of Moral Development. Erik Erikson developed the Theory of Psychosocial Development. Robert Havighurst described learned behaviors as developmental tasks that occur through various times in a human's life. Sigmund Freud developed the theory of Psychoanalytic Development

While caring for a Hispanic client, the nurse inadvertently offends the client. What is the best action by the nurse? a) Examine the interaction and focus on the majority culture. b) Learn from the mistake and do not repeat it. c) Recognize that there is a cultural bias that led to the mistake. d) Ask the client why she is so mad.

Learn from the mistake and do not repeat it. Explanation: All nurses makes mistakes at some time when caring for culturally diverse clients. The best action is to learn from the mistake and not repeat the offense. While it may be appropriate to discuss with the client, asking her why she is so mad is aggressive and may make the situation worse. The mistake was inadvertent and may not be the result of cultural bias. Focusing on your own majority culture will not help bring about learning associated with the mistake

A client has been using Chinese herbs and acupuncture to maintain health. What is the best response by the nurse when asked if this practice could be continued during recuperation from a long illness? a) "Once you have recovered from this illness, you can go back to your traditional ways." b) "Let's discuss your desire to integrate these practices with the physician and advocate on your behalf." c) "Have you spoken to the physician about using the Chinese herbs and acupuncture?" d) "What do you want to accomplish by using these methods rather than researched practices?"

Let's discuss your desire to integrate these practices with the physician and advocate on your behalf." Correct Explanation: The client has a right to incorporate some of the traditional Chinese therapies. It is important to be respectful of cultural beliefs and to advocate for the client. Contacting the physician is important because there could be herbal-drug interactions. Each of the other choices does not respect cultural choices or explore the possibility of interactions. Openness with health care members is important because clients may choose to integrate these therapies without notifying the nurse or the physician

Which of the following actions most clearly demonstrates a nurse's commitment to social justice? a) Ensuring that a hospital patient's diet is culturally acceptable b) Lobbying for an expansion of Medicare eligibility and benefits c) Answering a patient's questions about her care clearly and accurately d) Documenting nursing care in a timely, honest, and thorough manner

Lobbying for an expansion of Medicare eligibility and benefits Correct Explanation: Social justice is a professional value that encompasses efforts to promote universal access to healthcare, such as the expansion of publicly funded programs like Medicare. Culturally competent care is a reflection of human dignity while answering patients' questions and documenting accurately are expressions of the value of integrity

In evaluating the therapeutic outcome of using biofeedback to help a client work through a stressful situation, the nurse would expect the client to do which of the following? a) Redemonstrate a home care procedure properly b) Maintain vital signs within normal limits c) Verbalize the actions and side effects of his medications d) Identify support personnel and services

Maintain vital signs within normal limits Correct Explanation: Biofeedback is the relaxation technique that involves clients learning to use mental processes to control their physiologic responses to events. Clients can learn to alter their autonomic nervous system and control their vital signs and uncomfortable symptoms. Identifying support services, redemonstrating procedures, and understanding medications can reduce stressful feelings, but they are not biofeedback principles

While ambulating, a client who had an open cholecystectomy complains of feeling dizzy and then falls to the floor. After attending to the client, a nurse completes an incident report. Which action by the nurse should the charge nurse correct? a) Making a copy of the incident report for the client b) Submitting the incident report to the appropriate hospital administrator c) Notifying the physician of the incident and the client's condition d) Documenting the incident factually in her nurses' notes

Making a copy of the incident report for the client Correct Explanation: A nurse shouldn't copy an incident report for anyone. An incident report is a confidential and privileged document available to agency personnel for risk-management activities. After completing the report, the nurse should submit it according to facility policy. The nurse should document the incident factually in the client's record and notify the physician of the incident and the client's condition

During therapy, a client on the mental health unit is restless and is starting to make sarcastic remarks to others in the therapy session. The nurse responds by saying, "you look angry." Which of the following communication techniques is the nurse using? a) Making observations b) Reaffirming c) Clarification d) Mirroring

Making observations Explanation: The nurse has provided direct feedback as an observation to the client and the group. The nurse is not mirroring the behavior or seeking clarification or an explanation of the behavior. This is not an open-ended question. Making direct observations and providing feedback in this manner is useful in demonstrating attention and concern for group members as well as providing an external vantage point on behaviors exhibited in a group setting. While such a statement makes a space for later clarification, this statement itself if not a statement of clarification, it is simply an observation

Flu and cold season offers excellent examples of physiologic reflexes to ward off illness. One problem is that an effective mechanical defense for one person can complete a link in the chain of infection for someone else. To which link is the above referring? a) Means of transmission b) Reservoir c) Portal of entry d) Infectious agent

Means of transmission Explanation: As a person sneezes or coughs, if he or she doesn't cover his or her mouth and nose, the airborne microbes can be spread to others, finding a susceptible host. Covering up when coughing or sneezing is vital protection against infection. The reservoir refers to the environment in which the infectious agent can survive and reproduce. This refers to the route by which the infectious agent escapes from the environment in which it lives and reproduces. This refers to the agent that has the power to produce disease

Which of the following terms best describes a living will? a) Durable power of attorney for health b) Medical directive c) Health care power of attorney d) Proxy directive

Medical directive Explanation: A living will is a type of advance medical directive in which the individual, who is of sound mind, documents treatment preferences. A proxy directive is the appointment and authorization of another individual to make medical decisions on behalf of the person who created an advance directive when he or she is no longer able to speak for him or herself. Health care power of attorney is a legal document that enables the signer to designate another individual to make health care decisions on his or her behalf when he or she is unable to do so.

The physician has ordered that the client should ambulate three times a day. The nurse enters the room to ambulate the client and the client complains of pain. What is the nurse's most appropriate action? a) Explain to the client the benefits of ambulation. b) Medicate the client and wait to ambulate later. c) Emphasize to the client the importance of following the treatment plan. d) Ambulate the client and medicate later.

Medicate the client and wait to ambulate later. Explanation: It is most appropriate to manage the client's pain first. The client will be able to ambulate more easily and it is not necessary to cause the client further pain. Ambulating first considers the needs of the nurse, not the client. The client has not indicated misunderstanding of benefits or the importance of ambulation

Which of the following describes vertigo? Select all that apply. a) Fainting b) Misperception of motion c) Objects are moving around him or her d) Spinning sensation e) Syncope

Misperception of motion • Spinning sensation • Objects are moving around him or her Correct Explanation: Vertigo is defined as the misperception or illusion of motion of the person or their surroundings. Most people with vertigo describe a spinning sensation or say they feel as though objects are moving around them. Syncope, fainting, and loss of consciousness are not forms of vertigo and usually indicate disease in the cardiovascular system.

A client diagnosed with schizoaffective disorder is suffering from schizophrenia with elements of which other disorder? a) Thought disorder b) Amnestic disorder c) Mood disorder d) Personality disorder

Mood disorder Correct Explanation: According to the Diagnostic and Statistical Manual of Mental Disorders, Fifth Edition, schizoaffective disorder refers to schizophrenia with elements of a mood disorder, either mania or depression. The prognosis is generally better for schizoaffective disorder than for the other types of schizophrenia, but it's worse than the prognosis for a mood disorder alone. Personality disorders and psychotic illness aren't listed together on the same axis. Schizophrenia is a thought disorder in itself. Clients with schizoaffective disorder don't have an amnestic disorder.

When a patient says, "I don't care if I get better; I have nothing to live for, anyway," which type of counseling would be appropriate? a) Short-term counseling b) Long-term counseling c) Motivational counseling d) Professional counseling

Motivational counseling Explanation: The most appropriate counseling for the situation at hand would be motivational counseling. With motivational counseling, the nurse would discuss feelings and incentives with the client. Short-term counseling focuses on the immediate problem or concern of the patient or family. It can be a relatively minor concern or a major crisis, but in any case, it needs immediate attention. Long-term counseling extends over a prolonged period. A patient might need the counsel of the nurse at daily, weekly, or monthly intervals. A patient experiencing a developmental crisis, for example, might need long-term counseling. Professional counseling is a general term

Which neurons transmit impulses from the CNS? a) Dendrites b) Motor c) Sensory d) Neurilemma

Motor Correct Explanation: Neurons are either sensory or motor. Sensory neurons transmit impulses to the CNS; motor neurons transmit impulses from the CNS. A membranous sheath called the neurilemma covers the myelin of axons in peripheral nerves. Dendrites are threadlike projections or fibers.

Mr. S. was diagnosed with type 2 diabetes mellitus 3 years ago, but has failed to integrate regular blood glucose monitoring or dietary modifications into his lifestyle. He has been admitted to the hospital for treatment of acute renal failure secondary to diabetic nephropathy, an event that has prompted Mr. S. to reassess his values. Which of the following actions most clearly demonstrates that Mr. S. is engaging in the step of prizing within his valuing process? a) Mr. S. is now able to explain how his choices have contributed to his renal failure. b) Mr. S. expresses remorse at how his failure to take make lifestyle changes has adversely affected his health. c) Mr. S expresses pride that he now has the knowledge and skills to take control of his diabetes management. d) Mr. S. states that he will now begin to check his blood glucose before each meal and at bedtime.

Mr. S expresses pride that he now has the knowledge and skills to take control of his diabetes management. Correct Explanation: Within the valuing process, expressions of pride and happiness are considered to be indications of prizing. Resolving to make changes is an aspect of choosing while expressing insight about his role in his current diagnosis demonstrates that Mr. S. has the desire to re-examine his values

Hypocalcemia is associated with which of the following manifestations? a) Muscle twitching b) Polyuria c) Bowel hypomotility d) Fatigue

Muscle twitching Correct Explanation: Clinical manifestations of hypocalcemia include paresthesias and fasciculations (muscle twitching). Bowel hypomotility, fatigue, and polyuria are associated with hypocalcemia

Which of the following "awareness contexts" is characterized by the patient, family, and health care professionals understanding that the patient is dying, but pretending otherwise? a) Suspected awareness b) Closed awareness c) Open awareness d) Mutual pretense awareness

Mutual pretense awareness Correct Explanation: In mutual pretense awareness, the patient, the family, and the health care professionals are aware that the patient is dying, but all pretend otherwise. In closed awareness, the patient is unaware of his or her terminality in a context where others are aware. In suspected awareness, the patient suspects what others know and attempts to find it out. In open awareness, all are aware that the patient is dying and are able to acknowledge that reality openly.

To avoid the side effects of corticosteroids which medication classification is used as an alternative to treating inflammatory conditions of the eyes? a) NSAIDS b) Cycloplegics c) Mydriatics d) Miotics

NSAIDS Correct Explanation: NSAIDS are used as an alternative in controlling inflammatory eye conditions and postoperatively to reduce inflammation. Miotics are used to cause the pupil to constrict. Mydriatics cause the pupil to dilate. Cycloplegics cause paralysis of the iris sphincter.

A home health nurse who performs a careful safety assessment of the home of a frail elderly patient to prevent harm to the patient is acting in accord with which of the following, a principle of bioethics? a) Values b) Advocacy c) Nonmaleficence d) Morals

Nonmaleficence Correct Explanation: Nonmaleficence is a principle of bioethics and is defined as the obligation to prevent harm. Advocacy, morals, and values are not principles of bioethics.

When assessing if a procedural risk to a client is justified, the ethical principle underlying the dilemma is known as which of the following? a) Informed consent b) Pro-choice c) Self-determination d) Nonmaleficence

Nonmaleficence Correct Explanation: Nonmaleficence is the principle of creating no harm. It refers to preventing or minimizing harm to an individual. The other options do not represent the situation presented in the question

An emergency department nurse and healthcare team, caring for a 2-year-old, semiconscious child with numerous fractures and evidence of cigarette burns, suspect child abuse. The nurse reports the family to the child abuse hotline. The nurse is following which ethical principle? a) Justice b) Nonmaleficence c) Beneficence d) Fidelity

Nonmaleficence Correct Explanation: The principle of nonmaleficence means to avoid doing harm, to remove harm, and to prevent harm.

A nursing supervisor asks a pediatric nurse to work temporarily (float) in the intensive care unit (ICU) because there are few clients in the pediatric unit. The pediatric nurse has never worked in ICU and has no intensive care experience. Which action should this nurse take? a) Notify the nursing supervisor that she feels unqualified and untrained for the assignment. b) Report to the ICU, tell the ICU nurses she has never worked in the ICU, and let the nurses decide what tasks she can perform. c) Refuse to float to the ICU. d) Report to the ICU and accept a total client assignment; ask the nurses for assistance when necessary.

Notify the nursing supervisor that she feels unqualified and untrained for the assignment. Correct Explanation: The pediatric nurse should notify the nursing supervisor about feeling unqualified and untrained to float in the ICU. The nursing supervisor can advise the pediatric nurse about tasks she is qualified to perform in the ICU without jeopardizing her nursing license. When the census on a unit is low, many facilities use staff to float to another unit as a cost-effective and reasonable way for managing resources. Having the ICU nurses determine what tasks the pediatric nurse can perform makes the ICU nurses responsible for the pediatirc nurse's performance. However, the nursing supervisor should make those decisions because the supervisor knows the overall needs of the facility and can, therefore, best allocate nursing resources. A nurse should never accept responsibility for a total client care assignment if she doesn't have the skills to plan and deliver care.

A nurse is caring for a client who has hypertension and diabetes mellitus. The client's blood pressure this morning was 150/92 mm Hg. He asks the nurse what his blood pressure should be. The nurse's most appropriate response is: a) "The lower the better. Blood pressure of 120/80 mm Hg is best for everyone." b) "Clients with diabetes should have a lower blood pressure goal. You should strive for 130/80 mm Hg." c) "Your blood pressure is a little high. I will take it again in an hour." d) "The current recommendation is for everyone to have blood pressure of 140/90 mm Hg or lower."

"Clients with diabetes should have a lower blood pressure goal. You should strive for 130/80 mm Hg." Explanation: An individual with diabetes mellitus should strive for blood pressure of 130/80 mm Hg or less. An individual without diabetes should strive for blood pressure of 140/90 mm Hg or less

Which question would the nurse ask to determine a client's coping abilities during a lengthy hospital stay? a) "What are the worst challenges that you have faced?" b) "How is this illness impacting you and your family?" c) "How can we take away your worries while you are in the hospital?" d) "What could you have done to prevent this illness?"

"How is this illness impacting you and your family?" Explanation: This question helps address how illness affects the client as well as the family. This question seeks to assess the impact of the stressor and coping abilities. It also examines how the support system, the family, is responding. It is too late to address prevention issues. Taking away worries is not realistic because the client needs to work through concerns. Asking about worst challenges changes the topic of what the client is experiencing right now.

The nurse is educating a client who insists that the newly prescribed imipramine is not working for her feelings of depression. When evaluating the client's statement, which question is most important to ask first? a) "Do you feel worse since taking the medication?" b) "What time of day are you taking the medication?" c) "What is the dosage of medication that you are prescribed?" d) "How long have you been taking the medication?"

"How long have you been taking the medication?" Correct Explanation: Clients are often hopeful of positive results when a new medication is prescribed. It is frustrating to the client when symptom relief does not occur in a time frame which the client feels is acceptable. Understanding that symptom relief takes time, the nurse's next question is to ask how long she has been taking the medication. The nurse is correct to realize that one disadvantage of cyclic antidepressants is the lag time between initiation of drug therapy and relief of depressive symptoms. Nursing instruction includes maintaining the medication for at least a month before medication adjustments are made. Confirming the other questions is appropriate

The nurse is suctioning a client who had a laryngectomy. What is the maximum amount of time the nurse should suction the client? a) 20 seconds b) 10 seconds c) 25 seconds d) 30 seconds

10 seconds Correct Explanation: A client should be suctioned for no longer than 10 seconds at a time. Suctioning for longer than 10 seconds may reduce the client's oxygen level so much that he becomes hypoxic.

A pt has been taking bupropion (Wellbutrin) for more that 1 year. The pt has been in a car accident with LOC and is brought to the ED. Which reason would the nurse question the continued use of this medication? 1 the pt may have possible injury to the GI system 2 the pt is at risk for seizures from a potential closed head injury 3 the pt is at increased risk of bleeding while taking bupropion 4 the pt may experience sedation from bupropion, making assessment difficult

2

A nurse is caring for a patient diagnosed with depression in the mental health unit. The nurse understands that therapeutic effects of tricyclic antidepressants occur at which timeframe?? a) 2 weeks b) 3 weeks c) 1 week d) 4 weeks

3 weeks Explanation: Patients need to know that a therapeutic effect may not take effect until they have taken the medication for 3 weeks. The other timeframes are incorrect.

Which medication would be classified as a tricyclic antidepressant? 1 bupropion (wellbutrin) 2 mirtazapine (remeron) 3 citalopram (celexa) 4 nortriptyline (pamelor)

4

Which of the following typifies the speech of a person in the acute phase of mania? A Flight of ideas B Psychomotor retardation C Hesitant D Mutism

A

The yin and yang theory of illness proposes that the seat of energy in the body is within a specific area. Which of the following is the correct area of the body? a) Reproductive system b) Pulmonary system c) Autonomic nervous system d) Cardiac system

Autonomic nervous system Correct Explanation: The yin/yang theory proposes that all organisms and objects in the universe consist of yin and yang energy. The seat of the energy forces is within the autonomic nervous system, where balance between the opposing forces is maintained during health

Which of the following is considered a stimulant laxative? a) Bisacodyl (Dulcolax) b) Psyllium hydrophilic mucilloid (Metamucil) c) Mineral oil d) Magnesium hydroxide (milk of Magnesia)

Bisacodyl (Dulcolax) Explanation: Dulcolax is a stimulant laxative. Milk of magnesia a saline agent. Mineral oil is a lubricant. Metamucil is a bulk forming agent.

Which of the following type of shock are older adults more likely to develop? a) Neurogenic shock b) Cardiogenic shock c) Anaphylactic shock d) Septic shock

Cardiogenic shock Correct Explanation: Older adults, particularly those with cardiac disease, are prone to cardiogenic shock. Older adults are not prone to developing neurogenic, septic, or anaphylactic shock.

Members of which of the following religious traditions are likely to have the most stringent restrictions and parameters placed on their medical care? a) Buddhism b) Protestantism c) Christian Scientist d) Hinduism

Christian Scientist Correct Explanation: Christian Scientism places significant restrictions of the use of drugs, medical procedures, therapies, and surgeries. The scope of these restrictions greatly exceeds that dictated by Hinduism, Protestant Christianity, and Buddhism.

The nurse is preparing to give the client a bath early in the morning. The client states, "I prefer to take my bath at night. It helps me sleep." What is the nurse's most appropriate action? a) Reschedule the client's bath to the evening shift. b) Ask the client for permission to give the bath in the morning. c) Tell the client that the physician has ordered sleep medication if necessary. d) Determine if the nurses have time to give the client's bath at night.

Reschedule the client's bath to the evening shift. Correct Explanation: The client's preferences are a primary consideration in scheduling interventions. The client's preference to have a bath at night requires a change in scheduling. Asking for permission to give the bath in the morning does not address the client's preference. The schedule of the nurses should not take priority over client needs. Informing the client about sleep medication does not address the client's preference.

A client with five children suspects that she is pregnant again. She is also going through a financial crisis and asks the nurse for a medicine to cause abortion. The nurse is under obligation to promote family planning but feels the need to save the unborn fetus. What value is the nurse considering more important? a) Choice b) Respect for life c) Esthetics d) Accountability

Respect for life Correct Explanation: In this situation, the nurse values respect for life more important than anything else. Choice, accountability, and esthetics are not considered important in this situation. The nurse who values choice as most important would let the client decide whether she wants an abortion or not. The nurse who values accountability documents nursing care accurately and honestly. The nurse who adapts the environment so that it is pleasing to the client values esthetics

A nurse is conducting a physical assessment on an adolescent who doesn't want her parents informed that she had an abortion in the past. Which statement best describes the information security measures the nurse can implement in this situation? a) Because the adolescent is a minor, inform her parents about her medical history. b) Before agreeing to maintain confidentiality, determine whether the adolescent is an emancipated minor. c) Respect the adolescent's wishes and maintain her confidentiality. d) Discussing the adolescent's medical history with her parents and thoroughly document it in the medical record.

Respect the adolescent's wishes and maintain her confidentiality. Correct Explanation: The nurse should respect the rights of minors who don't want parents informed of medical problems; she shouldn't tell parents about an adolescent's past procedures. Many states have laws that emancipate minors for health care visits involving pregnancy, abortion, or sexually transmitted diseases

A nurse in a psychiatric care unit finds that a client with psychosis has become violent and has struck another client in the unit. What action should the nurse take in this case? a) Inform the physician and complete a comprehensive assessment. b) Restrain the client, as he is harmful to the other clients. c) Do not restrain the client, as it is equivalent to battery. d) Do not restrain the client, as it is equivalent to false imprisonment.

Restrain the client, as he is harmful to the other clients. Correct Explanation: The nurse should restrain the client because he is potentially harmful to other clients in the psychiatric care unit. Restraints should be used as a last resort and their use should be justified. Unnecessary restraining can lead to allegations of false imprisonment and battery; both are not applicable in this case, however. The nurse should inform the physician about the client, but sometimes it may not be logical to wait for orders to restrain a violent client.

When individuals or groups use utilitarianism to make ethical decisions, which of the following do they consider? a) Rightness or wrongness depends on consequences. b) Duty is equally important. c) Actions are independently right or wrong. d) Consequences are not the only issue.

Rightness or wrongness depends on consequences. Correct Explanation: Utilitarianism states that the rightness or wrongness of an action depends on the consequences of the action. Another theory of ethics, deontology, argues that an action is right or wrong independent of its consequences

The nurse is providing discharge teaching for a client with rheumatic endocarditis but no valvular dysfunction. On which nursing diagnosis should the nurse focus her teaching? a) Impaired gas exchange b) Impaired memory c) Chronic pain d) Risk for infection

Risk for infection Explanation: Clients with endocarditis have a Risk for infection. The nurse should stress to the client that he'll need to continue antibiotics for a minimum of 5 years and that he'll need to take prophylactic antibiotics before invasive procedures for life. There is no indication that the client has Chronic pain or Impaired memory. Because the client doesn't have valvular damage, Impaired gas exchange doesn't apply.

Which of the following categories of laxatives draw water into the intestines by osmosis? a) Stimulants (Dulcolax) b) Saline agents (milk of magnesia) c) Bulk-forming agents (Metamucil) d) Fecal softeners (Colace)

Saline agents (milk of magnesia) Explanation: Bulk-forming agents mix with intestinal fluids, swell, and stimulate peristalsis. Saline agents use osmosis to stimulate peristalsis and act within 2 hours of consumption. Stimulants irritate the colon epithelium. Fecal softeners hydrate the stool by surfactant action on the colonic epithelium, resulting in mixing of aqueous and fatty substances.

The nurse is meeting with a community group to discuss the changes that need to be made to meet their health needs after a community assessment has been done. One cultural group is insisting their views need to be implemented because they are in the majority in that community. What is the best action by the nurse? a) Make decisions based on findings from the community assessment. b) Seek to promote homogeneity and common views rather than focus on differences. c) Support the implementation of the ideas of the majority. d) Seek input from all groups and strive for consensus on what would benefit most or all of these people.

Seek input from all groups and strive for consensus on what would benefit most or all of these people. Explanation: The responsibility is to conduct the community assessment and to identify the key needs. All members need to have representation in this process. It is best to strive for consensus on what the key issues are and to implement programs that would benefit most of the people, rather than responding to one interest group. Listening to the majority viewpoint or helping everyone to change their views and have homogeneity would not be effective. Decisions based on the community alone are also not an appropriate answer.

A client who has been working with an organization for several years did not get a promotion. As a result, the client has gone into depression. What suggestion should the nurse make in order to help the client with his stress? a) Seek professional help b) Take a break from the job c) Change the job d) Accept the changes

Seek professional help Correct Explanation: The client should seek professional help, where he can talk freely about his anger and sense of betrayal. The client should then explore other options in a calmer frame of mind. Changing, compromising, or taking a break from the job will not help the client solve the problem.

A nurse is documenting a client who has had an appendectomy. During a dressing change of the surgical site, the nurse observed a watery pink drainage on the dressing. Which of the following drainage types should the nurse document? a) Sanguineous b) Serous c) Purulent d) Serosanguineous

Serosanguineous Correct Explanation: Serosanguineous drainage is a mixture of serum and red blood cells. It is usually pink in color. Serous drainage is a clear drainage consisting of the serous portion of the blood. Sanguineous drainage consists of red blood cells and looks like blood. Purulent drainage has various colors such as green or yellow; this drainage indicates infection

A client and her partner come to the clinic stating they have been unable to have sexual intercourse. The female client states she has pain and her "vagina is too tight." The client was raped at age 15 years of age. Which nursing problem is most appropriate for this client? a) Dysfunctional Grieving related to loss of self- esteem because of lack of sexual intimacy b) Vaginismus related to vaginal constriction c) Risk for Trauma related to fear of vaginal penetration d) Sexual Dysfunction related to sexual trauma

Sexual Dysfunction related to sexual trauma Correct Explanation: Sexual dysfunction is the problem that is the most appropriate. Dysfunctional grieving because of lack of intimacy is not correct as the couple may have emotional intimacy. The trauma occurred when the female client was 15 years of age thus is not an acute problem. Vaginismus is a medical diagnosis

A nurse is caring for an older adult who has cancer and is experiencing complications requiring a revision of the plan of care. The nurse sits down with the client and the family and discusses their preferences while sharing her judgments based on her expertise. Which of the following types of health care decision making does this represent? a) Shared decision making b) Paternalistic model c) Ethical decision making d) Patient sovereignty model

Shared decision making Explanation: Shared decision making is recommended by most Ethicists and involves the client?s preferences and the nurse?s expertise to make the best decision. The Paternalistic model involves the clinician making the decisions and the Patient Sovereignty model involves the patient making all the decisions without input from the clinician.

What should the nurse tell a female client who is about to begin chemotherapy and anxious about losing her hair? a) She should consider getting a wig or cap before she loses her hair. b) Alopecia related to chemotherapy is relatively uncommon. c) Her hair will grow back the same as it was before treatment. d) Her hair will grow back within 2 months post therapy.

She should consider getting a wig or cap before she loses her hair. Correct Explanation: If hair loss is anticipated, purchase a wig, cap, or scarf before therapy begins. Alopecia develops because chemotherapy affects rapidly growing cells of the hair follicles. Hair usually begins to grow again within 4 to 6 months after therapy. Clients should know that new growth may have a slightly different color and textures

A nurse has custody of a client's daily Kardex and care plan so she can give a change-of-shift report. After reporting to the next shift, what steps should the nurse implement to maintain client confidentiality? a) Throw the documents in the trash can. b) Leave the documents at the nurses' station. c) Shred the documents or place them in a container to protect confidentiality. d) Place the documents in the client's chart.

Shred the documents or place them in a container to protect confidentiality. Explanation: Kardexes, care plans, and other client documents contain confidential client information. The nurse should shred them or place them in a special confidential container for proper disposal. Regular garbage isn't secure and isn't an appropriate place to dispose of documents containing a client's name and information. Leaving the documents at the nurses' station may allow others to view them. It isn't necessary to place the nursing Kardex and care plan in the client's chart when the nurse has finished using them

A client with diabetes mellitus is receiving an oral antidiabetic agent. Which of the following aspects should the nurse observe when caring for this client? a) Blurred vision b) Polyuria c) Polydipsia d) Signs of hypoglycemia

Signs of hypoglycemia Correct Explanation: The nurse should observe the client receiving an oral antidiabetic agent for the signs of hypoglycemia. The time when the reaction might occur is not predictable and could be from 30 to 60 minutes to several hours after the drug is ingested. Observe the client receiving an oral antidiabetic agent for signs of hypoglycemia.

A nurse who comments to her coworkers at lunch that her patient with a sexually transmitted disease has been sexually active in the community may be guilty of what tort? a) Fraud b) Slander c) Assault d) Libel

Slander Correct Explanation: Defamation of character is an intentional tort in which one party makes derogatory remarks about another that diminish the other party's reputation. Slander is spoken defamation of character; libel is written defamation. Assault is a threat or an attempt to make bodily contact with another person without that person's consent. Fraud is willful and purposeful misrepresentation that could cause, or has caused, loss or harm to a person or property

A community nurse is working to decrease the incidence of cardiac disease in Nicaragua. The nurse should plan educational health promotion activities around which of the following? a) Birth control b) Vaccinations c) Smoking cessation d) Breast feeding

Smoking cessation Correct Explanation: Chronic conditions are increasing rapidly in lower-income countries due to adoption of unhealthy lifestyles. Modifiable health behaviors responsible for the high incidence of chronic disease such as cardiac disease include lack of physical activity, poor nutrition, tobacco use, and excessive alcohol consumption. By focusing on smoking cessation programs, the nurse is appropriately addressing a modified risk factor of a chronic condition

What would be an example of the nurse practicing fidelity? The nurse: a) Withholds information as requested b) Stays with the patient during his or her death as promised c) Provides continuity of care d) Regulates visitors

Stays with the patient during his or her death as promised Explanation: Fidelity requires the nurse to keep promises made and to be faithful to one's commitments.

A client is scheduled to receive a blood transfusion. In addition to taking vital signs and verifying that the unit of blood cells is checked, what other assessments/actions would the nurse be responsible for? Select all that apply. a) Assess the client for chills or low back pain. b) Stop the transfusion for reports of dyspnea or itching. c) Rapidly transfuse the blood for the first 15 minutes. d) Reduce intake of fluids during the transfusion. e) Transfuse the blood over 5 hours.

Stop the transfusion for reports of dyspnea or itching. • Assess the client for chills or low back pain. Correct Explanation: Checking for the possibility of transfusion reactions is an important responsibility. Chills can be associated with blood contamination, low back pain can be associated with incompatible blood, and dyspnea and skin itching can be associated with an allergic reaction. The transfusion would need to be stopped. Rapidly transfusing blood is incorrect because the transfusion is started slowly for the first 15 minutes to detect abnormal reactions. Transfusing blood over a prolonged period, e.g., 5 hours, increases the risk of blood contamination.

The general adaptation syndrome (GAS) is a nonspecific physiologic response to a stressor. Select the stage listed below which is not a part of the process. a) Resistance stage b) Alarm stage c) Exhaustion stage d) Stress awareness stage

Stress awareness stage Correct Explanation: The general adaptation syndrome can cycle many times through the alarm and resistance stages before reaching the exhaustion stage. The process occurs through the neuroendocrine and autonomic nervous systems

Which of the following conditions occurs when there is bleeding between the dura mater and arachnoid membrane? a) Epidural hematoma b) Extradural hematoma c) Intracerebral hemorrhage d) Subdural hematoma

Subdural hematoma Explanation: A subdural hematoma is bleeding between the dura mater and arachnoid membrane. Intracerebral hemorrhage is bleeding in the brain or the cerebral tissue with displacement of surrounding structures. An epidural hematoma is bleeding between the inner skull and the dura, compressing the brain underneath. An extradural hematoma is another name for an epidural hematoma.

A client is scheduled for electroconvulsive therapy (ECT). Before ECT begins, the nurse expects to administer which neuromuscular blocking agent? a) Pancuronium b) Succinylcholine c) Atracurium d) Vecuronium

Succinylcholine Correct Explanation: Succinylcholine, a depolarizing blocking agent, is the drug of choice when short-term muscle relaxation is desired — for example, during ECT or intubation. Vecuronium, pancuronium, and atracurium are nondepolarizing blocking agents used for intermediate- or long-term muscle relaxation

The nurse planning to insert an indwelling urinary catheter into a client should utilize which of the following techniques? a) Universal precautions b) Surgical asepsis c) Medical asepsis d) Contact precautions

Surgical asepsis Correct Explanation: Surgical asepsis, also known as sterile technique, is utilized to keep objects and areas free from microorganisms when performing surgery and procedures such as inserting an indwelling urinary catheter or IV catheter. Medical asepsis reduces the number and transfer of pathogens. Universal precautions and contact precautions help to decrease the risk of transmitting infection.

A client who is mentally incapacitated is scheduled to undergo surgery. The nurse demonstrates understanding of the principle of autonomy and checks the client's medical record to ensure that consent has been obtained from which person? a) Surrogate decision maker b) Operating surgeon c) Client d) Attending nurse

Surrogate decision maker Explanation: A surrogate decision maker should be identified to give consent for the client who is mentally incapacitated. Infants, young children, people who are severely mentally handicapped or incapacitated, and people in a persistent vegetative state or coma do not have the capacity to participate in decision making about their healthcare. For such people, a surrogate decision maker must be identified to act on their behalf. The surgeon and the nurse are not eligible to give consent for the client

Which of the following behaviors represent effective coping mechanisms? (Select all that apply.) a) Taking a vacation b) Setting limits with family members who upset you c) Sleeping 14 hours a night d) Learning relaxation techniques e) Denying responsibility for a DUI conviction f) Sleeping 3 hours a night

Taking a vacation • Setting limits with family members who upset you • Learning relaxation techniques Correct Explanation: Coping mechanisms can have positive or negative effects on a client's well-being. All of these examples represent coping, either effective or ineffective.

The nurse caring for a HIV patient diagnosed with acute pneumonia demonstrates understanding of the nurse's role in the current focus on management of chronic illness and disability in which of the following situations? a) Administering prescribed antibiotics b) Reviewing the patient's CD4 count c) Making a referral to an HIV support group d) Teaching the patient to avoid crowds

Teaching the patient to avoid crowds Explanation: Current focus on chronic disease conditions is focused on disease prevention. Teaching the patient to avoid crowds encourages the patient to take control of their health and reduce the risk of pneumonia exacerbations. Administering prescribed antibiotics is indicated in this situation; however, it does not promote independence in the patient. Making a referral to a HIV support group is indicated in this situation; however, the focus is on actions of the nurse not the patient. Reviewing the patient's CD4 count is important but does not indicate the patient's ability to control his or her health

A home care nurse is planning to visit a 60-year-old client diagnosed with heart failure for the first time. Which of the following would be most appropriate for the nurse to do? a) Telephone the client to obtain permission to visit. b) Contact the client to say that the nurse is coming out to visit. c) Obtain information about the client's health insurance. d) Ask the client if he lives alone or with someone else.

Telephone the client to obtain permission to visit. Explanation: When planning an initial visit, it is imperative that the nurse contact the client and obtain permission to visit first. The home care nurse is a guest in the client's home and must have permission to visit and give care. During this contact, the nurse would also schedule a time for the visit and verify the home address. Additionally, the nurse could also check with the client about his living situation in case special arrangements need to be made to enter the home. Obtaining information about health insurance would be done during the initial visit.

A nurse is caring for a female client with hypothyroidism. The client is extremely upset about her altered physical appearance. She doesn't want to take her medication because she doesn't believe it's doing any good. What should the nurse do? a) Tell the client that she looks fine and offer to help her with makeup. b) Tell the client she'll soon experience improvement in her looks as the medication corrects her hormone deficiency. c) Tell the client to ask her physician if she is taking the correct dosage of her medication. d) Tell the client she needs to learn to accept herself as she is and be compliant during treatment.

Tell the client she'll soon experience improvement in her looks as the medication corrects her hormone deficiency. Correct Explanation: Telling the client that she'll soon experience improvement is supportive and encouraging and offers direction in a way that motivates her to take her medication consistently. Telling the client to ask her physician about the medication dosage might cause her to alter her dosage on her own, and also is putting the client off instead of addressing her concerns. Telling the client that she looks fine discounts the feelings she's currently experiencing. Advising the client to accept herself is parental and direct at a time when the client needs support and understanding

A client with bipolar disorder, mania, has flight of ideas and grandiosity and becomes easily agitated. To prevent harmful behaviors, which of the following should the nurse do initially? a) Instruct the client to ask for medication when agitated. b) Seclude the client at the first sign of agitation. c) Tell the client to seek out staff when feeling agitated. d) Encourage the client to stay in his room.

Tell the client to seek out staff when feeling agitated. Correct Explanation: Initially, the nurse would tell the client to seek out staff when feeling agitated or upset to prevent violent episodes. Doing so helps the client to redirect negative feelings in an appropriate manner, such as talking. Encouraging the client to stay in his room is inappropriate because it does not help the client to deal with his feelings. Secluding the client at the first sign of agitation is not indicated and may be perceived by the client as punishment. Instructing the client to ask for medication when agitated would not be the initial course of action. The nurse would interact with the client and direct the client to an activity to decrease his anxiety before intervening with any required medication.

Which of the following results in decreased gas exchange in older adults? a) The alveolar walls contain fewer capillaries. b) The number of alveoli decreases with age. c) The elasticity of the lungs increases with age. d) The alveolar walls become thicker.

The alveolar walls contain fewer capillaries. Explanation: Although the number of alveoli remains stable with age, the alveolar walls become thinner and contain fewer capillaries, resulting in decreased gas exchange. The lungs also lose elasticity and become stiffer. Lungs elasticity does not increase with age, and number of alveoli does not decrease with age.

A client diagnosed with major depression and substance dependence is being admitted to the concurrent disorder treatment unit. In explaining the focus of this program, the nurse should tell what information to the client? a) The depression with be treated first, then the addiction. b) There will be simultaneous treatment of the addiction and depression. c) The addiction will be treated first, then the depression. d) As the addiction is treated, the depression will clear up on its own.

The best approach is to treat both illnesses simultaneously. Treating one and not the other is ineffective. The depression will not clear just by becoming sober or clean.

The client is ambulating in the room and walks around a bedside table. What is the best explanation for why the client does not bump into the table? a) The brain is sending impulses to the muscles to avoid the table. b) The client is aware of spatial relationships to avoid the table. c) The cerebellum is responding to impulses from the inner ear. d) The client?s muscles are being stretched to walk around the table.

The client is aware of spatial relationships to avoid the table. Correct Explanation: The client has awareness of spatial relationships (where objects are located in space). This ability comes from the visual or optic reflexes. The labyrinthine sense relates to the sensory organs in the inner ear and provides a sense of position, orientation, and movement. It does not contribute to where objects are in space. When the extensor muscles are stretched beyond a certain point, their stimulation causes a reflex contraction that aids a person to reestablish erect posture, such as when the knee buckles under, the reflex contraction aids the person to straighten the knee. This does not contribute to perception of where objects are in space

Which of the following is a true statement regarding regional enteritis (Crohn's disease)? a) The lesions are in continuous contact with one another. b) It is characterized by lower left quadrant abdominal pain. c) The clusters of ulcers take on a cobblestone appearance. d) It has a progressive disease pattern.

The clusters of ulcers take on a cobblestone appearance. Correct Explanation: The clusters of ulcers take on a cobblestone appearance. It is characterized by remissions and exacerbations. The pain is located in the lower right quadrant. The lesions are not in continuous contact with one another and are separated by normal tissue

The children of a 78-year-old female client with a recent diagnosis of early-stage Alzheimer's disease are attempting to convince their mother to move into an assisted living facility, a move to which the client is vehemently opposed. Both the client and her children have expressed to the nurse how they are entrenched in their position. Which of the following statements expresses a utilitarian approach to this dilemma? a) Benefits and burdens should be evenly distributed between the children and the client. b) The client has a right to self-determination that is the ultimate priority. c) The client's autonomy and independence are the priority considerations. d) The decision should be made in light of consequences.

The decision should be made in light of consequences. Explanation: Utilitarianism is the theory of ethics that weighs rightness and wrongness according to consequences and outcomes for all those who are affected. Utilitarianism prioritizes these consequences and outcomes over principles such as autonomy and justice, principles that underlie the other statements addressing the client's right to self-determination and fair distribution of benefits and burdens

Nursing students in an ethics class have been asked to define "ethics". What would be the best definition of ethics? a) The informal, systematic study of moral beliefs. b) The formal, systematic study of moral beliefs. c) The adherence to formal personal values. d) The adherence to informal personal values.

The formal, systematic study of moral beliefs. Correct Explanation: Ethics is a formal inquiry into principles of right and wrong conduct, of virtue and vice, and of good and evil as they relate to human conduct and human flourishing. "Morals" usually refers to personal or communal standards of right and wrong.

Nursing students in an ethics class have been asked to define "ethics". What would be the best definition of ethics? a) The formal, systematic study of moral beliefs. b) The adherence to formal personal values. c) The adherence to informal personal values. d) The informal, systematic study of moral beliefs.

The formal, systematic study of moral beliefs. Explanation: Ethics is a formal inquiry into principles of right and wrong conduct, of virtue and vice, and of good and evil as they relate to human conduct and human flourishing. "Morals" usually refers to personal or communal standards of right and wrong.

An infant who has been in foster care since birth requires a blood transfusion. Who is authorized to give written, informed consent for the procedure? a) The social worker who placed the infant in the foster home b) The registered nurse caring for the infant c) The foster mother d) The nurse manager

The foster mother Explanation: When children are minors and aren't emancipated, their parents or designated legal guardians are responsible for providing consent for medical procedures. Therefore, the foster mother is authorized to give consent for the blood transfusion. The social worker, the nurse, and the nurse manager have no legal rights to give consent in this scenario

A group of nurses who work at a large, long-term care facility have become embroiled in controversy over a large number of residents who are refusing a seasonal influenza vaccination. Specifically, there is controversy around the appropriate amount of influence that nurses can exercise when encouraging residents to become immunized. A teleological perspective on this issue would prioritize what consideration? a) Historical precedents. b) The legal rights of the individual. c) The greatest good for the greatest number. d) The "rightness" or "wrongness" of coercion.

The greatest good for the greatest number. Explanation: Teleology is ethical theory based on final outcomes. It is also known as utilitarianism because the ultimate ethical test for any decision is based on what is best for the most people. Deontology focuses on the morality of an act. Teleology does not prioritize historical precedent or the legal rights of the individual

A nurse caring for a client with a respiratory condition notices the client's breathing pattern is getting more irregular and the rate has greatly increased from 18 to 32 breaths per minute. The nurse notes that this client's vital signs are assessed once every shift, but believes the assessment should be done more frequently. Who is responsible for increasing the frequency of this client's assessments? a) The case manager b) The nurse c) The physician d) The nursing supervisor

The nurse Correct Explanation: The question focuses on independent actions that nurses can perform. Interventions for which the nurse may be legally responsible include increasing the frequency of assessments and initiating necessary changes in the treatment regimen. Nurses are responsible for alerting the appropriate professional whenever assessment data differ significantly from the baseline.

The health care facility is involved in litigation by four clients. When reviewing the cases, which legal case would the nurse attorney identify to best describe malpractice? a) The nurse using proper mechanics assists a client to a locked bed. He slips and breaks his left femur. b) The nurse administers amoxicillin (Amicar) to a client with known allergies to penicillin. The client has a seizure with resulting respiratory arrest. c) The nurse administered the wrong medication to the client, who had one episode of vomiting 5 minutes after consuming the medication with no further adverse reactions. d) The nurse applies an ice pack to a client?s lower back without an order and he feels better.

The nurse administers amoxicillin (Amicar) to a client with known allergies to penicillin. The client has a seizure with resulting respiratory arrest. Correct Explanation: All elements of liability are in place for administering amoxicillin to a client with documented allergies to penicillin: the nurse had a duty, but breached it when giving the medication. There also was causation (amoxicillin) and harm (seizures and respiratory arrest). The nurse is negligent when applying an ice pack without an order. The nurse used proper mechanics, so the client fall is an accident even though harm occurred. Giving the wrong medication could be cause for malpractice, but there was no harm.

A nurse follows the universal patient compact principles for partnership when providing care for patients. Which nursing action does not reflect this philosophy? a) The nurse includes the patient as a member of the health care team. b) The nurse allows the patient to review his own medical information. c) The nurse makes health care decisions for a patient who is uncooperative. d) The nurse asks for family input from the assigned advocate of the patient.

The nurse makes health care decisions for a patient who is uncooperative. Correct Explanation: The National Patient Safety Foundation's Principles for Partnership represent a concerted effort to demonstrate a health care organization's commitment to respect the rights of patients and incorporate these beliefs into their mission. The nurse making decisions for an uncooperative patient does not demonstrate these principles. Including the patient and family as a member of the health care team, and allowing the patient to review his own medical information demonstrates these principles

A nurse is providing homecare to a client with a diabetic foot ulcer who needs daily insulin injections. The family caregivers do not possess the technical skill to inject insulin. Which of the following should the nurse keep in mind? a) Nurses should avoid asking the family caregivers to conduct the skilled task b) The current reimbursement system recognizes the family's nontechnical value priorities c) Family caregivers are always perceived to be supportive of good care d) The nurse needs to be creative in integrating the technical and relational aspects of care

The nurse needs to be creative in integrating the technical and relational aspects of care Explanation: The nurse needs to be creative in integrating the technical and relational aspects of care. The current reimbursement system does not recognize the family's nontechnical value priorities. Nurses are expected to educate the family caregivers to conduct the skilled task where possible. In this case, the nurse can teach the family caregivers to inject insulin. Family caregivers can be perceived to be nonsupportive of good care if the families do not follow through

Which example may illustrate a breach of confidentiality and security of patient information? a) The nurse informs a colleague that she should not be discussing patient information in the hospital cafeteria. b) The nurse accesses patient information on the computer at the nurse's station then logs off before answering a patient's call bell. c) The nurse provides information to a professional caregiver involved in the care of the patient. d) The nurse provides information over the phone to the patient's family member who lives in a neighboring state.

The nurse provides information over the phone to the patient's family member who lives in a neighboring state. Explanation: Providing information over the phone to a family member without knowing whether or not the patient wants the family member to know the information is a breach of confidentiality and security of patient information. Providing information to a caregiver involved in the care of a patient is not a breach in confidentiality, while providing information to a professional not involved in the care of the patient is a breach in confidentiality. Patient information should not be discussed in public areas, such as elevators or the cafeteria. Logging off a computer that displays patient data is an appropriate method of protecting patient confidentiality and information.

The nurse is planning to give a new medication to a client. When administering the medication, what is the nurse's most appropriate step to ensure client safety? a) The nurse should know the client?s ability to pay for this medication. b) The nurse should know if the client?s family has taken this medication. c) The nurse should know the alternatives for this medication. d) The nurse should know what side effects are possible from this medication.

The nurse should know what side effects are possible from this medication. Correct Explanation: When implementing any interventions, the nurse should always be aware of any potential adverse reactions. In this case, the nurse should know the possible side effects of the medication. It is not essential for the client's safety for the nurse to know alternatives. The client's ability to pay for the medication or the client's family's experience with the medication is not relevant to the client's safety

A group of nurse researchers has proposed a study to examine the efficacy of a new wound care product. Which of the following aspects of the methodology demonstrates that the nurses are attempting to maintain the ethical principle of nonmaleficence? a) The nurses have completed a literature review that suggests the new treatment may result in decreased wound healing time. b) The nurses have organized the study in such a way that the foreseeable risks and benefits are distributed as fairly as possible. c) The nurses have given multiple opportunities for potential participants to ask questions and have been following the informed consent process systematically. d) The nurses are taking every reasonable measure to ensure that no participants experience impaired wound healing as a result of the study intervention.

The nurses are taking every reasonable measure to ensure that no participants experience impaired wound healing as a result of the study intervention. Correct Explanation: The principle of nonmaleficence dictates that nurses avoid causing harm. In this study, this may appear in the form of taking measures to ensure that the intervention will not cause more harm than good. The principle of justice addresses the distribution of risks and benefits and the informed consent process demonstrates that autonomy is being protected. Preliminary indications of the therapeutic value of the intervention show a respect for the principle of beneficence.

A patient with gastroesophageal reflux disease (GERD) has a diagnosis of Barrett's esophagus with minor cell changes. Which of the following principles should be integrated into the patient's subsequent care? a) Small amounts of blood are likely to be present in the stools and are not cause for concern. b) Antacids may be discontinued when symptoms of heartburn subside. c) The patient will require an upper endoscopy every 6 months to detect malignant changes. d) Liver enzymes must be checked regularly, as H2 receptor antagonists may cause hepatic damage.

The patient will require an upper endoscopy every 6 months to detect malignant changes. Correct Explanation: In the patient with Barrett's esophagus, the cells lining the lower esophagus have undergone change and are no longer squamous cells. The altered cells are considered precancerous and are a precursor to esophageal cancer. In order to facilitate early detection of malignant cells, an upper endoscopy is recommended every 6 months. H2 receptor antagonists are commonly prescribed for patients with GERD; however, monitoring of liver enzymes is not routine. Stools that contain evidence of frank bleeding or that are tarry are not expected and should be reported immediately. When antacids are prescribed for patients with GERD, they should be taken as ordered whether or not the patient is symptomatic

The nurse is caring for a client who just learned of his terminal diagnosis. After the physician leaves, the nurse remains to answer further questions so that the client can make an informed decision about further treatment. By providing all available information, the nurse is promoting which ethical principle? a) The principle of justice b) The principle of autonomy c) The principle of fidelity d) The principle of nonmaleficence

The principle of autonomy Correct Explanation: By promoting open discussion and informed decision making, the nurse is empowering the client to make his own decisions leading to autonomy. The principle of justice requires fairness and justice to all clients. The principle of nonmaleficence requires that nurse does not intentionally or unintentionally inflict harm on others. The principle of fidelity maintains that nurses are faithful to the care of the clients

Which of the following is a characteristic of the care-based approach to bioethics? a) The need for an orientation toward service b) The need to emphasize the relevance of clinical experience c) The promotion of the dignity and respect of patients as people d) The rightness or wrongness of an action is independent of its consequences

The promotion of the dignity and respect of patients as people Correct Explanation: The care-based approach to bioethics focuses on the specific situations of individual patients and characteristics of this approach include promoting the dignity and respect of patients and people. The need to emphasize the relevance of clinical experience and the need for an orientation toward service are part of criticisms of bioethics. The deontologic theory of ethics says that an action is right or wrong independent of its consequences

A nurse is teaching a female client about preventing osteoporosis. Which teaching point is correct? a) The recommended daily allowance of calcium may be found in a wide variety of foods. b) To prevent fractures, the client should avoid strenuous exercise. c) Obtaining an X-ray of the bones every 3 years is recommended to detect bone loss. d) Obtaining the recommended daily allowance of calcium requires taking a calcium supplement.

The recommended daily allowance of calcium may be found in a wide variety of foods. Explanation: Premenopausal women require 1,000 mg of calcium per day. Postmenopausal women require 1,500 mg per day. Clients usually can get the recommended daily requirement of calcium by eating a varied diet. Osteoporosis doesn't show up on ordinary X-rays until 30% of bone has been lost. Bone densitometry, however, can detect bone loss of 3% or less. This test is sometimes recommended routinely for women older than 35 who are at risk for osteoporosis. Strenuous exercise won't cause fractures. Although supplements are available, they aren't always necessary

A nurse is preparing to administer two types of insulin to a client with diabetes mellitus. Which of the following demonstrates that the nurse understands the correct procedure for preparing this medication? a) The intermediate-acting insulin is withdrawn before the short-acting insulin. b) If administered immediately, there is no requirement for withdrawing one type of insulin before another. c) The short-acting insulin is withdrawn before the intermediate-acting insulin. d) Different types of insulin are not to be mixed in the same syringe.

The short-acting insulin is withdrawn before the intermediate-acting insulin. Correct Explanation: When combining two types of insulin in the same syringe, the short-acting regular insulin is withdrawn into the syringe first and the intermediate-acting insulin is added next, a practice referred to as "clear to cloudy."

Of the following, which best explains the importance of theoretic frameworks? a) Theoretic frameworks advance nursing knowledge and practice b) Theoretic frameworks guide psychosocial nursing care c) Theoretic frameworks guide physiologic nursing care d) Theoretic frameworks advance the ethical aspects of practice

Theoretic frameworks advance nursing knowledge and practice Explanation: Theoretic frameworks are important to the advancement of nursing knowledge and professional practice.

Nursing students are reviewing information about infectious diseases and events associated with infection. Students demonstrate understanding of the information when they identify the incubation period as which of the following? a) State in which the host displays a decrease in wellness b) Presence of microorganisms without the host interacting with them c) Time between exposure and onset of symptoms d) Process of the host shedding the microorganisms to another

Time between exposure and onset of symptoms Correct Explanation: The incubation period is time between contact or exposure and the development of the first signs and symptoms. The presence of microorganisms without the host interacting with them is called colonization. The state in which the host displays a decrease in wellness characterizes an infectious disease. The process of the host shedding the microorganisms to another reflects the mode of exit.

While in the grocery store, a nurse is confronted by a neighbor who asks how the neighbor's hospitalized aunt is doing. The nurse decides that because she knows the neighbor well and the neighbor is related to the client, it is acceptbale to tell the neighbor about the client's progress. This scenario can be seen as a) Unethical, but not illegal b) Unethical and illegal c) Appropriate because the discussion occurred outside the work place d) Appropriate because the neighbor is a relative

Unethical and illegal Correct Explanation: Disclosing information about the client to others without the client's consent is a HIPAA violation and, therefore is not only unethical, but also illegal.

A nurse in a physician's office has noted on several occasions that one of the physicians frequently obtains controlled drug prescription forms for prescription writing. The physician reports that his wife has chronic back pain and requires pain medication. One day the nurse enters the physician's office and sees him take a pill out of a bottle, and he mentions he suffers from migraines and it really helps when he takes his wife's pain medication. What type of nurse-physician ethical situation is illustrated in this scenario? a) Unprofessional, incompetent, unethical, or illegal physician practice b) Disagreements about the proposed medical regimen c) Conflicts regarding the scope of the nurse's role d) Claims of loyalty

Unprofessional, incompetent, unethical, or illegal physician practice Correct Explanation: The physician is demonstrating unprofessional, incompetent, unethical, or illegal physician practice

A nurse is teaching a client about maintaining a healthy heart. The nurse should include which point in her teaching? a) Exercise one or two times per week. b) Smoke in moderation. c) Consume a diet high in saturated fats and low in cholesterol. d) Use alcohol in moderation.

Use alcohol in moderation. Correct Explanation: The nurse should advise the client that alcohol may be used in moderation as long as there are no other contraindications for its use. Smoking, a diet high in cholesterol and saturated fat, and a sedentary lifestyle are all known risk factors for cardiac disease. The client should be encouraged to quit smoking, exercise three to four times per week, and consume a diet low in cholesterol and saturated fat.

A patient in the clinic is diagnosed with diarrhea caused by Campylobacter. Which of the following instructions should the nurse provide to prevent further episodes? a) Drink water only from purified or filtered sources. b) Use proper storage and cooking of meat. c) Wash hands after going to the bathroom. d) Do not ingest raw eggs.

Use proper storage and cooking of meat. Explanation: Campylobacter infection is caused by the consumption of undercooked or raw meat. Proper storage and cooking of meat will prevent further episodes of Campylobacter. The patient should also be told to prepare meat separately from other foods, including the use of utensils. Giardia lamblia diarrhea is caused by drinking contaminated water. Shigella infection is transmitted via the fecal-oral route, so handwashing after going to the bathroom would help prevent the illness. Salmonella infections are usually caused by consuming raw eggs; it also can be transmitted via produce

Which ethics theory focuses on ends or consequences of actions? a) Adaptation theory b) Deontological theory c) Formalist theory d) Utilitarian theory

Utilitarian theory Explanation: Utilitarian theory is based on the concept of the greatest good for the greatest number. Formalist theory argues that moral standards exist independently of the ends or consequences. Deontological theory argues that moral standards exist independently of the ends or consequences. Adaptation theory is not an ethics theory.

One classic theory in ethics is teleologic theory or consequentialism, which focuses on the ends or consequences of actions. The nurse knows that the best known form of this theory is a) Utilitarianism b) Double effect c) Beneficence d) Formalist theory

Utilitarianism Correct Explanation: One classic theory in ethics is teleologic theory or consequentialism, which focuses on the ends or consequences of actions. The best-known form of this theory, utilitarianism, is based on the concept of "the greatest good for the greatest number." Beneficence and double effect are examples of common ethical principles. The formalist theory is another theory in ethics which argues that ethical standards or principles exist independently of the ends or consequences

What is the term for the beliefs held by the individual about what matters? a) Ethics b) Morals c) Bioethics d) Values

Values Correct Explanation: Values are ideals and beliefs held by an individual or group; values act as a standard to guide one's behavior. Ethics are moral principles and values that guide the behavior of honorable people. A moral is a standard for right and wrong. Bioethics is related to ethical questions surrounding life and death and questions and concerns regarding quality of life as it relates to advanced technology.

What is the term for the beliefs held by the individual about what matters? a) Morals b) Bioethics c) Values d) Ethics

Values Correct Explanation: Values are ideals and beliefs held by an individual or group; values act as a standard to guide one's behavior. Ethics are moral principles and values that guide the behavior of honorable people. A moral is a standard for right and wrong. Bioethics is related to ethical questions surrounding life and death and questions and concerns regarding quality of life as it relates to advanced technology.

The nurse applies which ethical principle when telling the truth to a client about the prognosis? a) Veracity b) Fidelity c) Beneficence d) Nonmaleficence

Veracity Correct Explanation: The ethical principle of veracity is the obligation to tell the truth and not to lie or deceive others. Nonmaleficence is the duty not to inflict harm as well as to prevent and remove harm. Fidelity is promise keeping -- the duty to be faithful to one's commitments. Beneficence is the duty to do good and to actively promote positive acts (eg, goodness, kindness, charity).

An oncology patient in an outpatient chemotherapy clinic asks several questions regarding his care and treatment. The nurse explains the clinic's routine, typical side effects of the chemotherapy, and ways to decrease the number of side effects experienced. What characteristic is the nurse demonstrating? a) Autonomy b) Fidelity c) Veracity d) Justice

Veracity Correct Explanation: Veracity means telling the truth, which is essential to the integrity of the patient-provider relationship.

An oncology patient in an outpatient chemotherapy clinic asks several questions regarding his care and treatment. The nurse explains the clinic's routine, typical side effects of the chemotherapy, and ways to decrease the number of side effects experienced. What characteristic is the nurse demonstrating? a) Justice b) Veracity c) Fidelity d) Autonomy

Veracity Explanation: Veracity means telling the truth, which is essential to the integrity of the patient-provider relationship

A patient is brought to the operating room for an elective surgery. What is the priority action by the circulating nurse? a) Obtain a sponge and syringe count. b) Document start of surgery. c) Verify consent. d) Acquire ordered blood products.

Verify consent. Correct Explanation: Without consent, surgery cannot be performed. Documentation of the start of surgery can only happen once the surgery has started. Blood products must be administered within an allotted time frame and therefore should not be acquired unless needed. The sponge and syringe count is a safety issue that should be completed before surgery and while the wound is being sutured, but the patient has not consented, the surgery should not take place

A 13-year-old is having surgery to repair a fractured left femur. As a part of the preoperative safety checklist, what should the nurse do? a) Ask the teen to point to the surgery site. b) Restate the surgery risks to the parents. c) Ask the parents if they have signed the operative permit. d) Verify that the site, side, and level are marked.

Verify that the site, side, and level are marked. Correct Explanation: As part of a surgery safety checklist, the nurse must verify that the site, side, and level are marked. Pointing to the area is not sufficient identification of the surgery site. The nurse must verify the form has been signed by reviewing the form. The surgeon holds primary responsibility for explaining the risks of surgery.

A client asks the nurse which vitamins should be taken daily for feelings of fatigue, anxiety, and depression 1 week before menses. Which of the following is the correct response by the nurse? a) Vitamin B6 b) Vitamin C c) Vitamin D d) Vitamin A

Vitamin B6 Correct Explanation: The nurse should encourage taking Vitamin B6 daily, as it may be effective at relieving symptoms of irritability, fatigue, and depression related to the premenstrual period.

Which nutrient plays an important role in normal blood clotting? a) Protein b) Vitamin K c) Vitamin C d) Zinc

Vitamin K Correct Explanation: Vitamin K is important for normal blood clotting.

A nurse is assessing a client at a mental health clinic who threatens suicide and describes having a plan. Which of the following should the nurse recognize as the priority goal for the client? a) Obtaining admission to an acute care facility b) Notifying family members of the suicide plan c) Working with the client to resolve the immediate crisis d) Establishing a foundation for long-term therapy

Working with the client to resolve the immediate crisis Explanation: The goal of crisis intervention is the resolution of an immediate problem. The client must learn to solve his/her own problems. Although some clients do enter long-term therapy or are admitted to an acute care facility, these are not the goals of crisis intervention

A nurse pages a client's primary care physician in response to a low blood pressure reading. When returning the nurse's page, the physician asks the nurse to temporarily hold the client's scheduled antihypertensive and diuretic medications. How should the nurse ensure correct documentation of this telephone order? a) Write "T.O." after the order and write out the physician's and nurse's names. b) Record the order verbatim in the client's charts and follow it with the nurse's printed name and signature alone. c) Obtain confirmation of the order from a physician or nurse practitioner present on the unit. d) Write out the order, the physician's name, the nurse's name, and the name of a witness.

Write "T.O." after the order and write out the physician's and nurse's names. Correct Explanation: When receiving telephone orders, the nurse should record the orders in the client's medical record, read the order back to the ordering practitioner, date and note the time the orders were issued, record T.O. (telephone orders) and the full name and title of the physician or nurse practitioner who issued the orders, and then sign the orders with name and title. It is unnecessary to obtain a confirmation from another practitioner or to have the order witnessed.

Which procedures can the nurse working on a pediatric floor safely delegate to the licensed practical/vocational nurse (LPN/VN)? Select all that apply. a) giving an IV push medication b) calling the morning blood sugars to the health care provider (HCP) c) inserting hearing aids d) refilling a baclofen pump e) administering gastrostomy tube feedings

administering gastrostomy tube feedings • inserting hearing aids Explanation: In general, LPN/VNs may perform skills related to feeding, oral medication administration, and activities of daily living, such as insertion of a hearing aid. Refilling a baclofen pump constitutes administering an intrathecal medication and is beyond the scope of practice for LPN/VNs in most areas. Some institutions allow LPN/VNs to give IV push medicines; however, special training is required. Communicating with the health care provider (HCP) would require discussion of the client's assessments and evaluations, which fall under the RN scope of practice.

The nurse is teaching a client about risk factors associated with atherosclerosis and how to reduce the risk. Which risk factor is the client unable to modify? a) diabetes b) exercise level c) dietary preferences d) age

age Correct Explanation: Age is a nonmodifiable risk factor for atherosclerosis. The nurse instructs the client to manage modifiable risk factors such as comorbid diseases (e.g., diabetes), activity level, and diet. Controlling serum blood glucose levels, engaging in regular aerobic activity, and choosing a diet low in saturated fats can reduce the risk of developing atherosclerosis.

The nurse has assisted the health care provider (HCP) at the bedside with insertion of a left subclavian, triple lumen catheter in a client admitted with lung cancer. Suddenly, the client becomes restless and tachypneic. The nurse should: a) assess breath sounds. b) insert a peripheral IV. c) remove the catheter. d) reposition the client.

assess breath sounds. Correct Explanation: The nurse should first assess for bilateral breath sounds since a complication of central line insertion is a pneumothorax, which would cause an increase in respiratory rate and drop in oxygen, causing irritability. The nurse should also assess blood pressure and heart rate for the complication of bleeding. A chest x-ray will be performed to determine correct placement and complications. A central line was most likely placed because peripheral IV access was not available or adequate for the client. Repositioning may be considered after assessments are done.

The nurse is assigning tasks to unlicensed assistive personnel (UAP) for a client with an abdominal hysterectomy on the first postoperative day. Which task cannot be delegated to the UAP? a) giving perineal care b) assessing the incision site c) taking vital signs d) recording intake and output

assessing the incision site Correct Explanation: The registered nurse (RN) is responsible for monitoring the surgical site for condition of the dressing, status of the incision, and signs and symptoms of complications. UAP who have been trained to report abnormalities to the RN supervising the care may take vital signs, record intake and output, and give perineal care

The client has had a myocardial infarction, and the nurse has instructed the client to prevent Valsalva's maneuver. The nurse determines the client is following the instructions when the client: a) avoids holding the breath during activity. b) clenches the teeth while moving in bed. c) assumes a side-lying position. d) drinks fluids through a straw.

avoids holding the breath during activity. Correct Explanation: Valsalva's maneuver, or bearing down against a closed glottis, can best be prevented by instructing the client to exhale during activities such as having a bowel movement or moving around in bed. Valsalva's maneuver is not prevented by having the client assume a side-lying position. Clenching the teeth will likely contribute to Valsalva's maneuver, not inhibit it. Drinking fluids through a straw has no effect on preventing or causing Valsalva's maneuver.

A client who is admitted to the adult unit of a mental health care facility with depression tells the nurse that he has pedophilia. The nurse should: a) recognize that because the client is depressed, the client will not be able to discuss the pedophilia. b) refer the client to group therapy. c) be aware of personal opinions and views. d) ensure that the client is never alone with other clients on the unit.

be aware of personal opinions and views. Correct Explanation: The nurse must be aware of personal opinions and views when caring for clients with psychosexual disorders. The care plan for the client will be developed to manage both the depression and the pedophilia. It is not necessary to restrict the client's interactions with others on this adult mental health unit. The health care provider (HCP) will determine the type of therapy that will be most appropriate for this client.

The client who is receiving chemotherapy is not eating well but otherwise feels healthy. What should the nurse suggest the client eat? a) cereal with milk and strawberries b) broiled chicken, green beans, and cottage cheese c) steak and french fries d) toast, gelatin dessert, and cookies

broiled chicken, green beans, and cottage cheese Correct Explanation: Carbohydrates are the first substance used by the body for energy. Proteins are needed to maintain muscle mass, repair tissue, and maintain osmotic pressure in the vascular system. Fats, in a small amount, are needed for energy production. Chicken, green beans, and cottage cheese are the best selection to provide a nutritionally well-balanced diet of carbohydrate, protein, and a small amount of fat. Cereal with milk and strawberries as well as toast, gelatin dessert, and cookies have a large amount of carbohydrates and not enough protein. Steak and french fries provide some carbohydrates and a good deal of protein; however, they also provide a large amount of fat

A registered nurse who is responsible for coordinating and documenting patient care in the operating room is a a) circulating nurse. b) scrub nurse. c) anesthetist. d) anesthesiologist.

circulating nurse. Explanation: A circulating nurse is a registered nurse who coordinates and documents patient care. The scrub nurse prepares instruments and supplies, and hands instruments to the surgeon during the procedure. The anesthetist is trained to deliver anesthesia and to monitor the patient's condition during surgery. The anesthesiologist is a physician trained to deliver anesthesia and to monitor the patient's condition during surgery.

The client with mania is skipping up and down the hallway, nearly running into other clients. The nurse should include which activity in the client's plan of care? a) leading a group activity b) reading the newspaper c) cleaning the dayroom tables d) watching television

cleaning the dayroom tables Correct Explanation: The client with mania is very active and needs to have this energy channeled in a constructive task such as cleaning or tidying the dayroom. Because the client is distracted easily and can concentrate only for short periods, the successful completion of a helpful task would give the nurse the opportunity to thank the client for the help, thereby enhancing the client's self-esteem. Leading a group activity is too stimulating for the client. Participating in this type of activity also may cause the client to be disruptive. Watching television or reading the newspaper would be inappropriate for the client who cannot sit for a period of time.

Parents of a 4-year-old child with acute leukemia ask a nurse to explain the concept of complementary therapy. The nurse should tell the parents that: a) there's no research that indicates that complementary therapies are effective. b) complementary therapy is an alternative to conventional medical therapies. c) complementary therapy wouldn't help their child. d) the physician should talk with them about it.

complementary therapy is an alternative to conventional medical therapies. Correct Explanation: The nurse should tell the parents that complementary therapy is a form of alternative medicine. This type of therapy can include diet, exercise, herbal remedies, and prayer. Answering the parents' questions builds rapport and trust. The nurse shouldn't dismiss the parents' idea by telling them complementary therapy wouldn't help their child. The nurse doesn't need to direct the parents to the physician. She can provide the basic information and let the parents determine if they'd like to seek further assistance. Studies indicate that complementary therapies are beneficial to the child and the parents

A client is admitted for treatment of Prinzmetal's angina. When developing this client's care plan, the nurse should keep in mind that this type of angina can result from: a) an unpredictable amount of activity. b) coronary artery spasm. c) the same type of activity that caused previous angina episodes. d) activities that increase myocardial oxygen demand.

coronary artery spasm. Explanation: Prinzmetal's angina results from coronary artery spasm. Activities that increase myocardial oxygen demand may trigger angina of effort. An unpredictable amount of activity may precipitate unstable angina. Worsening angina is brought on by the same type or level of activity that caused previous angina episodes; anginal pain becomes increasingly severe

A client with aplastic anemia is instructed to eat foods rich in iron. The nurse should instruct the client to include which food in the diet to increase iron intake? a) chicken breasts b) dark green leafy vegetables c) fresh fruits d) cheese

dark green leafy vegetables Correct Explanation: Foods high in iron include dark green leafy vegetables, liver and red meat, eggs, dried fruit, legumes, and whole grain breads. Fruits, cheese, and chicken are not high in iron.

A client admits to using cocaine and says, "When I stop using, I feel bad." Which effect is the client most likely to describe as occurring after he stops using cocaine? a) palpitations b) flashbacks c) double vision d) depression

depression Correct Explanation: Some feelings of depression may occur after a person stops using cocaine. Palpitations, flashbacks, and double vision are not associated with cocaine withdrawal.

Parents of a 15-year-old state that their child is moody and rude. The nurse should advise the parents to: a) restrict their child's activities. b) talk to other parents of adolescents. c) discuss their feelings with their child. d) obtain family counseling.

discuss their feelings with their child. Correct Explanation: Parents need to discuss with their adolescent how they perceive the behavior and how they feel about it. Moodiness is characteristic of adolescents. The adolescent may have a reason for or not be aware of his behavior. Restricting the adolescent's activities will not change the mood or the response to others. It may increase unacceptable responses. Counseling may not be needed at this time if the parents are open to communicating and listening to the adolescent. Talking to other parents may be of some help, but what is helpful to others may not be helpful to their child

The nurse is teaching a group of teenage boys about the risks of chewing tobacco. The nurse should teach the boys to recognize which possible signs or symptoms of oral cancer? Select all that apply. a) dysphagia b) white patches on the mucosa c) lump in the neck d) sensitive teeth e) unexplained mouth pain

dysphagia • unexplained mouth pain • lump in the neck • white patches on the mucosa Explanation: Chewing tobacco has become a more common practice among teenagers. It is important that they understand that this increases their risk for oral cancer. They should be instructed to inspect their mouth frequently and report any observed lesions or other changes in the oral mucosa. Potential indicators of oral cancer are dysphagia, unexplained mouth pain, a lump in the neck, and white patches on the mucosa (leukoplakia). Other indications may be a painless mouth ulcer, a reddened patch (erythroplasia), and rough patches on the mucosa. Sensitive teeth and decreased saliva are not associated with oral cancer.

The most common symptom of esophageal disease is a) dysphagia. b) vomiting. c) nausea. d) odynophagia.

dysphagia. Correct Explanation: This symptom may vary from an uncomfortable feeling that a bolus of food is caught in the upper esophagus to acute pain on swallowing. Nausea is the most common symptom of gastrointestinal problems in general. Vomiting is a nonspecific symptom that may have a variety of causes. Odynophagia refers specifically to acute pain on swallowing

A client whose condition remains stable after a myocardial infarction gradually increases activity. To determine whether the activity is appropriate for the client the nurse should assess the client for: a) edema. b) dyspnea. c) cyanosis. d) weight loss.

dyspnea. Correct Explanation: Physical activity is gradually increased after a myocardial infarction while the client is still hospitalized and through a period of rehabilitation. The client is progressing too rapidly if activity significantly changes respirations, causing dyspnea, chest pain, a rapid heartbeat, or fatigue. When any of these symptoms appears, the client should reduce activity and progress more slowly. Edema suggests a circulatory problem that must be addressed but does not necessarily indicate overexertion. Cyanosis indicates reduced oxygen-carrying capacity of red blood cells and indicates a severe pathology. It is not appropriate to use cyanosis as an indicator for overexertion. Weight loss indicates several factors but not overexertion

Risk factors for the development of breast cancer include: a) early menopause (before age 40). b) breastfeeding. c) early onset of menstruation. d) having had more than two children.

early onset of menstruation. Explanation: A woman's lifetime exposure to estrogen is implicated in breast cancer development. Therefore, early onset of menstruation, delayed onset of menopause, and childlessness or delayed childbearing all appear to increase a woman's risk of breast cancer. A family history of the disease also appears to increase a woman's risk. Menstruation is delayed until breastfeeding ceases, which limits a woman's exposure to estrogen. Therefore, breastfeeding may reduce, not increase, risk.

A nurse is caring for a client admitted to the hospital for dehydration. The physical findings consistent with the diagnosis include: a) cold intolerance and brittle nails. b) pallor and diaphoresis. c) easy wrinkling of the skin and sunken eyes. d) slow heart rate and prolonged capillary refill.

easy wrinkling of the skin and sunken eyes. Correct Explanation: Most illnesses cause at least some alterations in general physical appearance. Observing for changes in appearance is an important nursing responsibility for detecting illness or evaluating the effectiveness of care and therapy. For example, a person with an insufficient intake of fluids has dry skin that wrinkles easily, eyes that might be sunken and dull in appearance, and poor muscle tone. On the other hand, the person in good health tends to radiate his or her healthy status through general appearance. Although prolonged capillary refill is consistent with dehydration, slow heart rate is not. Pallor may be associated with dehydration but diaphoresis is not associated with this condition. Cold intolerance and brittle nails are consistent physiologic changes seen in clients with hypothyroidism

The nurse is preparing to teach a client with iron deficiency anemia about the diet to follow after discharge. Which food should be included in the diet? a) eggs b) citrus fruits c) cheese d) lettuce

eggs Correct Explanation: For the client with iron deficiency anemia, a rich source of iron is needed in the diet, and eggs are high in iron. Other foods high in iron include organ and muscle (dark) meats; shellfish, shrimp, and tuna; enriched, whole-grain, and fortified cereals and breads; legumes, nuts, dried fruits, and beans; oatmeal; and sweet potatoes. Dark green, leafy vegetables and citrus fruits are good sources of vitamin C. Cheese is a good source of calcium

When the indication for surgery is without delay, the nurse recognizes that the surgery will be classified as a) emergency. b) urgent. c) elective. d) required.

emergency. Explanation: Emergency surgery means that the patient requires immediate attention and the disorder may be life threatening. Urgent surgery means that the patient requires prompt attention within 24 to 30 hours. Required surgery means that the patient needs to have surgery, and it should be planned within a few weeks or months. Elective surgery means that there is an indication for surgery, but failure to have surgery will not be catastrophic.

A client with severe acute respiratory syndrome privately informs a nurse that he does not want to be placed on a ventilator if his condition worsens. The client's wife and children have repeatedly expressed their desire that every measure be taken for the client. The most appropriate intervention by the nurse would be to: a) arrange a conference to discuss the matter with all involved. b) ask the physician to discuss the client's prognosis with the client and and the family. c) encourage the client to consider a living will or power of attorney. d) assure the client that all possible measures will be taken.

encourage the client to consider a living will or power of attorney. Correct Explanation: The nurse is obligated to act as the client's advocate. A living will or power of attorney would clearly define the client's wishes. The nurse should not discuss the issue with the client's family unless the client gives permission. Assuring the family and client that all possible measures will be taken opposes the client's wishes and does not demonstrate client advocacy

A malignant tumor a) demonstrates cells that are well differentiated. b) grows by expansion. c) gains access to the blood and lymphatic channels. d) is usually slow growing.

gains access to the blood and lymphatic channels. Correct Explanation: By gaining access to blood and lymphatic channels, the tumor metastasizes to other areas of the body. Cells of malignant tumors are undifferentiated. Malignant tumors demonstrate variable rate of growth; however, the more anaplastic the tumor, the faster its growth. A malignant tumor grows at the periphery and sends out processes that infiltrate and destroy surrounding tissues.

A nurse may use self-disclosure with a client if: a) the client asks the nurse directly about her experience. b) it helps the client to talk more easily. c) the nurse has experienced the same situation as the client. d) it achieves a specific therapeutic goal.

it achieves a specific therapeutic goal. Correct Explanation: Self-disclosure (making personal statements about oneself) can be a useful nursing tool. However, a nurse should use self-disclosure judiciously and with a specific therapeutic purpose in mind. She should listen closely to the client and remember that the experiences of different people are sometimes similar but never identical. Using too many self-disclosures is unethical and can shift the focus from the client to the nurse. Self-disclosure that distracts the client from treatment issues doesn't benefit the client and may alienate the client from the nurse.

After a bronchoscopy with biopsy, the nurse assesses the client. The nurse should report which finding to the health care provider (HCP)? a) green sputum b) dry cough c) hemoptysis d) laryngeal stridor

laryngeal stridor Correct Explanation: Laryngeal stridor is characteristic of respiratory distress from inflammation and swelling after bronchoscopy. It must be reported immediately. Green sputum indicates infection and would occur 3 to 5 days after bronchoscopy. A mild cough or hemoptysis is typical after bronchoscopy. If a tissue biopsy specimen was obtained, sputum may be blood-streaked for several days

A client has been diagnosed with bacterial pneumonia. After 1 day of IV antibiotic therapy, the client's white blood cell count is still 14,000/mm3 (14 X 109/L). The nurse should: a) administer the next scheduled antibiotic dose early. b) initiate reverse isolation precautions. c) recheck the client's white blood cell count in 24 hours. d) notify the health care provider.

notify the health care provider. Correct Explanation: If the white blood cell count does not begin decreasing, it may indicate that the antibiotic is not effective against the organism causing the pneumonia. The health care provider should be notified as he or she may want to consider changing antibiotics. While rechecking the client's white blood cell count may be appropriate, it is the health care provider's responsibility to make this decision. Reverse isolation is used for clients with a very low white blood cell count. The antibiotic dosing schedule should be strictly maintained.

The nurse is conducting a cancer risk assessment for a middle-aged client. Which environmental factor increases the risk of cancer? a) nutrition b) gender c) immunologic status d) age

nutrition Correct Explanation: Environmental factors include place of residence, nutrition, occupation, personal habits, iatrogenic factors, and physical environment. Gender, immunologic status, and age are individual factors.

The nurse auscultates the lungs of a client who has been diagnosed with a tumor in the lung and notes wheezing over one lung. The nurse should assess the client further for: a) obstruction of the airway. b) the presence of exudate in the airways. c) an indication of pleural effusion. d) the client's history of smoking.

obstruction of the airway. Correct Explanation: Wheezing over one lung in the presence of a lung tumor is most likely caused by obstruction of the airway by a tumor. Exudate would be more likely to cause crackles. The client's history of smoking would not cause unilateral wheezing. Pleural effusion would produce diminished or absent breath sounds.

A 56-year-old male patient is experiencing withdrawal from alcohol and is placing himself at risk for falls by repeatedly attempting to scale his bedrails. Benzodiazepines have failed to alleviate his agitation and the nurse is considering obtaining an order for physical restraints to ensure his safety. The nurse should recognize that this measure may constitute: a) deception. b) advocacy. c) harm. d) paternalism.

paternalism. Correct Explanation: Paternalism involves the violation of a patient's autonomy in order to maximize good or minimize harm, a situation that requires careful consideration in light of ethical principles. Deception is unlikely to occur and the risk for harm is likely decreased by the use of restraints. Advocacy is the protection and support of another's rights.

A client at 4 weeks postpartum tells the nurse that she cannot cope any longer and is overwhelmed by her newborn. The baby has old formula on her clothes and under her neck. The mother does not remember when she last bathed the baby and states she does not want to care for the infant. The nurse should encourage the client and her husband to call their health care provider (HCP) because the mother should be evaluated further for: a) poor bonding. b) postpartum blues. c) infant abuse. d) postpartum depression.

postpartum depression. Correct Explanation: The client is experiencing and verbalizing signs of postpartum depression, which usually appears at about 4 weeks postpartum but can occur at any time within the first year after birth. It is more severe and lasts longer than postpartum blues, also called "baby blues." Baby blues are the mildest form of depression and are seen in the later part of the first week after birth. Symptoms usually disappear shortly. Depression may last several years and is disabling to the woman. Poor bonding may be seen at any time but commonly becomes evident as the mother begins interacting with the infant shortly after birth. Infant abuse may take the form of neglect or injuries to the infant. A depressed mother is at risk for injuring or abusing her infant

The nurse makes a home visit to a primigravida on the fourth postpartum day after birth of a viable neonate. When the nurse enters the house, the nurse finds the client sitting in a chair, crying inconsolably, while the neonate is crying in another room. The client tells the nurse that she has not been sleeping well and has been hearing voices. The nurse determines that the client is most likely experiencing: a) normal reactions to being a new mother. b) postpartum psychosis. c) the "baby blues." d) postpartum depression.

postpartum psychosis. Correct Explanation: The client's symptoms of insomnia, crying inconsolably, and hearing voices (hallucinations) are all symptoms of postpartum psychosis. The client needs immediate treatment to prevent injury to herself and the neonate. Postpartum psychosis occurs in about 1 in 1,000 pregnancies; thus, it is relatively rare but serious. Hospitalization, social support, and psychotherapy are used to treat postpartum psychosis. Prognosis for recovery is good, but the condition may recur with subsequent pregnancies. Although crying at times may be expected, ignoring a crying newborn and hearing voices are not normal reactions. "Baby blues" is a transient condition; mothers experiencing this do not hear voices. Postpartum depression continues for several weeks or months after birth. Crying, sadness, and lack of appetite may be present, but the client does not hear voices.

A nurse-manager for a community health organization is planning for the home health needs of an 8-year-old child who requires around-the-clock care by nursing assistants. The nurse-manager knows that when working with a nursing assistant, she must: a) provide written instructions, education, and ongoing supervision. b) ensure that the work is divided equitably to prevent staff burnout and rapid turnover. c) in the event of limited staff resources, provide health services to those in greatest need. d) ensure that the nursing assistant is paid fairly and for any additional time worked.

provide written instructions, education, and ongoing supervision. Correct Explanation: When working with a nursing assistant, the nurse-manager must provide written instructions, education, and ongoing supervision. Although the nurse-manager should be concerned with the equitable division of work and proper payment for hours worked, these concerns aren't the highest priorities. The provision of health services to those in greatest need is an important overall goal, but isn't specific to working with a nursing assistant.

A 1-month-old infant in the neonatal intensive care unit is dying. His parents request that a nurse give the infant an opioid analgesic. The infant's heart rate is 68 beats/minute and his respiratory rate is 18 breaths/minute. He is on room air; oxygen saturation is 92%. The nurse's response to the parents' request should be based on the fact that: a) withholding the opioid analgesic is clinically appropriate because administering it would hasten the infant's death. b) withholding the opioid analgesic during the last days and hours is an ethical duty; administering it would represent assisted suicide. c) providing an analgesic during the last days and hours is an ethically appropriate nursing action. d) administering an analgesic during the last days and hours is the parents' ethical decision.

providing an analgesic during the last days and hours is an ethically appropriate nursing action. Correct Explanation: The nurse's action should be based on the fact that all clients, regardless of age, have the right to die with dignity and to be free of pain. Assisted suicide requires some action on the part of the client, which isn't possible in the case a 1-month-old infant. The parent's decision doesn't eliminate the nurse's ethical obligation to the infant and to the nursing profession. Withholding the opioid analgesic isn't appropriate because it isn't known that administering the drug would hasten death in this case.

A client uses timolol maleate eyedrops. The expected outcome of this drug is to control glaucoma by: a) reducing aqueous humor formation. b) dilating the canals of Schlemm. c) improving the ability of the ciliary muscle to contract. d) constricting the pupils.

reducing aqueous humor formation. Correct Explanation: Timolol maleate is commonly administered to control glaucoma. The drug's action is not completely understood, but it is believed to reduce aqueous humor formation, thereby reducing intraocular pressure. Timolol does not constrict the pupils; miotics are used for pupillary constriction and contraction of the ciliary muscle. Timolol does not dilate the canal of Schlemm.

A nurse is assessing an 82-year-old for depression. Because of the client's age, the nurses' assessment should be guided by the fact that: a) psychosomatic tendencies do not tend to dominate. b) impairment of cognition usually is not present. c) antidepressant therapies are less effective in older adults. d) sadness of mood is usually present, but it is masked by other symptoms.

sadness of mood is usually present, but it is masked by other symptoms. Correct Explanation: Elderly clients are a high-risk group for depression. The classic symptoms of depression frequently are masked, and depression presents differently in the aging population. Depression in late life is underdiagnosed because the symptoms are incorrectly attributed to aging or medical problems. Impairment of cognition in a previously healthy elderly client or psychosomatic problems may be the presenting symptom of depression. Antidepressant therapy is usually effective.

A client is scheduled for cardiac catheterization the next morning. His physician ordered temazepam, 30 mg by mouth at bedtime, for sedation. Before administering the drug, the nurse should know that: a) sedatives cause predictable responses; hypnotics cause unpredictable ones. b) sedatives reduce excitement; hypnotics induce sleep. c) sedatives interact with few drugs; hypnotics interact with many. d) sedatives don't depress respirations; hypnotics do.

sedatives reduce excitement; hypnotics induce sleep. Explanation: Sedatives are drugs that act to reduce activity or excitement, calming a client. Hypnotics induce a state resembling natural sleep. Sedatives and hypnotics cause predictable responses, interact with many drugs, and can cause respiratory depression.

A nurse is caring for a client who has end-stage chronic obstructive pulmonary disease receiving IV push morphine for pain management. During rounds, the nurse discusses with the physician the need to begin the client on a continuous morphine infusion. The nurse bases this request on: a) ethnically death should come quick. b) an effort to assist the family cope with the end of life. c) servicing as a client advocate is an important role. d) increasing morphine is considered euthanasia.

servicing as a client advocate is an important role. Correct Explanation: Being an advocate is a major factor in caring for clients. Adequate pain relief, particularly for those with terminal illnesses, falls under this concept. Euthanasia is the deliberate act of hastening death. Increasing morphine to relieve the client's pain would not be a deliberate attempt to hasten death. Living wills do not dictate the amount of medication a client may receive. Suctioning is important, but will not assist with pain relief.

One goal in caring for a client with arterial occlusive disease is to promote vasodilation in the affected extremity. To achieve this goal, the nurse should encourage the client to: a) stop smoking. b) avoid eating low-fat foods. c) begin a jogging program. d) elevate the legs above the heart.

stop smoking. Correct Explanation: Nicotine causes vasospasm and impedes blood flow. Stopping smoking is the most significant lifestyle change the client can make. The client should eat low-fat foods as part of a balanced diet. The legs should not be elevated above the heart because this will impede arterial flow. The legs should be in a slightly dependent position. Jogging is not necessary and probably is not possible for many clients with arterial occlusive disease. A rehabilitation program that includes daily walking is suggested.

A school-age child with cystic fibrosis asks the nurse what sports she can become involved in as she becomes older. Which activity would be appropriate for the nurse to suggest? a) swimming b) track c) soccer d) baseball

swimming Correct Explanation: Swimming would be the most appropriate suggestion because it coordinates breathing and movement of all muscle groups and can be done on an individual basis or as a team sport. Because track events, baseball, and soccer usually are performed outdoors, the child would be breathing in large amounts of dust and dirt, which would be irritating to her mucous membranes and pulmonary system. The strenuous activity and increased energy expenditure associated with track events, in conjunction with the dust and possible heat, would play a role in placing the child at risk for an upper respiratory tract infection and compromising her respiratory function

When a nurse attempts to make sure the physician obtained informed consent for a thyroidectomy, she realizes the client doesn't fully understand the surgery. She approaches the physician, who curtly says, "I've told him all about it. Just get the consent." The nurse should: a) explain the procedure more fully to the client and obtain his signature. b) ask the charge nurse to talk with the physician. c) tell the physician the client isn't comfortable consenting to surgery at this point. d) tell the physician he didn't give the client enough information.

tell the physician the client isn't comfortable consenting to surgery at this point. Correct Explanation: The nurse has evaluated the client's knowledge concerning the surgery and determined that he doesn't have enough information to give informed consent. Even though the physician might want to move ahead, the nurse should advocate for the client by telling the physician the client isn't ready for the surgery. Telling the physician that he hasn't given the client enough information would be rude. The nurse shouldn't ask the charge nurse to talk with the physician unless the physician refuses to accept the nurse's professional opinion. Explaining surgery for the purpose of obtaining consent is beyond the nurse's scope of practice

When a patient is admitted to the hospital, admissions personnel are required to determine if the patient has a document indicating advanced directives. If so, a copy is made for the patient's medical record. The advanced directive document indicates: a) that an attorney has verified the living will papers. b) that the patient assigned a relative to act on their behalf. c) that he refuses to have resuscitation measures or any life-prolonging care. d) that the patient has made his wishes for terminal care known.

that the patient has made his wishes for terminal care known. Correct Explanation: It is important to determine if the patient has advanced directives, which indicate what he wants should he become incompetent or dying. Advanced directives may be documented in a living will or durable power of attorney for healthcare document. A copy should be placed in the patient's hospital record.

The client who has undergone a bilateral adrenalectomy is concerned about persistent body changes and unpredictable moods. The nurse should tell the client that: a) the body changes are permanent and the client will not be the same as before this condition. b) the physical changes are permanent, but the mood swings will disappear. c) the physical changes are temporary, but the mood swings are permanent. d) the body and mood will gradually return to normal.

the body and mood will gradually return to normal. Explanation: As the body readjusts to normal cortisol levels, mood and physical changes will gradually return to a normal state. The body changes are not permanent, and the mood swings should level off.

The nurse is preparing the client with a cerebrovascular accident for discharge to home. Which will influence the client's continuing progress in rehabilitation at home? a) frequency of follow-up visits with the health care provider b) the client's ability to ambulate c) availability of a home health aide to care for the client d) the family's ability to provide support to the client

the family's ability to provide support to the client Explanation: The strong support of family members is frequently identified as an important factor that influences a cerebrovascular accident client's continuing progress in rehabilitation after discharge. Discharge planning should prepare the client and family for the many changes necessary when the client returns home. Continuing progress in rehabilitation is not dependent upon the client's ability to ambulate. A client's continuing progress in rehabilitation is more dependent on the client and family support than the home health aide. Follow-up visits with the health care provider are important but are not the greatest influence on the client's progress with rehabilitation.

The nurse is preparing a client for nonemergency surgery. The nurse should: a) inform the client about the risks of the surgery to be performed. b) verify the client understands the informed consent form. c) obtain informed consent from the client. d) explain the surgical procedure

verify the client understands the informed consent form. Correct Explanation: The surgeon is responsible for explaining the surgical procedure to be performed and the risks of the procedure, as well as for obtaining the informed consent from the client. A nurse may be responsible for obtaining and witnessing a client's signature on the consent form. The nurse is the client's advocate, verifying that a client (or family member) understands the consent form and its implications, and that consent for the surgery is truly voluntary

The nurse is conducting a health history with a client with active tuberculosis. The nurse should ask the client about: a) dyspnea on exertion. b) increased appetite. c) mental status changes. d) weight loss.

weight loss. Explanation: Tuberculosis typically produces anorexia and weight loss. Other signs and symptoms may include fatigue, low-grade fever, and night sweats.

A nursing instructor is explaining the benefits of bathing to a group of nursing students. She states there are numerous benefits beyond hygiene. A student understands the concepts when she lists the following benefits orally to the class. Select all that apply. a) Bathing decreases joint mobility if the client is on bed rest. b) Bathing can improve appearance and self-image. c) Bathing removes organisms from the skin, reducing infection. d) Bathing decreases comfort because clients are being turned and re-positioned. e) Bathing can stimulate circulation.

• Bathing can stimulate circulation. • Bathing removes organisms from the skin, reducing infection. • Bathing can improve appearance and self-image. Correct Explanation: Bathing serves a variety of purposes including: cleansing; acting as a skin conditioner; helping to relax a person; promoting circulation by stimulating the skin?s peripheral nerve endings and underlying tissues; serving as a musculoskeletal exercise through activity involved in bathing, thereby improving joint mobility and muscle tone; stimulating the rate and depth of respiration; promoting comfort through muscle relaxation and skin stimulation; providing sensory input; and helping improve self-image. Bathing also provides a means to establishing a therapeutic relationship.

A client is undergoing a surgical procedure to repair his ulcerated colon. During your care, you discuss at length pertinent information for his condition peri operatively. Which of the following client education topics will be discussed preoperatively? Select all that apply. a) Intravenous fluids and other lines and tubes b) Postoperative pain control c) His wife's thoughts about the upcoming surgery d) Cough and deep-breathing exercises e) The surgeon's fee and other hospital charges

• Cough and deep-breathing exercises • Postoperative pain control • Intravenous fluids and other lines and tubes Correct Explanation: Preoperative teaching involves teaching clients about their upcoming surgical procedure and expectations. Topics include preoperative medications (when they are given and their effects); postoperative pain control; explanation and description of the post anesthesia recovery room or postsurgical area; and deep-breathing and coughing exercises.

A client is refusing to take the prescribed oral medication. Which of the following measures by the nurse can be used to get the client to take the medication? Select all that apply. a) Suggesting a liquid form of the medication instead of a pill b) Having a family member give the medication c) Crushing the medication and hiding it in apple sauce d) Explaining the purpose of the medication to the client e) Asking the client the reason for not taking the medication

• Explaining the purpose of the medication to the client • Asking the client the reason for not taking the medication • Suggesting a liquid form of the medication instead of a pill Correct Explanation: The correct answers provide an alternative solution for the client and provide the client an opportunity to consent to taking the medication in another form, neither of which would be considered abuse. Providing health education regarding the medications to ensure the client has all the information needed to make an informed consent would be appropriate. Hiding medication or disguising it in food knowing that the client has refused the medication would be considered abuse. The client has the right to refuse care, including medication, and a family member should not be placed in a position of having to give the medication.

A client has been diagnosed with an adjustment disorder with mixed anxiety and depression. What are the primary nursing diagnoses the nurse would associate with this type of adjustment disorder? Select all that apply. a) Acute confusion. b) Activity intolerance. c) Impaired social interaction. d) Risk for situational low self-esteem. e) Impaired memory. f) Disturbed personal identity.

• Impaired social interaction. • Risk for situational low self-esteem. Explanation: Adjustment disorder occurs when an individual is unable to adjust or cope with a particular stressor such as a major life event. A client with an adjustment disorder is likely to exhibit impaired social interaction and risk for situational low self-esteem. Primary nursing diagnosis would not include the others as the client is able to participate in activities, if desired, and has no deficits in cognition or personal identity.

A nurse is caring for a client diagnosed with persistent depressive disorder. Which defining characteristics are associated with this disorder? Select all that apply. a) Symptoms that occur in the winter and resolve in spring. b) Onset of symptoms within a 2-week period. c) Appetite disturbance. d) Insomnia or hypersomnia. e) Suicidal thoughts. f) Delusions or hallucinations.

• Insomnia or hypersomnia. • Suicidal thoughts. • Appetite disturbance. Explanation: Persistent depressive disorder is a mild, but chronic, form of depression. Sleep and appetite disturbances and suicidal thoughts can appear in clients with this or major depressive disorders. Onset of symptoms are gradual and may appear over weeks or months. Delusions and other psychotic symptoms may occur in major depression but do not occur in persistent depressive disorder. Episodes of depression occurring solely in the winter are indicative of seasonal affective disorder.

The nurse advises a client recovering from a myocardial infarction to decrease fat and sodium intake. Which foods should the nurse instruct the client to avoid? Select all that apply. a) Bacon b) Oatmeal c) Apple juice d) Soft drinks e) Cheese f) Pepperoni pizza

• Pepperoni pizza • Bacon • Cheese • Soft drinks Explanation: Foods high in sodium include cheese, processed meats such as pepperoni and bacon, and soft drinks. Bacon and cheese also have a high fat content.

A school nurse is using Havighurst's developmental theory to teach parents of adolescents what to expect at this developmental stage. Which behaviors are typical of adolescents? (Select all that apply.) a) The adolescent learns physical skills necessary for games. b) The adolescent learns to get along with age-mates. c) The adolescent acquires an ethical system as a guide to behavior. d) The adolescent achieves social and civic responsibility. e) The adolescent accepts his or her body and uses it effectively. f) The adolescent achieves emotional independences from parents.

• The adolescent accepts his or her body and uses it effectively. • The adolescent achieves emotional independences from parents. • The adolescent acquires an ethical system as a guide to behavior. Explanation: Behaviors typical of adolescents would include acceptance of his or her body and use of it effectively, achieving emotional independence from parents, and acquiring an ethical system as a guide to behavior. Learning the physical skills necessary for games is typical of behaviors in middle childhood. Learning to get along with age-mates is typical of behaviors in middle childhood. Achieving social and civic responsibility is typical of behaviors in middle adulthood.

The nurse is obtaining a health history for a client with osteoporosis. What should the nurse ask the client about? Select all that apply. a) use of Vitamin K supplements b) amount of alcohol consumed daily c) intake of fruit juices d) use of antacids e) dietary intake of fiber

• amount of alcohol consumed daily • use of antacids • dietary intake of fiber • use of Vitamin K supplements Explanation: The nurse should ask the client about alcohol use because heavy alcohol use causes fluid excretion resulting in heavy losses of calcium in urine. If the client uses antacids containing aluminum or magnesium, a net loss of calcium can occur. If the client has a high-fiber diet, the fiber can bind up some of the dietary calcium. People with hip fractures have been found to have low vitamin K intakes; vitamin K plays an important role in production of at least one bone protein. Fruit juices do not affect calcium absorption.


Conjuntos de estudio relacionados

Principles of Microeconomics (Ch. 1,2,3,4,10)

View Set

Real Estate Principles Chapter 18

View Set

Public Speaking Exam #3 (Chapters 12, 13, 14, 16, 17, 18 & 19)

View Set

Unit 1 - Solving Equations and Inequalities Practice Scrimmage

View Set

English-We Ate The Children Last

View Set

AP Art History Oceania and Indigenous Americas

View Set

Nursing Fundamentals NUR 120D Exam 1

View Set

SAT Practice 1 - October 11, 2023 Reading and Writing: Question 1

View Set